All Study Questions for NR-226

Ace your homework & exams now with Quizwiz!

A nurse is prioritizing care. Match the level of priority to the patients. a. Patient that needs to be turned to prevent pneumonia b. Patient with acute asthma attack c. Patient who will be discharged in 2 days who needs teaching 1. High priority 2. Intermediate priority 3. Low priority

1. b 2. a 3. c

A nurse is monitoring patients for fluid and electrolyte and acid-base imbalances. Match the body's regulators to the function it provides. a. Increases excretion of sodium and water b. Reduces excretion of sodium and water c. Reduces excretion of water d. Major buffer in the extracellular fluid e. Vasoconstricts and stimulates aldosterone release 1.Antidiuretic hormone 2.Angiotensin II 3.Aldosterone 4.Atrial natriuretic peptide 5.Bicarbonate

1.ANS:C 2.ANS:E 3.ANS:B 4.ANS:A 5.ANS:D

A nurse is caring for a group of patients. Which evaluative measures will the nurse use to determine a patient's responses to nursing care? (Select all that apply.) a. Observations of wound healing b. Daily blood pressure measurements c. Findings of respiratory rate and depth d. Completion of nursing interventions e. Patient's subjective report of feelings about a new diagnosis of cancer

A, B, C, E (You examine the results of care by using evaluative measures, which are assessment skills and techniques (e.g., observations, physiological measurements, use of measurement scales, and patient interview). Examples of evaluative measures include assessment of wound healing and respiratory status, blood pressure measurement, and assessment of patient feelings. You conduct evaluative measures to determine if your patients met expected outcomes, not if nursing interventions were completed.)

12. A nurse performs an assessment on a patient. Which assessment data will the nurse use as an etiology for Acute pain? a.Discomfort while changing position b.Reports pain as a 7 on a 0 to 10 scale c.Disruption of tissue integrity d.Dull headache

ANS: C Disruption of tissue integrity is a possible cause or etiology of pain. A report of pain, headache, and discomfort are examples of things a patient might say (subjective data or defining characteristics) that may lead a nurse to select Acute pain as a nursing diagnosis.

A nurse identifies a fall risk when assessing a patient upon admission. The nurse and the patient agree that the goal is for the patient to remain free from falls. However, the patient fell just before shift change. Which action is the nurse's priority when evaluating the patient? a. Identify factors interfering with goal achievement. b. Counsel the nursing assistive personnel on duty when the patient fell. c. Remove the fall risk sign from the patient's door because the patient has suffered a fall. d. Request that the more experienced charge nurse complete the documentation about the fall.

A ( When goals and outcomes are not met, you identify the factors that interfere with their achievement. The nurse identifies factors that interfered with goal achievement to determine the cause of the fall. The fall may not have been due to an error by the nursing assistive personnel; therefore, counseling should be reserved until after the cause has been determined. The patient remains a fall risk, so the fall risk sign should remain on the door. The nurse witnessing the fall or the nurse assigned to the patient needs to complete the documentation. The charge nurse can be consulted to review the documentation.)

A goal for a patient with diabetes is to demonstrate effective coping skills. Which patient behavior will indicate to the nurse achievement of this outcome? a. States feels better after talking with family and friends b. Consumes high-carbohydrate foods when stressed c. Dislikes the support group meetings d. Spends most of the day in bed

A (Evaluative data that show signs of effective coping will help the nurse determine whether the patient has met the outcome. Talking to family and friends is the only positive option. During evaluation, you perform evaluative measures that allow you to compare clinical data, patient behavior measures, and patient self-report measures collected before implementation with the evaluation findings gathered after administering nursing care. Next, you evaluate whether the results of care match the expected outcomes and goals set for a patient. Consuming high-carbohydrate foods (patient is a diabetic), disliking support group, and spending the day in bed indicate unsuccessful progress toward meeting the patient's goal.)

A nurse is evaluating an expected outcome for a patient that states heart rate will be less than 80 beats/min by 12/3. Which finding will alert the nurse that the goal has been met? a. Heart rate 78 beats/min on 12/3 b. Heart rate 78 beats/min on 12/4 c. Heart rate 80 beats/min on 12/3 d. Heart rate 80 beats/min on 12/4

A (Heart rate 78 beats/min on 12/3 indicates the goal has been met. Comparing expected and actual findings allows you to interpret and judge a patient's condition and whether predicted changes have occurred. Expected outcome states less than 80, not 80. The date is by 12/3, not 12/4.)

A nurse has instituted a turn schedule for a patient to prevent skin breakdown. Upon evaluation, the nurse finds that the patient has a stage II pressure ulcer on the buttocks. Which action will the nurse take next? a. Reassess the patient and situation. b. Revise the turning schedule to increase the frequency. c. Delegate turning to the nursing assistive personnel. d. Apply medication to the area of skin that is broken down.

A (If a nursing diagnosis is unresolved or if you determine that a new problem has perhaps developed, reassessment is necessary. A complete reassessment of patient factors relating to an existing nursing diagnosis and etiology is necessary when modifying a plan. The nurse must assess before revising, delegating and applying medication. The breakdown may be a result of inadequate nutritional intake and medication cannot be applied unless there is an order.)

1.A patient has 250 mL of a jejunostomy feeding with 30 mL of water before and after feeding and 200 mL of urine. Thirty minutes later the patient has 100 mL of diarrhea. At 1300 the patient receives 150 mL of blood and voids another 200 mL. Calculate the patient's intake. Record your answer as a whole number. _____ mL

ANS: 460 The patient's fluid intake is 250 mL of feeding, 60 mL of water (30 mL before and after), and 150 blood: . Fluid intake includes all liquids that a person eats (e.g., gelatin, ice cream, soup), drinks (e.g., water, coffee, juice), or receives through nasogastric or jejunostomy feeding tubes. IV fluids (continuous infusions and intermittent IV piggybacks) and blood components also are sources of intake. Fluid output includes urine, diarrhea, vomitus, gastric suction, and drainage from postsurgical wounds or other tubes. DIF:Apply (application)REF:949 | 951 | 955

32. The nurse is caring for a patient in the operating suite who is experiencing hypercarbia, tachypnea, tachycardia, premature ventricular contractions, and muscle rigidity. Which condition does the nurse suspect the patient is experiencing? a. Malignant hyperthermia b. Fluid imbalance c. Hemorrhage d. Hypoxia

ANS: A A life-threatening, rare complication of anesthesia is malignant hyperthermia. Malignant hyperthermia causes hypercarbia, tachycardia, tachypnea, premature ventricular contractions, unstable blood pressure, cyanosis, skin mottling, and muscular rigidity. It often occurs during anesthesia induction. Hypoxia would manifest with decreased oxygen saturation as one of its signs and symptoms. Fluid imbalance would be assessed with intake and output and can manifest with tachycardia and blood pressure fluctuations but does not have muscle rigidity. Hemorrhage can manifest with tachycardia and decreased blood pressure, along with a thready pulse. Usually some sign or symptom of blood loss is noted (e.g., drains, incision, orifice, and abdomen).

29. The nurse is caring for a patient in the operating suite. Which outcome will be most appropriate for this patient at the end of the intraoperative phase? a. The patient will be free of burns at the grounding pad. b. The patient will be free of nausea and vomiting. c. The patient will be free of infection. d. The patient will be free of pain.

ANS: A A primary focus of intraoperative care is to prevent injury and complications related to anesthesia, surgery, positioning, and equipment use, including use of the electrical cautery grounding pad for the prevention of burns. The perioperative nurse is an advocate for the patient during surgery and protects the patient's dignity and rights at all times. Signs and symptoms of infection do not have the time to present during the intraoperative phase. During the intraoperative phase, the patient is anesthetized and unconscious and typically has an endotracheal tube that prevents conversation. Nausea, vomiting, and pain typically begin in the postoperative phase of the experience.

3. The standing orders for a patient include acetaminophen 650 mg every 4 hours prn for headache. After assessing the patient, the nurse identifies the need for headache relief and determines that the patient has not had acetaminophen in the past 4 hours. Which action will the nurse take next? a.Administer the acetaminophen. b.Notify the health care provider to obtain a verbal order. c.Direct the nursing assistive personnel to give the acetaminophen. d.Perform a pain assessment only after administering the acetaminophen.

ANS: A A standing order is a preprinted document containing orders for the conduct of routine therapies, monitoring guidelines, and/or diagnostic procedures for specific patients with identified clinical problems. The nurse will administer the medication. Notifying the health care provider is not necessary if a standing order exists. The nursing assistive personnel are not licensed to administer medications; therefore, medication administration should not be delegated to this person. A pain assessment should be performed before and after pain medication administration to assess the need for and effectiveness of the medication.

7. The nurse is reviewing a patient's database for significant changes and discovers that the patient has not voided in over 8 hours. The patient's kidney function lab results are abnormal, and the patient's oral intake has significantly decreased since previous shifts. Which step of the nursing process should the nurse proceed to after this review? a.Diagnosis b.Planning c.Implementation d.Evaluation

ANS: A After a thorough assessment, the nurse should proceed to analyzing the data and formulating a nursing diagnosis before proceeding with developing the plan of care and determining appropriate interventions; this is the diagnosis phase. The evaluation phase involves determining whether the goals were met and interventions were effective.

1. A nurse encounters a family who experienced the death of their adult child last year. The parents are talking about the upcoming anniversary of their child's death. The nurse spends time with them discussing their child's life and death. Which nursing principle does the nurse's action best demonstrate? a. Facilitation of normal mourning b. Pain-management technique c. Grief evaluation d. Palliative care

ANS: A Anniversary reactions can reopen grief processes. A nurse should openly acknowledge the loss and talk about the common renewal of grief feeling around the anniversary of the individual's death; this facilitates normal mourning. The nurse is not attempting to alleviate a physical pain. The actions are of open communication, not evaluation. Palliative care refers to comfort measures for symptom relief.

4. Which action indicates a nurse is using critical thinking for implementation of nursing care to patients? a.Determines whether an intervention is correct and appropriate for the given situation b.Reads over the steps and performs a procedure despite lack of clinical competency c.Establishes goals for a particular patient without assessment d.Evaluates the effectiveness of interventions

ANS: A As you implement interventions, use critical thinking to confirm whether the interventions are correct and still appropriate for a patient's clinical situation. You are responsible for having the necessary knowledge and clinical competency to perform interventions for your patients safely and effectively. The nurse needs to recognize the safety hazards of performing an intervention without clinical competency and seek assistance from another nurse. The nurse cannot evaluate interventions until they are implemented. Patients need ongoing assessment before establishing goals because patient conditions can change very rapidly.

4. A nurse suspects an older-adult patient is experiencing caregiver neglect. Which assessment findings are consistent with the nurse's suspicions? a.Flea bites and lice infestation b.Left at a grocery store c.Refuses to take a bath d.Cuts and bruises

ANS: A Caregiver neglect includes unsafe and unclean living conditions, soiled bedding, and animal or insect infestation. Abandonment includes desertion at a hospital, nursing facility, or public location such as a shopping center. Self-neglect includes refusal or failure to provide oneself with basic necessities such as food, water, clothing, shelter, personal hygiene, medication, and safety. Physical abuse includes hitting, beating, pushing, slapping, kicking, physical restraint, inappropriate use of drugs, fractures, lacerations, rope burns, and untreated injuries.

23. Which assessment finding of an older adult, who has a urinary tract infection, requires an immediate nursing intervention? a.Confusion b.Presbycusis c.Temperature of 97.9° F d.Death of a spouse 2 months ago

ANS: A Confusion is a common manifestation in older adults with urinary tract infection; however, the cause requires further assessment. There may be another reason for the confusion. Confusion is not a normal finding in the older adult, even though it is commonly seen with concurrent infections. Difficulty hearing, presbycusis, is an expected finding in an older adult. Older adults tend to have lower core temperatures. Coping with the death of a spouse is a psychosocial concern to be addressed after the acute physiological concern in this case.

12. A patient has had two family members die during the past 2 days. Which coping strategy is most appropriate for the nurse to suggest to the patient? a. Writing in a journal b. Drinking alcohol to go to sleep c. Exercising vigorously rather than sleeping d. Avoiding talking with friends and family members

ANS: A Coping strategies may be healthy and effective like talking, journaling, and sharing emotions with others. They may also be unhealthy and ineffective like increased use of alcohol, drugs, and violence. Although exercise is important for self-care, sleep is also important. Shutting oneself away from friends and family by not talking about the sadness is not effective; the patient should spend time with people who are nurturing.

15. A nurse adds a nursing diagnosis to a patient's care plan. Which information did the nurse document? a.Decreased cardiac output related to altered myocardial contractility. b.Patient needs a low-fat diet related to inadequate heart perfusion. c.Offer a low-fat diet because of heart problems. d.Acute heart pain related to discomfort.

ANS: A Decreased cardiac output related to altered myocardial contractility is a correctly written nursing diagnosis. Patient needs a low-fat diet related to inadequate heart perfusion is a goal phrased statement, not a nursing diagnosis. Offer a low-fat diet is an intervention, not a diagnosis. Acute pain related to discomfort is a circular diagnosis and gives no direction to nursing care.

35. The nurse is caring for a patient in the postanesthesia care unit. The patient asks for a bedpan and states to the nurse, "I feel like I need to go to the bathroom, but I can't." Which nursing intervention will be most appropriate initially? a. Assess the patient for bladder distention. b. Encourage the patient to wait a minute and try again. c. Inform the patient that everyone feels this way after surgery. d. Call the health care provider to obtain an order for catheterization.

ANS: A Depending on the surgery, some patients do not regain voluntary control over urinary function for 6 to 8 hours after anesthesia. Palpate the lower abdomen just above the symphysis pubis for bladder distention. Another option is to use a bladder scan or ultrasound to assess bladder volume. The nurse must assess before deciding if the patient can try again. Not everyone feels as if they need to go but can't after surgery. Calling the health care provider is not the initial best action. The nurse needs to have data before calling the provider.

38. The nurse is caring for a group of patients. Which patient will the nurse see first? a. A patient who had cataract surgery is coughing. b. A patient who had vascular repair of the right leg is not doing right leg exercises. c. A patient after knee surgery is wearing intermittent pneumatic compression devices and receiving heparin. d. A patient after surgery has vital signs taken every 15 minutes twice, every 30 minutes twice, hourly for 2 hours then every 4 hours.

ANS: A For patients who have had eye, intracranial, or spinal surgery, coughing may be contraindicated because of the potential increase in intraocular or intracranial pressure. The nurse will need to see this patient first to control the cough and intraocular pressure. All the rest are normal postoperative patients. Leg exercise should not be performed on the operative leg with vascular surgery. A patient after knee surgery should receive heparin and be wearing intermittent pneumatic compression devices; while the nurse will check on the patient, it does not have to be first. Monitoring vital signs after surgery is required and this is the standard schedule.

15. A patient with an indwelling urinary catheter has been given a bed bath by a new nursing assistive personnel. The nurse evaluating the cleanliness of the patient notices crusting at the urinary meatus. Which action should the nurse take next? a. Ask the nursing assistive personnel to observe while the nurse performs catheter care. b. Leave the room and ask the nursing assistive personnel to go back and perform proper catheter care. c. Tell the nursing assistive personnel that catheter care is sloppy. d. Remove the catheter.

ANS: A If the staff member's performance is not satisfactory, give constructive and appropriate feedback. You may discover the need to review a procedure with staff and offer demonstration. Because the nursing assistant is new, it is best for the nurse to perform catheter care while the assistant observes. This action will ensure that the assistant has been shown the proper way to perform the task and fosters collaboration rather than leaving the room just to tell the assistant to come back. Telling that catheter care is sloppy does not correct the problem. The catheter does not need to be removed.

6. The nurse is caring for a patient in the postanesthesia care unit who has undergone a left total knee arthroplasty. The anesthesia provider has indicated that the patient received a left femoral peripheral nerve block. Which assessment will be an expected finding for this patient? a. Sensation decreased in the left leg b. Patient report of pain in the left foot c. Pulse decreased at the left posterior tibia d. Left toes cool to touch and slightly cyanotic

ANS: A Induction of regional anesthesia results in loss of sensation in an area of the body—in this case, the left leg. The peripheral nerve block influences the portions of sensory pathways that are anesthetized in the targeted area of the body. Decreased pulse, toes cool to touch, and cyanosis are indications of decreased blood flow and are not expected findings. Reports of pain in the left foot may indicate that the block is not working or is subsiding and is not an expected finding in the immediate postoperative period.

8. The nurse is completing a medication history for the surgical patient in preadmission testing. Which medication should the nurse instruct the patient to hold (discontinue) in preparation for surgery according to protocol? a. Warfarin b. Vitamin C c. Prednisone d. Acetaminophen

ANS: A Medications such as warfarin or aspirin alter normal clotting factors and thus increase the risk of hemorrhaging. Discontinue at least 48 hours before surgery. Acetaminophen is a pain reliever that has no special implications for surgery. Vitamin C actually assists in wound healing and has no special implications for surgery. Prednisone is a corticosteroid, and dosages are often temporarily increased rather than held.

20. The nurse is revising the care plan. In which order will the nurse perform the tasks, beginning with the first step? 1. Revise specific interventions. 2. Revise the assessment column. 3. Choose the evaluation method. 4. Delete irrelevant nursing diagnoses. a.2, 4, 1, 3 b.4, 2, 1, 3 c.3, 4, 2, 1 d.4, 2, 3, 1

ANS: A Modification of an existing written care plan includes four steps: 1. Revise data in the assessment column to reflect the patient's current status. Date any new data to inform other members of the health care team of the time that the change occurred. 2. Revise the nursing diagnoses. Delete nursing diagnoses that are no longer relevant and add and date any new diagnoses. Revise related factors and the patient's goals, outcomes, and priorities. Date any revisions. 3. Revise specific interventions that correspond to the new nursing diagnoses and goals. Be sure that revisions reflect the patient's present status. 4. Choose the method of evaluation for determining whether you achieved patient outcomes.

3. Which information from a co-worker on a gerontological unit will cause the nurse to intervene? a.Most older people have dependent functioning. b.Most older people have strengths we should focus on. c.Most older people should be involved in care decision. d.Most older people should be encouraged to have independence.

ANS: A Most older people remain functionally independent despite the increasing prevalence of chronic disease; therefore, this misconception should be addressed. It is critical for you to respect older adults and actively involve them in care decisions and activities. You also need to identify an older adult's strengths and abilities during the assessment and encourage independence as an integral part of your plan of care.

17. A staff development nurse is providing an inservice for other nurses to educate them about the Nursing Interventions Classification (NIC) system. During the inservice, which statement made by one of the nurses in the room requires the staff development nurse to clarify the information provided? a."This system can help medical students determine the cost of the care they provide to patients." b."If the nursing department uses this system, communication among nurses who work throughout the hospital may be enhanced." c."We could use this system to help organize orientation for new nursing employees because we can better explain the nursing interventions we use most frequently on our unit." d."The NIC system provides one way to improve safe and effective documentation in the hospital's electronic health record."

ANS: A NIC does not help determine the cost of services provided by nurses. The staff development nurse would need to correct this misconception. Because this system is specific to nursing practice, it would not help medical students determine the costs of care. The NIC system developed by the University of Iowa differentiates nursing practice from that of other health care disciplines. All the other statements are true. Benefits of using NIC include enhancing communication among nursing staff and documentation, especially within health information systems such as an electronic documentation system. NIC also helps nurses identify the nursing interventions they implement most frequently. Units that identify routine nursing interventions can use this information to develop checklists for orientation.

31. The nurse is assessing a postoperative patient with a history of obstructive sleep apnea for airway obstruction. Which assessment finding will best alert the nurse to this complication? a. Drop in pulse oximetry readings b. Moaning with reports of pain c. Shallow respirations d. Disorientation

ANS: A One of the greatest concerns after general anesthesia is airway obstruction, especially in patients with obstructive sleep apnea. A drop in oxygen saturation by pulse oximetry is a sign of airway obstruction in patients with obstructive sleep apnea. Weak pharyngeal/laryngeal muscle tone from anesthetics; secretions in the pharynx, bronchial tree, or trachea; and laryngeal or subglottic edema also contribute to airway obstruction. In the postanesthetic patient, the tongue is a major cause of airway obstruction. Shallow respirations are indicative of respiratory depression. Moaning and reports of pain are common in all surgical patients and are an expected event. Disorientation is common when first awakening from anesthesia but can be a sign of hypoxia.

19. The nurse is providing preoperative teaching for the ambulatory surgery patient who will be having a cyst removed from the right arm. Which will be the best explanation for diet progression after surgery? a. "Start with clear liquids, soup, and crackers. Advance to a normal diet as tolerated." b. "Stay with ice chips for several hours. After that, you can have whatever you want." c. "Stay on clear liquids for 24 hours. Then you can progress to a normal diet." d. "Start with clear liquids for 2 hours and then full liquids for 2 hours. Then progress to a normal diet."

ANS: A Patients usually receive a normal diet the first evening after surgery unless they have undergone surgery on GI structures. Implement diet intake while judging the patient's response. For example, provide clear liquids such as water, apple juice, broth, or tea after nausea subsides. If the patient tolerates liquids without nausea, advance the diet as ordered. There is no need to stay on ice chips for several hours or clear liquids for 24 hours after this procedure. Putting a time frame on the progression is too prescriptive. Progression should be adjusted for the patient's needs.

16. An older patient with dementia and confusion is admitted to the nursing unit after hip replacement surgery. Which action will the nurse include in the plan of care? a.Keep a routine. b.Continue to reorient. c.Allow several choices. d.Socially isolate patient.

ANS: A Patients with dementia need a routine. Continuing to reorient a patient with dementia is nonproductive and not advised. Patients with dementia need limited choices. Social interaction based on the patient's abilities is to be promoted.

17. The nurse is preparing to assist the patient in using the incentive spirometer. Which nursing intervention should the nurse provide first? a. Perform hand hygiene. b. Explain use of the mouthpiece. c. Instruct the patient to inhale slowly. d. Place in the reverse Trendelenburg position.

ANS: A Performing hand hygiene reduces microorganisms and should be performed first. Placing the patient in the correct position such as high Fowler's for the typical postoperative patient or reverse Trendelenburg for the bariatric patient would be the next step in the process. Demonstration of use of the mouthpiece followed by the instruction to inhale slowly would be the last step in this scenario.

13. Which nursing assessment will indicate the patient is performing diaphragmatic breathing correctly? a. Hands placed on the border of the rib cage with fingers extended will touch as the chest wall contracts. b. Hands placed on the chest wall with fingers extended will separate as the chest wall contracts. c. The patient will feel upward movement of the diaphragm during inspiration. d. The patient will feel downward movement of the diaphragm during expiration.

ANS: A Positioning the hands along the borders of the rib cage allows the patient to feel movement of the chest and abdomen as the diaphragm descends and the lungs expand. As the patient takes a deep breath and slowly exhales, the middle fingers will touch while the chest wall contracts. The fingers will separate as the chest wall expands. The patient will feel normal downward movement of the diaphragm during inspiration and normal upward movement during expiration.

20. An outcome for an older-adult patient living alone is to be free from falls. Which statement indicates the patient correctly understands the teaching on safety concerns? a."I'll take my time getting up from the bed or chair." b."I should dim the lighting outside to decrease the glare in my eyes." c."I'll leave my throw rugs in place so that my feet won't touch the cold tile." d."I should wear my favorite smooth bottom socks to protect my feet when walking around."

ANS: A Postural hypotension is an intrinsic factor that can cause falls. Changing positions slowly indicates a correct understanding of this concept. Environmental hazards outside and within the home such as poor lighting, slippery or wet flooring, and items on floor that are easy to trip over such as throw rugs are other factors that can lead to falls. Impaired vision and poor lighting are other risk factors for falls and should be avoided (dim lighting). Inappropriate footwear such as smooth bottom socks also contributes to falls.

13. Which initial intervention is most appropriate for a patient who has a new onset of chest pain? a.Reassess the patient. b.Notify the health care provider. c.Administer a prn medication for pain. d.Call radiology for a portable chest x-ray.

ANS: A Preparation for implementation ensures efficient, safe, and effective nursing care; the first activity is reassessment. The cause of the patient's chest pain is unknown, so the patient needs to be reassessed before pain medication is administered or a chest x-ray is obtained. The nurse then notifies the patient's health care provider of the patient's current condition in anticipation of receiving further orders. The patient's chest pain could be due to muscular injury or a pulmonary issue. The nurse needs to reassess first.

22. An older-adult patient has developed acute confusion. The patient has been on tranquilizers for the past week. The patient's vital signs are normal. What should the nurse do? a.Take into account age-related changes in body systems that affect pharmacokinetic activity. b.Increase the dose of tranquilizer if the cause of the confusion is an infection. c.Note when the confusion occurs and medicate before that time. d.Restrict phone calls to prevent further confusion.

ANS: A Some sedatives and tranquilizers prescribed for acutely confused older adults sometimes cause or exacerbate confusion. Carefully administer drugs used to manage confused behaviors, taking into account age-related changes in body systems that affect pharmacokinetic activity. When confusion has a physiological cause (such as an infection), specifically treat that cause, rather than the confused behavior. When confusion varies by time of day or is related to environmental factors, nonpharmacological measures such as making the environment more meaningful, providing adequate light, etc., should be used. Making phone calls to friends or family members allows older adults to hear reassuring voices, which may be beneficial.

21. A nurse's goal for an older adult is to reduce the risk of adverse medication effects. Which action will the nurse take? a.Review the patient's list of medications at each visit. b.Teach that polypharmacy is to be avoided at all cost. c.Avoid information about adverse effects. d.Focus only on prescribed medications.

ANS: A Strategies for reducing the risk for adverse medication effects include reviewing the medications with older adults at each visit; examining for potential interactions with food or other medications; simplifying and individualizing medication regimens; taking every opportunity to inform older adults and their families about all aspects of medication use; and encouraging older adults to question their health care providers about all prescribed and over-the-counter medications. Although polypharmacy often reflects inappropriate prescribing, the concurrent use of multiple medications is often necessary when an older adult has multiple acute and chronic conditions. Older adults are at risk for adverse drug effects because of age-related changes in the absorption, distribution, metabolism, and excretion of drugs. Work collaboratively with the older adult to ensure safe and appropriate use of all medications—both prescribed medications and over-the-counter medications and herbal options.

22. The nurse is reviewing the surgical consent with the patient during preoperative education and finds the patient does not understand what procedure will be completed. What is the nurse's best next step? a. Notify the health care provider about the patient's question. b. Explain the procedure that will be completed. c. Continue with preoperative education. d. Ask the patient to sign the form.

ANS: A Surgery cannot be legally or ethically performed until the patient fully understands the need for a procedure and all the implications. It is the surgeon's responsibility to explain the procedure, associated risks, benefits, alternatives, and possible complications. It is important for the nurse to pause with preoperative education to notify the health care provider of the patient's questions. It is not within the nurse's scope to explain the procedure. The nurse can certainly reinforce what the health care provider has explained, but the information needs to come from the health care provider. It is not prudent to ask a patient to sign a form for a procedure that he/she does not understand.

19. The nurse is intervening for a patient that has a risk for a urinary infection. Which direct care nursing intervention is most appropriate? a.Teaches proper handwashing technique b.Properly cleans the patient's toilet c.Transports urine specimen to the lab d.Informs the oncoming nurse during hand-off

ANS: A Teaching proper handwashing technique is a direct care nursing intervention. All the rest are indirect nursing care: cleaning the toilet, transporting specimens, and performing hand-off reports.

20. A nurse is developing nursing diagnoses for a patient. Beginning with the first step, place in order the steps the nurse will use. 1. Observes the patient having dyspnea (shortness of breath) and a diagnosis of asthma 2. Writes a diagnostic label of impaired gas exchange 3. Organizes data into meaningful clusters 4. Interprets information from patient 5. Writes an etiology a.1, 3, 4, 2, 5 b.1, 3, 4, 5, 2 c.1, 4, 3, 5, 2 d.1, 4, 3, 2, 5

ANS: A The diagnostic process flows from the assessment process (observing and gathering data) and includes decision-making steps. These steps include data clustering, identifying patient health problems, and formulating the diagnosis (diagnosis is written as problem or NANDA-I approved diagnosis then etiology or cause).

9. A nurse administers an antihypertensive medication to a patient at the scheduled time of 0900. The nursing assistive personnel (NAP) then reports to the nurse that the patient's blood pressure was low when it was taken at 0830. The NAP states that was busy and had not had a chance to tell the nurse yet. The patient begins to complain of feeling dizzy and light-headed. The blood pressure is rechecked and it has dropped even lower. In which phase of the nursing process did the nurse first make an error? a.Assessment b.Diagnosis c.Implementation d.Evaluation

ANS: A The diagnostic process should flow from the assessment. In this case, the nurse should have assessed the patient's blood pressure before giving the medication. The nurse could have prevented the patient's untoward reaction if the low blood pressure was assessed first. Diagnosis follows assessment. Administering the medication occurs in implementation, but this is not the first error. There are no errors in evaluation.

4. The nurse is reviewing a patient's plan of care, which includes the nursing diagnostic statement, Impaired physical mobility related to tibial fracture as evidenced by patient's inability to ambulate. Which part of the diagnostic statement does the nurse need to revise? a.Etiology b.Nursing diagnosis c.Collaborative problem d.Defining characteristic

ANS: A The etiology, or related to factor, of tibial fracture is a medical diagnosis and needs to be revised. The nursing diagnosis is appropriate because the patient is unable to ambulate. A collaborative problem is an actual or potential physiological complication that nurses monitor to detect the onset of changes in a patient's health status; there is no collaborative problem listed. The defining characteristic (subjective and objective data that support the diagnosis) is appropriate for Impaired physical mobility.

25. The nurse has completed a preoperative assessment for a patient going to surgery and gathers assessment data. Which will be the most important next step for the nurse to take? a. Notify the operating suite that the patient has a latex allergy. b. Document that the patient had a bath at home this morning. c. Administer the ordered preoperativ intravenous antibiotic. d. Ask the nursing assistive personnel to obtain vital signs.

ANS: A The most important step is notifying the operating suite of the patient's latex allergy. Many products that contain latex are used in the operating suite and the postanesthesia care unit (PACU). When preparing for a patient with this allergy, special considerations are required from preparation of the room to the types of tubes, gloves, drapes, and instruments utilized. Obtaining vital signs, documenting, and administering medications are all part of the process and should be done—with the latex allergy in mind. However, making sure that the patient has a safe environment is the first step.

18. A nurse is assigned to care for the following patients who all need vital signs taken right now. Which patient is most appropriate for the nurse to delegate vital sign measurement to the nursing assistive personnel (NAP)? a. Patient scheduled for a procedure in the nuclear medicine department b. Patient transferring from the intensive care unit (ICU) c. Patient returning from a cardiac catheterization d. Patient returning from hip replacement surgery

ANS: A The nurse does not assign vital sign measurement or other tasks to NAP when patients are experiencing a change in level of care. The patient awaiting the procedure in nuclear medicine is the only patient who has not experienced a change in level of care. According to the rights of delegation, tasks that are repetitive, require little supervision, are relatively noninvasive, have results that are predictable, and have minimal risk can be delegated to assistive personnel. The patient in this question with the most predictable condition is the patient awaiting the nuclear medicine procedure. Once the nurse determines that the other patients are stable, the nurse could delegate their future vital sign measurement to the NAP. However, it is important for the nurse to assess patients coming from the ICU, the cardiac cath lab, and surgery when they first arrive on the unit.

8. A nurse manager sent one of the staff nurses on the unit to a conference about new, evidence-based wound care techniques. The nurse manager asks the staff nurse to prepare a poster to present at the next unit meeting, which will be mandatory for all nursing staff on the unit. Which type of opportunity is the nurse manager providing for the staff? a. Staff education b. Interprofessional collaboration c. Providing a professional shared governance council d. Establishing a nursing practice committee

ANS: A The nurse manager is planning a staff education opportunity. Staff education is one way the nurse manager supports staff involvement in a shared decision-making model. Interprofessional collaboration between nurses and health care providers (e.g., MD, PT, TR, etc.) is critical to the delivery of quality, safe patient care and the creation of a positive work culture for practitioners. The question does not state that the nurse is establishing a practice committee or a professional shared governance council. Chaired by senior clinical staff nurses, these groups establish and maintain care standards for nursing practice on their work unit.

19. A nurse assesses that a patient has not voided in 6 hours. Which question should the nurse ask to assist in establishing a nursing diagnosis of Urinary retention? a."Do you feel like you need to go to the bathroom?" b."Are you able to walk to the bathroom by yourself?" c."When was the last time you took your medicine?" d."Do you have a safety rail in your bathroom at home?"

ANS: A The nurse must establish that the patient feels the urge and is unable to void. The question "Do you feel like you need to go to the bathroom?" is the most appropriate to ask. This question can be answered without knowledge of the diagnosis of Urinary retention. Discussing the ability to walk to the bathroom and asking about safety rails pertain to mobility and safety issues, not to retention of urine. Taking certain medications may lead to urinary retention, but that information would establish the etiology. The question is asking for the nurse to first establish the correct diagnosis.

9. A nurse is making a home visit and discovers that a patient's wound infection has gotten worse. The nurse cleans and redresses the wound. What should the nurse do next? a. Notify the health care provider of the findings before leaving the home. b. Ask the home health facility nurse manager to contact the health care provider. c. Document the findings and confirm with the patient the date of the next home visit. d. Tell the patient that the health care provider will be notified before the next home visit.

ANS: A The nurse should notify the health care provider before leaving the home. Regardless of the setting, an enriching professional environment is one in which staff members respect one another's ideas, share information, and keep one another informed. The manager should avoid taking care of problems for staff. The staff nurse needs to learn how to professionally communicate with other members of the health care team and demonstrate interprofessional collaboration.

33. The nurse is caring for a postoperative patient who has had a minimally invasive carpel tunnel repair. The patient has a temperature of 97° F and is shivering. Which reason will the nurse most likely consider as the primary cause when planning care? a. Anesthesia lowers metabolism. b. Surgical suites have air currents. c. The patient is dressed only in a gown. d. The large open body cavity contributed to heat loss.

ANS: A The operating suite and recovery room environments are extremely cool. The patient's anesthetically depressed level of body function results in lowering of metabolism and a fall in body temperature. Although the patient is dressed in a gown and there are air currents in the operating room, these are not the primary reasons for the low temperature. Also, the patient in this type of case does not have a large open body cavity to contribute to heat loss.

23. During preoperative assessment for a 7:30 AM (0730) surgery, the nurse finds the patient drank a cup of coffee this morning. The nurse reports this information to the anesthesia provider. Which action does the nurse anticipate next? a. A delay in or cancellation of surgery b. Questions regarding components of the coffee c. Additional questions about why the patient had coffee d. Instructions to determine what education was provided in the preoperative visit

ANS: A The recommendations before nonemergent procedures requiring general and regional anesthesia or sedation/analgesia include fasting from intake of clear liquids for 2 or more hours. A delay in or cancellation of surgery will be in order for this case. Questions regarding components of the coffee, asking why, and evaluating the preoperative education may all be items to be addressed, especially from a performance improvement perspective, but at this time in caring for this patient, a delay or cancellation is in order to prevent aspiration.

39. The nurse demonstrates postoperative exercises for a patient. In which order will the nurse instruct the patient to perform the exercises? 1. Turning 2. Breathing 3. Coughing 4. Leg exercises a. 4, 1, 2, 3 b. 1, 2, 3, 4 c. 2, 3, 4, 1 d. 3, 1, 4, 2

ANS: A The sequence of exercises is leg exercises, turning, breathing, and coughing.

21.The health care provider has ordered a hypotonic intravenous (IV) solution to be administered. Which IV bag will the nurse prepare? a. 0.45% sodium chloride (1/2 NS) b. 0.9% sodium chloride (NS) c. Lactated Ringer's (LR) d. Dextrose 5% in Lactated Ringer's (D5LR)

ANS: A 0.45% sodium chloride is a hypotonic solution. NS and LR are isotonic. D5LR is hypertonic. DIF:Apply (application)REF:956

31. A nurse is providing discharge teaching for a patient who is going home with a guaiac test. Which statement by the patient indicates the need for further education? a. "If I get a blue color that means the test is negative." b. "I should not get any urine on the stool I am testing." c. "If I eat red meat before my test, it could give me false results." d. "I should check with my doctor to stop taking aspirin before the test."

ANS: A A blue color indicates a positive guaiac, or presence of fecal occult blood the patient needs more teaching to correct this misconception. Proper patient education is important for viable results. Be sure specimen is free of toilet paper and not contaminated with urine. The patient needs to avoid certain foods, like red meat, to rule out a false positive. While the health care provider should be consulted before asking a patient to stop any medication, if there are no contraindications, the patient should be instructed to stop taking aspirin, ibuprofen, naproxen or other nonsteroidal antiinflammatory drugs for 7 days because these could cause a false-positive test result. DIF:Apply (application)REF:1157-1158

A nurse is developing a care plan for a patient with a pelvic fracture on bed rest. Which goal statement is realistic for the nurse to assign to this patient? a. Patient will increase activity level this shift. b. Patient will turn side to back to side with assistance every 2 hours. c. Patient will use the walker correctly to ambulate to the bathroom as needed. d. Patient will use a sliding board correctly to transfer to the bedside commode as needed.

ANS: A A goal is a broad statement of desired change; the patient will increase activity level is a broad statement. Turning is the expected outcome. When determining goals, the nurse needs to ensure that the goal is individualized and realistic for the patient. Since the patient is on bed rest, using a walker and bedside commode is contraindicated.

17.A nurse is caring for a patient with peripheral intravenous (IV) therapy. Which task will the nurse assign to the nursing assistive personnel? a. Recording intake and output b. Regulating intravenous flow rate c. Starting peripheral intravenous therapy d. Changing a peripheral intravenous dressing

ANS: A A nursing assistive personnel (NAP) can record intake and output. An RN cannot delegate regulating flow rate, starting an IV, or changing an IV dressing to an NAP. DIF:Understand (comprehension)REF:951

A nursing student has been written up several times for being late with providing patient care and for omitting aspects of patient care and not knowing basic procedures that were taught in the skills course one term earlier. The nursing student says, "I don't understand what the big deal is. As my instructor, you are there to protect me and make sure I don't make mistakes." What is the best response from the nursing instructor? a. "You are expected to perform at the level of a professional nurse." b. "You are expected to perform at the level of a nursing student." c. "You are practicing under the license of the nurse assigned to the patient." d. "You are expected to perform at the level of a skilled nursing assistant."

ANS: A Although nursing students are not employees of the health care agency where they are having their clinical experience, they are expected to perform as professional nurses would in providing safe patient care. Different levels of standards do not apply. Nursing students, just as nurses, provide safe, complete patient care, or they don't. No standard is used for nursing students other than that they must meet the standards of a professional nurse. The nursing instructor, not the nurse assigned to the patient, is responsible for the actions of the nursing student.

The following statements are on a patient's nursing care plan. Which statement will the nurse use as an outcome for a goal of care? a. The patient will verbalize a decreased pain level less than 3 on a 0 to 10 scale by the end of this shift. b. The patient will demonstrate increased tolerance to activity over the next month. c. The patient will understand needed dietary changes by discharge. d. The patient will demonstrate increased mobility in 2 days.

ANS: A An expected outcome is a specific and measurable change that is expected as a result of nursing care. Verbalizing decreased pain on a 0 to 10 scale is an outcome. The other three options in this question are goals. Demonstrating increased mobility in 2 days and understanding necessary dietary changes by discharge are short-term goals because they are expected to occur in less than a week. Demonstrating increased tolerance to activity over a month-long period is a long-term goal because it is expected to occur over a longer period of time.

6. The nurse questions a physician's order to administer a placebo to the patient. The nurse's action is based on which ethical principle? A. Autonomy B. Beneficence C. Justice D. Fidelity

ANS: A Autonomy refers to the freedom to make decisions free of external control. In this case, the nurse questions the physician's order for a placebo because it supports the patient's autonomy. Although beneficence, taking a positive action for others, has implications here, it is not the primary operating principle. Justice refers to fairness and is most often used in discussions about access to health care resources. Fidelity refers to the agreement to keep promises.

17. A nurse is caring for a dying patient. When is the best time for the nurse to discuss end-of-life care? a. During assessment b. During planning c. During implementation d. During evaluation

ANS: A Because most deaths are now "negotiated" among patients, family members, and the health care team, discuss end-of-life care preferences early in the assessment phase of the nursing process. Doing so during the planning, implementation, and evaluation phases is too late.

A nurse performs cardiopulmonary resuscitation (CPR) on a 92-year-old with brittle bones and breaks a rib during the procedure, which then punctures a lung. The patient recovers completely without any residual problems and sues the nurse for pain and suffering, and for malpractice. What key point will the prosecution attempt to prove? a. The CPR procedure was done incorrectly. b. The patient would have died if nothing was done. c. The patient was resuscitated according to policy. d. Patients with brittle bones might sustain fractures when chest compressions are done.

ANS: A Certain criteria are necessary to establish nursing malpractice. In this situation, although harm was caused, it was not because of failure of the nurse to perform a duty according to standards the way other nurses would have performed in the same situation. The nurse would have had to have done the procedure correctly, or the patient most likely would not have survived without any residual problems such as brain damage. The fact that the patient sustained injury as a result of age and physical status does not mean the nurse breached any duty to the patient. The nurse would need to make sure the defense attorney knew that the cardiopulmonary resuscitation (CPR) was done correctly. Without intervention, the patient most likely would not have survived. The prosecution would try to prove that a breach of duty had occurred, which had caused injury, not that cardiopulmonary resuscitation was done correctly. The defense team, not the prosecution, would explain the correlation between brittle bones and rib fractures during CPR.

10.The patient has an intravenous (IV) line and the nurse needs to remove the gown. In which order will the nurse perform the steps, starting with the first one? 1. Remove the sleeve of the gown from the arm without the IV. 2. Remove the sleeve of the gown from the arm with the IV. 3. Remove the IV solution container from its stand. 4. Pass the IV bag and tubing through the sleeve. a. 1, 2, 3, 4 b. 2, 3, 4, 1 c. 3, 4, 1, 2 d. 4, 1, 2, 3

ANS: A Change regular gowns by following these steps for maximum speed and arm mobility: (1) To remove a gown, remove the sleeve of the gown from the arm without the IV line, maintaining the patient's privacy. (2) Remove the sleeve of the gown from the arm with the IV line. (3) Remove the IV solution container from its stand, and pass it and the tubing through the sleeve. (If this involves removing the tubing from an EID, use the roller clamp to slow the infusion to prevent the accidental infusion of a large volume of solution or medication.) DIF:Understand (comprehension)REF:959-960

14. A nurse is developing a care plan. Which intervention is most appropriate for the nursing diagnostic statement Risk for loneliness related to impaired verbal communication? a. Provide the patient with a writing board each shift. b. Obtain an interpreter for the patient as soon as possible. c. Assist the patient in performing swallowing exercises each shift. d. Ask the family to provide a sitter to remain with the patient at all times.

ANS: A Choose interventions to alter the etiological (related to) factor or causes of the diagnosis. If the etiology is impaired verbal communication, then the nurse should choose an intervention that will address the problem. Providing the patient with a writing board will allow the patient to communicate by writing because the patient is unable to communicate verbally at this time. Obtaining an interpreter might be an appropriate intervention if the patient spoke a foreign language. Assisting with swallowing exercises will help the patient with swallowing, which is a different etiology than impaired verbal communication. Asking the family to provide a sitter at all times is many times unrealistic and does not relate to the impaired verbal communication; the goal would relate to the loneliness.

18. The nurse is interviewing a patient with a hearing deficit. Which area should the nurse use to conduct this interview? a. The patient's room with the door closed b. The waiting area with the television turned off c. The patient's room before administration of pain medication d. The waiting room while the occupational therapist is working on leg exercises

ANS: A Distractions should be eliminated as much as possible when interviewing a patient with a hearing deficit. The best place to conduct this interview is in the patient's room with the door closed. The waiting area does not provide privacy. Pain can sometimes inhibit someone's ability to concentrate, so before pain medication is administered is not advisable. It is best for the patient to be as comfortable as possible when conducting an interview. Assessing a patient while another member of the health care team is working would be distracting and is not the best time for an interview to take place.

15. The mother of a child who died recently keeps the child's room intact. Family members are encouraging her to redecorate and move forward in life. Which type of grief will the home health nurse recognize the mother is experiencing? a. Normal b. End-of-life c. Abnormal d. Complicated

ANS: A Family members will grieve differently. One sign of normal grief is keeping the deceased individual's room intact as a way to keep that person alive in the minds of survivors. This is happening after the family member is deceased, so it is not end-of-life grief. It is not abnormal or complicated grief; the child died recently.

9. Which philosophy of healthcare ethics would be particularly useful when making ethical decisions about vulnerable populations? A. Feminist ethics B. Deontology C. Bioethics D. Utilitarianism

ANS: A Feminist ethics particularly focuses on the nature of relationships, especially those where there is a power imbalance or a point of view that is not routinely accepted. Examples of populations that are considered vulnerable include children, pregnant women, incarcerated persons, and minority groups. Deontology refers to making decisions or "right-making characteristics," bioethics focuses on consensus building, and utilitarianism speaks to the greatest good for the greatest number.

15. The nurse is caring for a severely ill patient with AIDS who now requires ventilator support. Which intervention is considered futile? A. Administering the influenza vaccine B. Providing oral care every 5 hours C. Applying fentanyl patches prn for pain D. Supporting the patient's lower extremities with pillows

ANS: A Futile refers to something that is hopeless or serves no useful purpose and in nursing refers to interventions that are unlikely to produce benefit for the patient. Care delivered to a patient at the end of life is focused on pain management and comfort measures. A vaccine is administered to prevent or lessen the likelihood of contracting an infectious disease at some time in the future.

4.The nurse administers an intravenous (IV) hypertonic solution to a patient. In which direction will the fluid shift? a. From intracellular to extracellular b. From extracellular to intracellular c. From intravascular to intracellular d. From intravascular to interstitial

ANS: A Hypertonic solutions will move fluid from the intracellular to the extracellular (intravascular). A hypertonic solution has a concentration greater than normal body fluids, so water will shift out of cells because of the osmotic pull of the extra particles. Movement of water from the extracellular (intravascular) into cells (intracellular) occurs when hypotonic fluids are administered. Distribution of fluid between intravascular and interstitial spaces occurs by filtration, the net sum of hydrostatic and osmotic pressures. DIF:Understand (comprehension)REF:936 | 956

A hospital's wound nurse consultant made a recommendation for nurses on the unit about how to care for the patient's dressing changes. Which action should the nurses take next? a. Include dressing change instructions and frequency in the care plan. b. Assume that the wound nurse will perform all dressing changes. c. Request that the health care provider look at the wound. d. Encourage the patient to perform the dressing changes.

ANS: A Incorporate the consultant's recommendations into the care plan. The wound nurse clearly recommends that nurses on the unit, not the patient, should continue dressing changes. The nurses should not make a wrong assumption that the wound nurse is doing all the dressing changes. The recommendation states for the nurses to do the dressing changes. If the nurses feel strongly about obtaining another opinion, then the health care provider should be contacted. No evidence in the question suggests that the patient needs a second opinion.

27.The nurse is calculating intake and output on a patient. The patient drinks 150 mL of orange juice at breakfast, voids 125 mL after breakfast, vomits 250 mL of greenish fluid, sucks on 60 mL of ice chips, and for lunch consumes 75 mL of chicken broth. Which totals for intake and output will the nurse document in the patient's medical record? a. Intake 255; output 375 b. Intake 285; output 375 c. Intake 505; output 125 d. Intake 535; output 125

ANS: A Intake = 150 mL of orange juice, 60 mL of ice chips (but only counts as 30 since ice chips are half of the amount), and 75 mL of chicken broth; 150 + 30 + 75 = 255. Output = 125 mL of urine (void) and 250 mL of vomitus; 125 + 250 = 375. DIF:Apply (application)REF:949 | 951 | 955

1.A patient has dehydration. While planning care, the nurse considers that the majority of the patient's total water volume exists in with compartment? a. Intracellular b. Extracellular c. Intravascular d. Transcellular

ANS: A Intracellular (inside the cells) fluid accounts for approximately two thirds of total body water. Extracellular (outside the cells) is approximately one third of the total body water. Intravascular fluid (liquid portion of the blood) and transcellular fluid are two major divisions of the extracellular compartment.

1. Four patients in labor all request epidural analgesia to manage their pain at the same time. Which ethical principle is compromised when only one nurse anesthetist is on call? A. Justice B. Nonmaleficence C. Beneficence D. Fidelity

ANS: A Justice refers to fairness and is used frequently in discussion regarding access to health care resources. Here the just distribution of resources, in this case pain management, cannot be justly apportioned. Nonmaleficence means "do no harm," beneficence means "to do good," and fidelity means "to be true to, or honest." Each of these principles is partially expressed in the question; however, justice is most comprised because not all laboring patients have equal access to pain management owing to lack of personnel resources

32. A nurse is preparing to lavage a patient in the emergency department for an overdose. Which tube should the nurse obtain? a. Ewald b. Dobhoff c. Miller-Abbott d. Sengstaken-Blakemore

ANS: A Lavage is irrigation of the stomach in cases of active bleeding, poisoning, or gastric dilation. The types of tubes include Levin, Ewald, and Salem sump. Sengstaken-Blakemore is used for compression by internal application of pressure by means of inflated balloon to prevent internal esophageal or GI hemorrhage. Dobhoff is used for enteral feeding. Miller-Abbott is used for gastric decompression. DIF:Apply (application)REF:1167

18. A nurse is providing postmortem care. Which action will the nurse take? a. Leave dentures in the mouth. b. Lower the head of the bed. c. Cover the body with a sterile sheet. d. Remove all tubes for an autopsy.

ANS: A Leave dentures in the mouth to maintain facial shape. Raise the head of the bed as soon as possible after death to prevent discoloration of the face. Cover the body with a clean sheet. Autopsy often does not allow removal of tubes, equipment, and indwelling lines.

5. Ethical dilemmas are common occurrences when caring for patients. The nurse understands that dilemmas are a result of A. Presence of conflicting values. B. Hierarchical systems. C. Judgmental perceptions of patients. D. Poor communication with the patient.

ANS: A Poor communication and the hierarchical systems that exist in health care, such as reporting structures within the hospital or the historically unequal relationship between physicians and nurses, may complicate dilemmas. The primary, underlying reason that ethical dilemmas occur is that there are no clear cut, universally accepted solutions to a problem when participating individuals do not share the same values. Without clarification of values, the nurse may not be able to distinguish fact from opinion or value, and this can lead to judgmental attitudes

6.The nurse is reviewing laboratory results. Which cation will the nurse observe is the mostabundant in the blood? a. Sodium b. Chloride c. Potassium d. Magnesium

ANS: A Sodium is the most abundant cation in the blood. Potassium is the predominant intracellular cation. Chloride is an anion (negatively charged) rather than a cation (positively charged). Magnesium is found predominantly inside cells and in bone. DIF:Apply (application)REF:935-936

6. A patient is using laxatives three times daily to lose weight. After stopping laxative use, the patient has difficulty with constipation and wonders if laxatives should be taken again. Which information will the nurse share with the patient? a. Long-term laxative use causes the bowel to become less responsive to stimuli, and constipation may occur. b. Laxatives can cause trauma to the intestinal lining and scarring may result, leading to decreased peristalsis. c. Long-term use of emollient laxatives is effective for treatment of chronic constipation and may be useful in certain situations. d. Laxatives cause the body to become malnourished, so when the patient begins eating again, the body absorbs all of the food, and no waste products are produced.

ANS: A Teach patients about the potential harmful effects of overuse of laxatives, such as impaired bowel motility and decreased response to sensory stimulus. Make sure the patient understands that laxatives are not to be used long term for maintenance of bowel function. Increasing fluid and fiber intake can help with this problem. Laxatives do not cause scarring. Even if malnourished, the body will produce waste if any substance is consumed. DIF:Understand (comprehension)REF:1163

24. A patient is diagnosed with a bowel obstruction. Which type of tube is the best for the nurse to obtain for gastric decompression? a. Salem sump b. Small bore c. Levin d. 8 Fr

ANS: A The Salem sump tube is preferable for stomach decompression. The Salem sump tube has two lumina: one for removal of gastric contents and one to provide an air vent. When the main lumen of the sump tube is connected to suction, the air vent permits free, continuous drainage of secretions. While the Levin tube can be used for decompression, it is only a single-lumen tube with holes near the tip. Large-bore tubes, 12 Fr and above, are usually used for gastric decompression or removal of gastric secretions. Fine- or small-bore tubes are frequently used for medication administration and enteral feedings. DIF:Apply (application)REF:1166

1. The nurse is using critical thinking skills during the first phase of the nursing process. Which action indicates the nurse is in the first phase? a. Completes a comprehensive database b. Identifies pertinent nursing diagnoses c. Intervenes based on priorities of patient care d. Determines whether outcomes have been achieved

ANS: A The assessment phase of the nursing process involves data collection to complete a thorough patient database and is the first phase. Identifying nursing diagnoses occurs during the diagnosis phase or second phase. The nurse carries out interventions during the implementation phase (fourth phase), and determining whether outcomes have been achieved takes place during the evaluation phase (fifth phase) of the nursing process.

A nurse is preparing to make a consult. In which order, beginning with the first step, will the nurse take? 1. Identify the problem. 2. Discuss the findings and recommendation. 3. Provide the consultant with relevant information about the problem. 4. Contact the right professional, with the appropriate knowledge and expertise. 5. Avoid bias by not providing a lot of information based on opinion to the consultant. a. 1, 4, 3, 5, 2 b. 4, 1, 3, 2, 5 c. 1, 4, 5, 3, 2 d. 4, 3, 1, 5, 2

ANS: A The first step in making a consultation is to assess the situation and identify the general problem area. Second, direct the consultation to the right professional such as another nurse or social worker. Third, provide a consultant with relevant information about the problem area and seek a solution. Fourth, do not prejudice or influence consultants. Fifth, be available to discuss a consultant's findings and recommendations.

A nurse is caring for a patient with a nursing diagnosis of Constipation related to slowed gastrointestinal motility secondary to pain medications. Which outcome is most appropriate for the nurse to include in the plan of care? a. Patient will have one soft, formed bowel movement by end of shift. b. Patient will walk unassisted to bathroom by the end of shift. c. Patient will be offered laxatives or stool softeners this shift. d. Patient will not take any pain medications this shift.

ANS: A The identified problem, or nursing diagnosis, is constipation. Therefore, the outcome should be that the constipation is relieved. To measure constipation relief, the nurse will be observing for the patient to have a bowel movement. During planning, you select goals and expected outcomes for each nursing diagnosis or problem to provide clear direction for the type of interventions needed to care for your patient and to then evaluate the effectiveness of these interventions. Not taking pain medications may or may not relieve the constipation. Although not taking pain medicines might be an intervention, the nurse doesn't want the patient to be in pain to relieve constipation. Other measures, such as administering laxatives or stool softeners, might be appropriate interventions but they are not outcomes. The patient walking unassisted to the bathroom addresses mobility, not constipation. The patient may need to walk to the bathroom to have a bowel movement, but the appropriate outcome for constipation is that the constipation is relieved as evidenced by a bowel movement—something that the nurse can observe.

9.Four patients arrive at the emergency department at the same time. Which patient will the nurse see first? a. An infant with temperature of 102.2° F and diarrhea for 3 days b. A teenager with a sprained ankle and excessive edema c. A middle-aged adult with abdominal pain who is moaning and holding her stomach d. An older adult with nausea and vomiting for 3 days with blood pressure 112/60

ANS: A The infant should be seen first. An infant's proportion of total body water (70% to 80% total body weight) is greater than that of children or adults. Infants and young children have greater water needs and immature kidneys. They are at greater risk for extracellular volume deficit and hypernatremia because body water loss is proportionately greater per kilogram of weight. A teenager with excessive edema from a sprained ankle can wait. A middle-aged adult moaning in pain can wait as can an older adult with a blood pressure of 112/60. DIF:Analyze (analysis)REF:947

16.A patient is experiencing respiratory acidosis. Which organ system is responsible for compensation in this patient? a. Renal b. Endocrine c. Respiratory d. Gastrointestinal

ANS: A The kidneys (renal) are responsible for respiratory acidosis compensation. A problem with the respiratory system causes respiratory acidosis, so another organ system (renal) needs to compensate. Problems with the gastrointestinal and endocrine systems can cause acid-base imbalances, but these systems cannot compensate for an existing imbalance. DIF:Understand (comprehension)REF:943-944

18.The nurse is caring for a diabetic patient in renal failure who is in metabolic acidosis. Which laboratory findings are consistent with metabolic acidosis? a. pH 7.3, PaCO2 36 mm Hg, HCO3- 19 mEq/L b. pH 7.5, PaCO2 35 mm Hg, HCO3- 35 mEq/L c. pH 7.32, PaCO2 47 mm Hg, HCO3- 23 mEq/L d. pH 7.35, PaCO2 40 mm Hg, HCO3- 25 mEq/L

ANS: A The laboratory values that reflect metabolic acidosis are pH 7.3, PaCO2 36 mm Hg, HCO3- 19 mEq/L. A laboratory finding of pH 7.5, PaCO2 35 mm Hg, HCO3- 35 mEq/L is metabolic alkalosis. pH 7.32, PaCO2 47 mm Hg, HCO3- 23 mEq/L is respiratory acidosis. pH 7.35, PaCO2 40 mm Hg, HCO3- 25 mEq/L values are within normal range. DIF:Analyze (analysis)REF:944-945

3. A nurse is teaching a patient about the large intestine in elimination. In which order will the nurse list the structures, starting with the first portion? a. Cecum, ascending, transverse, descending, sigmoid, and rectum b. Ascending, transverse, descending, sigmoid, rectum, and cecum c. Cecum, sigmoid, ascending, transverse, descending, and rectum d. Ascending, transverse, descending, rectum, sigmoid, and cecum

ANS: A The large intestine is divided into the cecum, ascending colon, transverse colon, descending colon, sigmoid colon, and rectum. The large intestine is the primary organ of bowel elimination. DIF:Understand (comprehension)REF:1150

34.The nurse is caring for a group of patients. Which patient will the nurse see first? a. A patient with D5W hanging with the blood b. A patient with type A blood receiving type O blood c. A patient with intravenous potassium chloride that is diluted d. A patient with a right mastectomy and an intravenous site in the left arm

ANS: A The nurse will see the patient with D5W and blood to prevent a medication error. When preparing to administer blood, prime the tubing with 0.9% sodium chloride (normal saline) to prevent hemolysis or breakdown of RBCs. All the rest are normal. A patient with type A blood can receive type O. Type O is considered the universal donor. A patient with a mastectomy should have the IV in the other arm. Potassium chloride should be diluted, and it is never given IV push. DIF:Analyze (analysis)REF:962

14. The nurse is devising a plan of care for a patient with the nursing diagnosis of Constipation related to opioid use. Which outcome will the nurse evaluate as successful for the patient to establish normal defecation? a. The patient reports eliminating a soft, formed stool. b. The patient has quit taking opioid pain medication. c. The patient's lower left quadrant is tender to the touch. d. The nurse hears bowel sounds in all four quadrants.

ANS: A The nurse's goal is for the patient to take opioid medication and to have normal bowel elimination. Normal stools are soft and formed. Ceasing pain medication is not a desired outcome for the patient. Tenderness in the left lower quadrant indicates constipation and does not indicate success. Bowel sounds indicate that the bowels are moving however, they are not an indication of defecation. DIF:Apply (application)REF:1150 | 1169

15. Which scenario best illustrates the nurse using data validation when making a nursing clinical decision for a patient? a. The nurse determines to remove a wound dressing when the patient reveals the time of the last dressing change and notices old and new drainage. b. The nurse administers pain medicine due at 1700 at 1600 because the patient reports increased pain and the family wants something done. c. The nurse immediately asks the health care provider for an order of potassium when a patient reports leg cramps. d. The nurse elevates a leg cast when the patient reports decreased mobility.

ANS: A The only scenario that validates a patient's report with a nurse's observation is changing the wound dressing. The nurse validates what the patient says by observing the dressing. The rest of the examples have the nurse acting only from a patient and/or family reports, not the nurse's assessment.

2.The nurse is teaching about the process of passively moving water from an area of lower particle concentration to an area of higher particle concentration. Which process is the nurse describing? a. Osmosis b. Filtration c. Diffusion d. Active transport

ANS: A The process of moving water from an area of low particle concentration to an area of higher particle concentration is known as osmosis. Filtration is mediated by fluid pressure from an area of higher pressure to an area of lower pressure. Diffusion is passive movement of electrolytes or other particles down the concentration gradient (from areas of higher concentration to areas of lower concentration). Active transport requires energy in the form of adenosine triphosphate (ATP) to move electrolytes across cell membranes against the concentration gradient (from areas of lower concentration to areas of higher concentration).

38.A nurse is assessing a patient who is receiving a blood transfusion and finds that the patient is anxiously fidgeting in bed. The patient is afebrile and dyspneic. The nurse auscultates crackles in both lung bases and sees jugular vein distention. On which transfusion complication will the nurse focus interventions? a. Fluid volume overload b. Hemolytic reaction c. Anaphylactic shock d. Septicemia

ANS: A The signs and symptoms are concurrent with fluid volume overload. Anaphylactic shock would have presented with urticaria, dyspnea, and hypotension. Septicemia would include a fever. A hemolytic reaction would consist of flank pain, chills, and fever. DIF:Apply (application)REF:963-964

21. A patient with a fecal impaction has an order to remove stool digitally. In which order will the nurse perform the steps, starting with the first one? 1. Obtain baseline vital signs. 2. Apply clean gloves and lubricate. 3. Insert index finger into the rectum. 4. Identify patient using two identifiers. 5. Place patient on left side in Sims' position. 6. Massage around the feces and work down to remove. a. 4, 1, 5, 2, 3, 6 b. 1, 4, 2, 5, 3, 6 c. 4, 1, 2, 5, 3, 6 d. 1, 4, 5, 2, 3, 6

ANS: A The steps for removing a fecal impaction are as follows: identify patient using two identifiers obtain baseline vital signs TERM place on left side in Sims' position apply clean gloves and lubricate TERM insert index finger into the rectum and gently loosen the fecal mass by massaging around it and work the feces downward toward the end of the rectum. DIF:Understand (comprehension)REF:1166

31.A nurse is caring for a diabetic patient with a bowel obstruction and has orders to ensure that the volume of intake matches the output. In the past 4 hours, the patient received dextrose 5% with 0.9% sodium chloride through a 22-gauge catheter infusing at 150 mL/hr and has eaten 200 mL of ice chips. The patient also has an NG suction tube set to low continuous suction that had 300-mL output. The patient has voided 400 mL of urine. After reporting these values to the health care provider, which order does the nurse anticipate? a. Add a potassium supplement to replace loss from output. b. Decrease the rate of intravenous fluids to 100 mL/hr. c. Administer a diuretic to prevent fluid volume excess. d. Discontinue the nasogastric suctioning.

ANS: A The total fluid intake and output equals 700 mL, which meets the provider goals. Patients with nasogastric suctioning are at risk for potassium deficit, so the nurse would anticipate a potassium supplement to correct this condition. Remember to record half the volume of ice chips when calculating intake. The other measures would be unnecessary because the net fluid volume is equal. DIF:Analyze (analysis)REF:949 | 951 | 955

34. A nurse is performing an assessment on a patient who has not had a bowel movement in 3 days. The nurse will expect which other assessment finding? a. Hypoactive bowel sounds b. Increased fluid intake c. Soft tender abdomen d. Jaundice in sclera

ANS: A Three or more days with no bowel movement indicates hypomotility of the GI tract. Assessment findings would include hypoactive bowel sounds, a firm distended abdomen, and pain or discomfort upon palpation. Increased fluid intake would help the problem a decreased intake can lead to constipation. Jaundice does not occur with constipation but can occur with liver disease. DIF:Apply (application)REF:1152 | 1157 | 1161

8. While interviewing an older female patient of Asian descent, the nurse notices that the patient looks at the ground when answering questions. What should the nurse do? a. Consider cultural differences during this assessment. b. Ask the patient to make eye contact to determine her affect. c. Continue with the interview and document that the patient is depressed. d. Notify the health care provider to recommend a psychological evaluation.

ANS: A To conduct an accurate and complete assessment, consider a patient's cultural background. This nurse needs to practice culturally competent care and appreciate the cultural differences. Assuming that the patient is depressed or in need of a psychological evaluation or to force eye contact is inappropriate.

The nurse calculates the medication dose for an infant on the pediatric unit and determines that the dose is twice what it should be. The pediatrician is contacted and says to administer the medication as ordered. What is the next action that the nurse should take? (Select all that apply.) a. Notify the nursing supervisor. b. Check the chain of command policy for such situations. c. Give the medication as ordered. d. Give the amount calculated to be correct. e. Contact the pharmacy for clarification.

ANS: A, B Nurses follow health care providers' orders unless they believe the orders are in error or may harm patients. Therefore, the nurse needs to assess all orders. If an order seems to be erroneous or harmful, further clarification from the health care provider is necessary. If the health care provider confirms an order and the nurse still believe that it is inappropriate, the nurse should inform the supervising nurse or follow the established chain of command. The supervising nurse should be able to help resolve the questionable order, but only the health care provider who wrote the order or a health care provider covering for the one who wrote the order can change the order. Harm to the infant could occur if the medication dosage was too high. The nurse cannot change an order. Giving the amount calculated to be correct would not be what another nurse would do in the same situation. Although the pharmacy is an excellent resource, only the health care provider can change the order.

1. The nurse is participating in a "time-out." In which activities will the nurse be involved? (Select all that apply.) a. Verify the correct site. b. Verify the correct patient. c. Verify the correct procedure. d. Perform "time-out" after surgery. e. Perform the actual marking of the operative site.

ANS: A, B, C A time-out is performed just before starting the procedure for final verification of the correct patient, procedure, site, and any implants. The marking and time-out most commonly occur in the holding area, just before the patient enters the OR. The individual performing surgery and who is accountable for it must personally mark the site, and the patient must be involved if possible.

1. A recently widowed older-adult patient is dehydrated and is admitted to the hospital for intravenous fluid replacement. During the evening shift, the patient becomes acutely confused. Which possible reversible causes will the nurse consider when assessing this patient? (Select all that apply.) a.Electrolyte imbalance b.Sensory deprivation c.Hypoglycemia d.Drug effects e.Dementia

ANS: A, B, C, D Delirium, or acute confusional state, is a potentially reversible cognitive impairment that is often due to a physiological event. Physiological causes include electrolyte imbalances, untreated pain, infection, cerebral anoxia, hypoglycemia, medication effects, tumors, subdural hematomas, and cerebrovascular infarction or hemorrhage. Sometimes it is also caused by environmental factors such as sensory deprivation or overstimulation, unfamiliar surroundings, or sleep deprivation or psychosocial factors such as emotional distress. Dementia is a gradual, progressive, and irreversible cerebral dysfunction.

2. A nurse is teaching a health class about colorectal cancer. Which information should the nurse include in the teaching session? (Select all that apply.) a. A risk factor is smoking. b. A risk factor is high intake of animal fats or red meat. c. A warning sign is rectal bleeding. d. A warning sign is a sense of incomplete evacuation. e. Screening with a colonoscopy is every 5 years, starting at age 50. f. Screening with flexible sigmoidoscopy is every 10 years, starting at age 50.

ANS: A, B, C, D Risk factors for colorectal cancer are a diet high in animal fats or red meat and low intake of fruits and vegetables smoking and heavy alcohol consumption are also risk factors. Warning signs are change in bowel habits, rectal bleeding, a sensation of incomplete evacuation, and unexplained abdominal or back pain. A flexible sigmoidoscopy is every 5 years, starting at age 50, while a colonoscopy is every 10 years, starting at age 50. DIF:Understand (comprehension)REF:1162

1. A nurse uses the five rights of delegation when providing care. Which "rights" did the nurse use? (Select all that apply.) a. Right task b. Right person c. Right direction d. Right supervision e. Right circumstances f. Right cost-effectiveness

ANS: A, B, C, D, E The five rights of delegation are right task, circumstances, person, direction, and supervision. Cost-effectiveness is not a right.

2. A nurse is providing nursing care to a group of patients. Which actions are direct care interventions? (Select all that apply.) a.Ambulating a patient b.Inserting a feeding tube c.Performing resuscitation d.Documenting wound care e.Teaching about medications

ANS: A, B, C, E All of the interventions listed (ambulating, inserting a feeding tube, performing resuscitation, and teaching) are direct care interventions involving patient and nurse interaction, except documenting wound care. Documenting wound care is an example of an indirect intervention.

4. Which interventions are appropriate for a patient with diabetes and poor wound healing? (Select all that apply.) a.Perform dressing changes twice a day as ordered. b.Teach the patient about signs and symptoms of infection. c.Instruct the family about how to perform dressing changes. d.Gently refocus patient from discussing body image changes. e.Administer medications to control the patient's blood sugar as ordered.

ANS: A, B, C, E Nursing priorities include interventions directed at enhancing wound healing. Teaching the patient about signs and symptoms of infection will help the patient identify signs of appropriate wound healing and know when the need for calling the health care provider arises. Performing dressing changes, controlling blood sugars through administration of medications, and instructing the family in dressing changes all contribute to wound healing. As long as a patient is stable and alert, it is appropriate to allow family to assist with care. The patient should be allowed to discuss body image changes.

3. A nurse is preparing to carry out interventions. Which resources will the nurse make sure are available? (Select all that apply.) a.Equipment b.Safe environment c.Confidence d.Assistive personnel e.Creativity

ANS: A, B, D A nurse will organize time and resources in preparation for implementing nursing care. Most nursing procedures require some equipment or supplies. Before performing an intervention, decide which supplies you need and determine their availability. Patient care staff (assistive personnel) work together as patients' needs demand it. A patient's care environment needs to be safe and conducive to implementing therapies. Confidence and creativity are needed to provide safe and effective patient care; however, these are critical thinking attitudes, not resources.

A patient has just been told that he has approximately six months to live and asks about advance directives. Which statements by the nurse give the patient correct information? (Select all that apply.) a. "You have the right to refuse treatment at any time." b. "If you want certain procedures or actions taken or not taken, and you might not be able to tell anyone at the time, you need to complete documents ahead of time that give your health care provider this information." c. "You will be resuscitated at any time to allow you the longest length of survival." d. "You might want to think about choosing someone who will make medical decisions for you in the event that you are unable to make your desires known." e. "We will get someone who knows the state's guidelines to assist you in setting up your living will." f. "If you travel to another state, your living will should cover your wishes."

ANS: A, B, D, E The ethical doctrine of autonomy ensures the patient the right to refuse medical treatment. Living wills are written documents that direct treatment in accordance with a patient's wishes in the event of a terminal illness or condition. With this legal document, the patient is able to declare which medical procedures he or she wants or does not want when terminally ill or in a persistent vegetative state. Each state providing for living wills has its own requirements for executing the health care proxy or durable power of attorney for health care (DPAHC). This is a legal document that designates a person or persons of one's choosing to make health care decisions when the patient is no longer able to make decisions on his or her own behalf. This agent makes health care treatment decisions based on the patient's wishes. Cardiopulmonary resuscitation (CPR) is an emergency treatment provided without patient consent. Health care providers perform CPR on an appropriate patient unless a do not resuscitate (DNR) order has been placed in the patient's chart. The statutes assume that all patients will be resuscitated unless a written DNR order is found in the chart. Legally competent adult patients can consent to a DNR order verbally or in writing after receiving appropriate information from the health care provider. Differences among the states have been noted regarding advance directives, so the patient should check state laws to see if a state will honor an advance directive that was originated in another state.

1. A nurse is preparing a bowel training program for a patient. Which actions will the nurse take? (Select all that apply.) a. Record times when the patient is incontinent. b. Help the patient to the toilet at the designated time. c. Lean backward on the hips while sitting on the toilet. d. Maintain normal exercise within the patient's physical ability. e. Apply pressure with hands over the abdomen, and strain while pushing. f. Choose a time based on the patient's pattern to initiate defecation-control measures.

ANS: A, B, D, F A successful program includes the following: Assessing the normal elimination pattern and recording times when the patient is incontinent. Choosing a time based on the patient's pattern to initiate defecation-control measures. Maintaining normal exercise within the patient's physical ability. Helping the patient to the toilet at the designated time. Offering a hot drink (hot tea) or fruit juice (prune juice) (or whatever fluids normally stimulate peristalsis for the patient) before the defecation time. Instructing the patient to lean forward at the hips while sitting on the toilet, apply manual pressure with the hands over the abdomen, and bear down but do not strain to stimulate colon emptying. DIF:Understand (comprehension)REF:1168

6. The operating room nurse is providing a hand-off report to the postanesthesia care unit (PACU) nurse. Which components will the operating room nurse include? (Select all that apply.) a. IV fluids b. Vital signs c. Insurance data d. Family location e. Anesthesia provided f. Estimated blood loss

ANS: A, B, E, F The surgical teams report will include topics such as the type of anesthesia provided, vital sign trends, intraoperative medications, IV fluids, estimated blood and urine loss, and pertinent information about the surgical wound (e.g., dressings, tubes, drains). When the patient enters the PACU, the nurse and members of the surgical team discuss his or her status. A standardized approach or tool for hand-off communications assists in providing accurate information about a patient's care, treatment and services, current condition, and any recent or anticipated changes. The hand-off is interactive, multidisciplinary, and done at the patient's bedside, allowing for a communication exchange that gives caregivers the chance to dialogue and ask questions. Insurance data and family location are unnecessary.

The nurse hears a physician say to the charge nurse that he doesn't want that same nurse caring for his patients because she is stupid and won't follow his orders. The physician also writes on his patient's medical records that the same nurse, by name, is not to care for any of his patients because of her incompetence. What component(s) of defamation has the physician committed? (Select all that apply.) a. Slander b. Invasion of privacy c. Libel d. Assault e. Battery

ANS: A, C Slander occurred when the physician spoke falsely about the nurse, and libel occurred when the physician wrote false information in the chart. Both of these situations could cause problems for the nurse's reputation. Invasion of privacy is the release of a patient's medical information to an unauthorized person such as a member of the press, the patient's employer, or the patient's family. Assault is any action that places a person in apprehension of a harmful or offensive contact without consent. No actual contact is necessary. Battery is any intentional touching without consent.

4. The nurse is preparing for a patient who will be going to surgery. The nurse screens for risk factors that can increase a person's risks in surgery. What risk factors are included in the nurse's screening? (Select all that apply.) a. Age b. Race c. Obesity d. Nutrition e. Pregnancy f. Ambulatory surgery

ANS: A, C, D, E Very young and old patients are at risk during surgery because of immature or declining physiological status. Normal tissue repair and resistance to infection depend on adequate nutrients. Obesity increases surgical risk by reducing respiratory and cardiac function. During pregnancy, the concern is for the mother and the developing fetus. Because all major systems of the mother are affected during pregnancy, risks for operative complications are increased. Race and ambulatory surgery are not risks associated with a surgical procedure.

.A nurse is selecting a site to insert an intravenous (IV) catheter on an adult. Which actions will the nurse take? (Select all that apply.) a. Check for contraindications to the extremity. b. Start proximally and move distally on the arm. c. Choose a vein with minimal curvature. d. Choose the patient's dominant arm. e. Select a vein that is rigid. f. Avoid areas of flexion.

ANS: A, C, F The vein should be relatively straight to avoid catheter occlusion. Contraindications to starting an IV catheter are conditions such as mastectomy, AV fistula, and central line in the extremity and should be checked before initiation of IV. Avoid areas of flexion if possible. The nurse should start distally and move proximally, choosing the nondominant arm if possible. The nurse should feel for the best location; a good vein should feel spongy, a rigid vein should be avoided because it might have had previous trauma or damage. DIF:Apply (application)REF:957 | 972

A nurse is planning care for a patient with a nursing diagnosis of Impaired skin integrity. The patient needs many nursing interventions, including a dressing change, several intravenous antibiotics, and a walk. Which factors does the nurse consider when prioritizing interventions? (Select all that apply.) a. Rank all the patient's nursing diagnoses in order of priority. b. Do not change priorities once they've been established. c. Set priorities based solely on physiological factors. d. Consider time as an influencing factor. e. Utilize critical thinking.

ANS: A, D, E By ranking a patient's nursing diagnoses in order of importance and always monitoring changing signs and symptoms (defining characteristics) of patient problems, you attend to each patient's most important needs and better organize ongoing care activities. Prioritizing the problems, or nursing diagnoses, will help the nurse decide which problem to address first. Symptom pattern recognition from your assessment database and certain knowledge triggers help you understand which diagnoses require intervention and the associated time frame to intervene effectively. Planning requires critical thinking applied through deliberate decision making and problem solving. The nurse avoids setting priorities based solely on physiological factors; other factors should be considered as well. The order of priorities changes as a patient's condition and needs change, sometimes within a matter of minutes.

3. The nurse is caring for a postoperative patient with an incision. Which actions will the nurse take to decrease wound infections? (Select all that apply.) a. Maintain normoglycemia. b. Use a straight razor to remove hair. c. Provide bath and linen change daily. d. Perform first dressing change 2 days postoperatively. e. Perform hand hygiene before and after contact with the patient. f. Administer antibiotics within 60 minutes before surgical incision.

ANS: A, E Performing hand hygiene before and after contact with the patient helps to decrease the number of microorganisms and break the chain of infection. Maintaining blood glucose levels at less than 150 mg/dL has resulted in decreased wound infection. Removing unwanted hair by clipping instead of shaving decreases the numbers of nicks and cuts caused by a razor and the potential for the introduction of microbes. The patient is postoperative; administration of an antibiotic 60 minutes before the surgical incision supports the defense against infection preoperatively. Providing a bath and linen change daily is positive but is not necessarily important for infection control. Many surgeons prefer to change surgical dressings the first time so they can inspect the incisional area, but this is done before 2 days postoperatively.

A nurse gives an incorrect medication to a patient without doing all of the mandatory checks, but the patient has no ill effects from the medication. What actions should the nurse take after reassessing the patient? (Select all that apply.) a. Notify the health care provider of the situation. b. Document in the patient's medical record that an occurrence report was filed. c. Document in the patient's medical record why the omission occurred. d. Discuss what happened with all of the other nurses and staff on the unit. e. Continue to monitor the patient for any untoward effects from the medication. f. Send an occurrence report to risk management after completing it.

ANS: A, E, F Examples of an occurrence include an error in technique or procedure such as failing to properly identify a patient. Institutions generally have specific guidelines to direct health care providers how to complete the occurrence report. The report is confidential and separate from the medical record. The nurse is responsible for providing information in the medical record about the occurrence. It is also best for the nurse to discuss the occurrence with nursing management only. The risk management department of the institution also requires complete documentation. The fact that an occurrence report was completed is not documented in the patient's medical record. No discussion of why the omission in procedure occurred should be documented in the patient's medical record. Errors should be discussed only with those who need to know such as the health care provider, appropriate administrative personnel, and risk management.

22. A previously toilet trained toddler has started wetting again. A nurse is gathering a health history from the grandparent. Which health history finding will the nurse most likely consider as the cause of the wetting? a. Dietary changes b. Recent parental death c. Playmate moved away d. Sibling was sick 2 days

ANS: B A child's stage of development and chronological age will influence grieving. Toddlers can show grief from a loss of parent(s) through changes in their eating patterns, changes in their sleeping patterns, fussiness, and changes in their bowel and bladder habits. It is common for younger children to regress when under increased stress. Siblings being sick, dietary changes, and playmates moving away are unlikely to cause wetting.

2. The nurse is teaching a new nurse about protocols. Which information from the new nurse indicates a correct understanding of the teaching? a.Protocols are guidelines to follow that replace the nursing care plan. b.Protocols assist the clinician in making decisions and choosing interventions for specific health care problems or conditions. c.Protocols are policies designating each nurse's duty according to standards of care and a code of ethics. d.Protocols are prescriptive order forms that help individualize the plan of care.

ANS: B A clinical practice guideline or protocol is a systematically developed set of statements that helps nurses, physicians, and other health care providers make decisions about appropriate health care for specific clinical situations. This guideline establishes interventions for specific health care problems or conditions. The protocol does not replace the nursing care plan. Evidence-based guidelines from protocols can be incorporated into an individualized plan of care. A clinical guideline is not the same as a hospital policy. Standing orders contain orders for the care of a specific group of patients. A protocol is not a prescriptive order form like a standing order.

2. Which diagnosis will the nurse document in a patient's care plan that is NANDA-I approved? a.Sore throat b.Acute pain c.Sleep apnea d.Heart failure

ANS: B Acute pain is the only NANDA-I approved diagnosis listed. Sleep apnea and heart failure are medical diagnoses, and sore throat is subjective data.

10. The nurse is encouraging the postoperative patient to utilize diaphragmatic breathing. Which priority goal is the nurse trying to achieve? a. Manage pain b. Prevent atelectasis c. Reduce healing time d. Decrease thrombus formation

ANS: B After surgery, patients may have reduced lung volume and may require greater effort to cough and deep breathe; inadequate lung expansion can lead to atelectasis and pneumonia. Purposely utilizing diaphragmatic breathing can decrease this risk. During general anesthesia, the lungs are not fully inflated during surgery and the cough reflex is suppressed, so mucus collects within airway passages. Diaphragmatic breathing does not manage pain; in some cases, if splinting and pain medications are not given, it can cause pain. Diaphragmatic breathing does not reduce healing time or decrease thrombus formation. Better, more effective interventions are available for these situations.

30. The nurse is concerned about the skin integrity of the patient in the intraoperative phase of surgery. Which action will the nurse take to minimize skin breakdown? a. Encouraging the patient to bathe before surgery b. Securing attachments to the operating table with foam padding c. Periodically adjusting the patient during the surgical procedure d. Measuring the time a patient is in one position during surgery

ANS: B Although it may be necessary to place a patient in an unusual position, try to maintain correct alignment and protect the patient from pressure, abrasion, and other injuries. Special mattresses, use of foam padding, and attachments to the operating suite table provide protection for the extremities and bony prominences. Bathing before surgery helps to decrease the number of microbes on the skin. Periodically adjusting the patient during the surgical procedure is impractical and can present a safety issue with regard to maintaining sterility of the field and maintaining an airway. Measuring the time the patient is in one position may help with monitoring the situation but does not prevent skin breakdown.

20. The nurse explains the pain relief measures available after surgery during preoperative teaching for a surgical patient. Which comment from the patient indicates the need for additional education on this topic? a. "I will be asked to rate my pain on a pain scale." b. "I will have minimal pain because of the anesthesia." c. "I will take the pain medication as the provider prescribes it." d. "I will take my pain medications before doing postoperative exercises."

ANS: B Anesthesia will be provided during the procedure itself, and the patient should not experience pain during the procedure; however, this will not minimize the pain after surgery. Pain management is utilized after the postoperative phase. Inform the patient of interventions available for pain relief, including medication, relaxation, and distraction. The patient needs to know and understand how to take the medications that the health care provider will prescribe postoperatively. During the stay in the facility, the level of pain is frequently assessed by the nurses. Coordinating pain medication with postoperative exercises helps to minimize discomfort and allows the exercises to be more effective.

6. A staff member verbalizes satisfaction in working on a particular nursing unit because of the freedom of choice and responsibility for the choices. This nurse highly values which element of shared decision making? a. Authority b. Autonomy c. Responsibility d. Accountability

ANS: B Autonomy is freedom of choice and responsibility for the choices. Authority refers to legitimate power to give commands and make final decisions specific to a given position. Responsibility refers to the duties and activities that an individual is employed to perform. Accountability refers to individuals being answerable for their actions.

36. The postanesthesia care unit (PACU) nurse transports the inpatient surgical patient to the medical-surgical floor. Before leaving the floor, the medical-surgical nurse obtains a complete set of vital signs. What is the rationale for this nursing action? a. This is done to complete the first action in a head-to-toe assessment. b. This is done to compare and monitor for vital sign variation during transport. c. This is done to ensure that the medical-surgical nurse checks on the postoperative patient. d. This is done to follow hospital policy and procedure for care of the surgical patient.

ANS: B Before the PACU nurse leaves the acute care area, the staff nurse assuming care for the patient takes a complete set of vital signs to compare with PACU findings. Minor vital sign variations normally occur after transporting the patient. The PACU nurse reviews the patient's information with the medical-surgical nurse, including the surgical and PACU course, physician orders, and the patient's condition. While vital signs may or may not be the first action in a head-to-toe assessment, this is not the rationale for this situation. While following policy or ascertaining that the floor nurse checks on the patient are good reasons for safe care, they are not the best rationale for obtaining vital signs.

14. The nurse is caring for a postoperative patient with an abdominal incision. The nurse provides a pillow to use during coughing. Which activity is the nurse promoting? a. Pain relief b. Splinting c. Distraction d. Anxiety reduction

ANS: B Deep breathing and coughing exercises place additional stress on the suture line and cause discomfort. Splinting incisions with hands and a pillow provides firm support and reduces incisional pull. Providing a pillow during coughing does not provide distraction or reduce anxiety. Providing a pillow does not provide pain relief. Coughing can increase anxiety because it can cause pain. Analgesics provide pain relief.

2. A nurse is developing a plan of care for an older adult. Which information will the nurse consider? a.Should be standardized because most geriatric patients have the same needs b.Needs to be individualized to the patient's unique needs c.Focuses on the disabilities that all aging persons face d.Must be based on chronological age alone

ANS: B Every older adult is unique, and the nurse needs to approach each one as a unique individual. The nursing care of older adults poses special challenges because of great variation in their physiological, cognitive, and psychosocial health. Aging does not automatically lead to disability and dependence. Chronological age often has little relation to the reality of aging for an older adult.

18. The nurse is intervening for a family member with role strain. Which direct care nursing intervention is most appropriate? a.Assisting with activities of daily living b.Counseling about respite care options c.Teaching range-of-motion exercises d.Consulting with a social worker

ANS: B Family caregivers need assistance in adjusting to the physical and emotional demands of caregiving. Sometimes they need respite (i.e., a break from providing care). Counseling is an example of a direct care nursing intervention. The other options do not address the identified problem of role strain (activities of daily living and range-of-motion exercises). Consulting is an indirect care nursing intervention.

24. Which patient statement is the most reliable indicator that an older adult has the correct understanding of health promotion activities? a."I need to increase my fat intake and limit protein." b."I still keep my dentist appointments even though I have partials now." c."I should discontinue my fitness club membership for safety reasons." d."I'm up-to-date on my immunizations, but at my age, I don't need the influenza vaccine."

ANS: B General preventive measures for the nurse to recommend to older adults include keeping regular dental appointments to promote good oral hygiene, eating a low-fat, well-balanced diet, exercising regularly, and maintaining immunizations for seasonal influenza, tetanus, diphtheria and pertussis, shingles, and pneumococcal disease.

10. A new nurse is working in a unit that uses interdisciplinary collaboration. Which action will the nurse take? a.Act as a leader of the health care team. b.Develop good communication skills. c.Work solely with nurses. d.Avoid conflict.

ANS: B Good communication between other health care providers builds trust and is related to the acceptance of your role in the health care team. As a beginning nurse, you will not be considered a leader of the health care team, but your input as an interdisciplinary team member is critical. Interdisciplinary involves other health care providers, not just nurses. Organizational culture includes leadership, communication processes, shared beliefs about the quality of clinical guidelines, and conflict resolution.

12. A nurse is teaching a health promotion class for older adults. In which order will the nurse list the most common to least common conditions that can lead to death in older adults? 1. Chronic obstructive lung disease 2. Cerebrovascular accidents 3. Heart disease 4. Cancer a.4, 1, 2, 3 b.3, 4, 1, 2 c.2, 3, 4, 1 d.1, 2, 3, 4

ANS: B Heart disease is the leading cause of death in older adults followed by cancer, chronic lung disease, and stroke (cerebrovascular accidents).

13. A new nurse writes the following nursing diagnoses on a patient's care plan. Which nursing diagnosis will cause the nurse manager to intervene? a.Wandering b.Hemorrhage c.Urinary retention d.Impaired swallowing

ANS: B Hemorrhage is a collaborative problem, not a nursing diagnosis; the nurse manager will need to correct this misunderstanding with the new nurse. Nurses manage collaborative problems such as hemorrhage, infection, and paralysis using medical, nursing, and allied health (e.g., physical therapy) interventions. Wandering, urinary retention, and impaired swallowing are all examples of nursing diagnoses.

9. The nurse is prescreening a surgical patient in the preadmission testing unit. The medication history indicates that the patient is currently taking an anticoagulant. Which action should the nurse take when consulting with the health care provider? a. Ask for a radiological examination of the chest. b. Ask for an international normalized ratio (INR). c. Ask for a blood urea nitrogen (BUN). d. Ask for a serum sodium (Na).

ANS: B INR, PT (prothrombin time), APTT (activated partial thromboplastin time), and platelet counts reveal the clotting ability of the blood. Anticoagulants can be utilized for different conditions, but its action is to increase the time it takes for the blood to clot. This action can put the surgical patient at risk for bleeding tendencies. Typically, if at all possible, this medication is held several days before a surgical procedure to decrease this risk. Chest x-ray, BUN, and Na are diagnostic screening tools for surgery but are not specific to anticoagulants.

12. The nurse is caring for a preoperative patient. The nurse teaches the principles and demonstrates leg exercises for the patient. The patient is unable to perform leg exercises correctly. What is the nurse's best next step? a. Encourage the patient to practice at a later date. b. Assess for the presence of anxiety, pain, or fatigue. c. Ask the patient why exercises are not being done. d. Evaluate the educational methods used to educate the patient.

ANS: B If the patient is unable to perform leg exercises, the nurse should look for circumstances that may be impacting the patient's ability to learn. In this case, the patient can be anticipating the upcoming surgery and may be experiencing anxiety. The patient may also be in pain or may be fatigued; both of these can affect the ability to learn. Evaluation of educational methods may be needed, but in this case, principles and demonstrations are being utilized. Asking anyone "why" can cause defensiveness and may not help in attaining the answer. The nurse is aware that the patient is unable to do the exercises. Moving forward without ascertaining that learning has occurred will not help the patient in meeting goals.

1. A nurse is obtaining a history on an older adult. Which finding will the nurse most typically find? a.Lives in a nursing home b.Lives with a spouse c.Lives divorced d.Lives alone

ANS: B In 2012, 57% of older adults in non-institutional settings lived with a spouse (45% of older women, 71% of older men); 28% lived alone (35% of older women, 19% of older men); and only 3.5% of all older adults resided in institutions such as nursing homes or centers. Most older adults have lost a spouse due to death rather than divorce.

8. A patient visiting with family members in the waiting area tells the nurse "I don't feel good, especially in the stomach." What should the nurse do? a.Request that the family leave, so the patient can rest. b.Ask the patient to return to the room, so the nurse can inspect the abdomen. c.Ask the patient when the last bowel movement was and to lie down on the sofa. d.Tell the patient that the dinner tray will be ready in 15 minutes and that may help the stomach feel better.

ANS: B In this case, the environment needs to be conducive to completing a thorough assessment. A patient's care environment needs to be safe and conducive to implementing therapies. When you need to expose a patient's body parts, do so privately by closing room doors or curtains because the patient will then be more relaxed; the patient needs to return to the room for an abdominal assessment for privacy and comfort. The family can remain in the waiting area while the nurse assists the patient back to the room. Beginning the assessment in the waiting area (lie down on the sofa) in the presence of family and other visitors does not promote privacy and patient comfort. Telling the patient that the dinner tray is almost ready is making an assumption that the abdominal discomfort is due to not eating. The nurse needs to perform an assessment first.

11. A nurse is discussing sexuality with an older adult. Which action will the nurse take? a.Ask closed-ended questions about specific symptoms the patient may experience. b.Provide information about the prevention of sexually transmitted infections. c.Discuss the issues of sexuality in a group in a private room. d.Explain that sexuality is not necessary as one ages.

ANS: B Include information about the prevention of sexually transmitted infections when appropriate. Open-ended questions inviting an older adult to explain sexual activities or concerns elicit more information than a list of closed-ended questions about specific activities or symptoms. You need to provide privacy for any discussion of sexuality and maintain a nonjudgmental attitude. Sexuality and the need to express sexual feelings remain throughout the human life span.

8. An Orthodox Jewish rabbi has been pronounced dead. The nursing assistive personnel respectfully ask family members to leave the room and go home as postmortem care is provided. Which statement from the supervising nurse is best? a. "I should have called a male colleague to handle the body." b. "Family members stay with the body until burial the next day." c. "I wish they would go home because we have work to do here." d. "Family will quietly leave after praying and touching the rabbi's head."

ANS: B Jewish culture calls for family members or religious officials to stay with the body until the time of burial. A male provider is unnecessary. Requesting or expecting the family to go home is not providing culturally sensitive care. Hindus and Muslims usually have persons of the same gender handle the body after death. Buddhists often say prayers while touching and standing at the head of the deceased.

21. During a follow-up visit, a female patient is describing new onset of marital discord with her terminally ill spouse to the hospice nurse. Which Kübler-Ross stage of dying is the patient experiencing? a. Denial b. Anger c. Bargaining d. Depression

ANS: B Kübler-Ross's traditional theory involves five stages of dying. The anger stage of adjustment to an impending death can involve resistance, anger at God, anger at people, and anger at the situation. Denial would involve failure to accept a death. Bargaining is an action to delay acceptance of death by bartering. Depression would present as withdrawal from others.

10. A veteran is hospitalized after surgical amputation of both lower extremities owing to injuries sustained during military service. Which type of loss will the nurse focus the plan of care on for this patient? a. Perceived loss b. Situational loss c. Maturational loss d. Uncomplicated loss

ANS: B Loss of a body part from injury is a situational loss. Maturational losses occur as part of normal life transitions across the life span. A perceived loss is uniquely defined by the person experiencing the loss and is less obvious to other people. Uncomplicated loss is not a type of loss; it is a description of normal grief.

5. The patient has presented to the ambulatory surgery center to have a colonoscopy. The patient is scheduled to receive moderate sedation (conscious sedation) during the procedure. How will the nurse interpret this information? a. The procedure results in loss of sensation in an area of the body. b. The procedure requires a depressed level of consciousness. c. The procedure will be performed on an outpatient basis. d. The procedure necessitates the patient to be immobile.

ANS: B Moderate sedation (conscious sedation) is used routinely for procedures that do not require complete anesthesia but rather a depressed level of consciousness. Not all patients who are treated on an outpatient basis receive moderate sedation. Regional anesthesia such as local anesthesia provides loss of sensation in an area of the body. General anesthesia is used for patients who need to be immobile and to not remember the surgical procedure.

15. The nurse establishes trust and talks with a school-aged patient before administering an injection. Which type of implementation skill is the nurse using? a.Cognitive b.Interpersonal c.Psychomotor d.Judgmental

ANS: B Nursing practice includes cognitive, interpersonal, and psychomotor skills. Interpersonal skills involve developing trusting relationships with patients, conveying caring and compassion, and communicating clearly. Cognitive skills include critical thinking and decision-making skills. Psychomotor skill requires the integration of cognitive and motor abilities, such as administering the injection. Being judgmental is not appropriate in nursing; nurses are nonjudgmental.

24. The nurse has administered a preoperative medication to the patient going to surgery. Which action will the nurse take next? a. Notify the operating suite that the medication has been given. b. Instruct the patient to call for help to go to the restroom. c. Waste any unused medication according to policy. d. Ask the patient to sign the consent for surgery.

ANS: B Once a preoperative medication has been administered, instruct the patient to call for help when getting out of bed to prevent falls. For patient safety, explain the purpose of a preoperative medication and its effects. Notifying the operating suite that the medication has been given may be part of a facilities procedure but is not the best next step. It is important to have the patient sign consents before the patient has received medication that may make him/her drowsy. Wasting unused medication according to policy is important but is not the best next step.

7. A nursing assistive personnel (NAP) is caring for a dying patient. Which action by the NAP will cause the nurse to intervene? a. Elevating head of bed b. Making the patient eat c. Giving mouth care every 2 to 4 hours d. Keeping skin clean, dry, and moisturized

ANS: B Patients should never be forced to eat so the nurse will intervene to correct this inappropriate behavior. Eating in the last days of life often causes the patient pain and discomfort. Equally, as the body is shutting down the nutrients in food are not able to be absorbed. Therefore, forcing patients to eat serves no beneficial purpose for the patient. All the rest are correct and do not need the nurse to intervene. Elevating the head of the bed is appropriate and will promote ease of breathing and lung expansion and facilitate postural drainage. Giving mouth care will protect membranes if dehydrated, nauseated, or vomiting. Keeping skin clean, dry, and moisturized will decrease skin discomfort and prevent skin breakdown.

37. The nurse is caring for a patient who will undergo a removal of a lung lobe. Which level of care will the patient require immediately post procedure? a. Acute care—medical-surgical unit b. Acute care—intensive care unit c. Ambulatory surgery d. Ambulatory surgery—extended stay

ANS: B Patients undergoing extensive surgery and requiring anesthesia of long duration recover slowly. If a patient is undergoing major surgery such as a procedure on the lung, a stay in the hospital and specifically in the intensive care unit is required to monitor for potential risks to well-being. This patient would require more care than can be provided on a medical-surgical unit. It is not appropriate for this type of patient to go home after the procedure or to stay in an extended stay area of an ambulatory surgery area because of the complexity and associated risks.

2. The nurse is caring for a patient who is scheduled to undergo a surgical procedure. The nurse is completing an assessment and reviews the patient's laboratory tests and allergies and prepares the patient for surgery. In which perioperative nursing phase is the nurse working? a. Perioperative b. Preoperative c. Intraoperative d. Postoperative

ANS: B Reviewing the patient's laboratory tests and allergies is done before surgery in the preoperative phase. Perioperative means before, during, and after surgery. Intraoperative means during the surgical procedure in the operating suite; postoperative means after the surgery and could occur in the postanesthesia care unit, in the ambulatory surgical area, or on the hospital unit.

27. The circulating nurse is caring for a patient intraoperatively. Which primary role of the circulating nurse will be implemented? a. Suturing the surgical incision in the OR suite b. Managing patient care activities in the OR suite c. Assisting with applying sterile drapes in the OR suite d. Handing sterile instruments and supplies to the surgeon in the OR suite

ANS: B The circulating nurse is an RN who remains unscrubbed and uses the nursing process in the management of patient care activities in the OR suite. The circulating nurse also manages patient positioning, antimicrobial skin preparation, medications, implants, placement and function of intermittent pneumatic compression (IPC) devices, specimens, warming devices and surgical counts of instruments, and dressings. The RN first assistant collaborates with the surgeon by handling and cutting tissue, using instruments and medical devices, providing exposure of the surgical area and hemostasis, and suturing. The scrub nurse, who can be a registered nurse, a licensed practical nurse, or a surgical technologist, maintains the sterile field, assists with applying the sterile drapes, and hands sterile instruments and supplies to the surgeon.

7. The nurse is preparing a patient for surgery. Which goal is a priority for assessing the patient before surgery? a. Plan for care after the procedure. b. Establish a patient's baseline of normal function. c. Educate the patient and family about the procedure. d. Gather appropriate equipment for the patient's needs.

ANS: B The goal of the preoperative assessment is to identify a patient's normal preoperative function and the presence of any risks to recognize, prevent, and minimize possible postoperative complications. Gathering appropriate equipment, planning care, and educating the patient and family are all important interventions that must be provided for the surgical patient; they are part of the nursing process but are not the priority reason/goal for completing an assessment of the surgical patient.

17. A nursing assistive personnel (NAP) reports seeing a reddened area on the patient's hip while bathing the patient. Which action should the nurse take? a. Request a wound nurse consult. b. Go to the patient's room to assess the patient's skin. c. Document the finding per the NAP's report. d. Ask the NAP to apply a dressing over the reddened area.

ANS: B The nurse needs to assess the patient's skin. Assessment should not be delegated; it is the responsibility of the licensed registered nurse. The nurse needs to document the assessment findings objectively, not subjectively, per the nursing assistive personnel. Before requesting a consult or determining treatment, the nurse needs to assess the skin.

18. Which question would be most appropriate for a nurse to ask a patient to assist in establishing a nursing diagnosis of Diarrhea? a."What types of foods do you think caused your upset stomach?" b."How many bowel movements a day have you had?" c."Are you able to get to the bathroom in time?" d."What medications are you currently taking?"

ANS: B The nurse needs to first ensure that the symptoms support the diagnosis. By definition, diarrhea means that a patient is having frequent stools; therefore, asking about the number of bowel movements is most appropriate. Asking about irritating foods and medications may help the nurse determine the cause of the diarrhea, but first the nurse needs to make sure the diagnosis is appropriate. Asking the patient if he can make it to the bathroom will help to establish a diagnosis of incontinence, not diarrhea. The question is asking for the most appropriate statement to establish the diagnosis of Diarrhea.

1. After assessing a patient, a nurse develops a standard formal nursing diagnosis. What is the rationale for the nurse's actions? a.To form a language that can be encoded only by nurses b.To distinguish the nurse's role from the physician's role c.To develop clinical judgment based on other's intuition d.To help nurses focus on the scope of medical practice

ANS: B The standard formal nursing diagnosis serves several purposes. Nursing diagnoses distinguish the nurse's role from that of the physician/health care provider and help nurses focus on the scope of nursing practice (not medical) while fostering the development of nursing knowledge. A nursing diagnosis provides the precise definition that gives all members of the health care team a common language for understanding the patient's needs. A diagnosis is a clinical judgment based on information.

10. A nurse is observing for the universal loss in an older-adult patient. What is the nurse assessing? a.Loss of finances through changes in income b.Loss of relationships through death c.Loss of career through retirement d.Loss of home through relocation

ANS: B The universal loss for older adults usually revolves around the loss of relationships through death. Life transitions, of which loss is a major component, include retirement and the associated financial changes, changes in roles and relationships, alterations in health and functional ability, changes in one's social network, and relocation. However, these are not the universal loss.

11. The patient database reveals that a patient has decreased oral intake, decreased oxygen saturation when ambulating, reports of shortness of breath when getting out of bed, and a productive cough. Which elements will the nurse identify as defining characteristics for the diagnostic label of Activity intolerance? a.Decreased oral intake and decreased oxygen saturation when ambulating b.Decreased oxygen saturation when ambulating and reports of shortness of breath when getting out of bed c.Reports of shortness of breath when getting out of bed and a productive cough d.Productive cough and decreased oral intake

ANS: B There are defining characteristics (observable assessment cues such as patient behavior, physical signs) that support each problem-focused diagnostic judgment. The signs and symptoms, or defining characteristics, for the diagnosis Activity intolerance include decreased oxygen saturation when ambulating and reports of shortness of breath when getting out of bed. The key to supporting the diagnosis of Activity intolerance is that only these two characteristics involve how the patient tolerates activity. Decreased oral intake and productive cough do not define activity intolerance.

3. A nurse is working in an intensive care unit (critical care). Which type of nursing care delivery model will this nurse most likely use? a. Team nursing b. Total patient care c. Primary nursing d. Case-management

ANS: B Total patient care is found primarily in critical care areas. Total patient care involves an RN being responsible for all aspects of care for one or more patients. In the team nursing care model, the RN assumes the role of group or team leader and leads a team made up of other RNs, practical nurses, and nursing assistive personnel. The primary nursing model of care delivery was developed to place RNs at the bedside and improve the accountability of nursing for patient outcomes and the professional relationships among staff members. Case-management is a care approach that coordinates and links health care services to patients and families while streamlining costs.

30.A nurse begins infusing a 250-mL bag of IV fluid at 1845 on Monday and programs the pump to infuse at 50 mL/hr. At what time should the infusion be completed? a. 2300 Monday b. 2345 Monday c. 0015 Tuesday d. 0045 Tuesday

ANS: B 250 mL ÷ 50 mL/hr = 5 hr 1845 + 5 hr = 2345, which would be 2345 on Monday. DIF:Apply (application)REF:978

35.A nurse is administering a blood transfusion. Which assessment finding will the nurse report immediately? a. Blood pressure 110/60 b. Temperature 101.3° F c. Poor skin turgor and pallor d. Heart rate of 100 beats/min

ANS: B A fever should be reported immediately and the blood transfusion stopped. All other assessment findings are expected. Blood is given to elevate blood pressure, improve pallor, and decrease tachycardia. DIF:Apply (application)REF:962 | 964

15.In which patient will the nurse expect to see a positive Chvostek sign? a. A 7-year-old child admitted for severe burns b. A 24-year-old adult admitted for chronic alcohol abuse c. A 50-year-old patient admitted for an acute exacerbation of hyperparathyroidism d. A 75-year-old patient admitted for a broken hip related to osteoporosis

ANS: B A positive Chvostek sign is representative of hypocalcemia or hypomagnesemia. Hypomagnesemia is common with alcohol abuse. Hypocalcemia can be brought on by alcohol abuse and pancreatitis (which also can be affected by alcohol consumption). Burn patients frequently experience extracellular fluid volume deficit. Hyperparathyroidism causes hypercalcemia. Immobility is associated with hypercalcemia. DIF:Apply (application)REF:942

2. A nurse is using the problem-oriented approach to data collection. Which action will the nurse take first? a. Complete the questions in chronological order. b. Focus on the patient's presenting situation. c. Make accurate interpretations of the data. d. Conduct an observational overview.

ANS: B A problem-oriented approach focuses on the patient's current problem or presenting situation rather than on an observational overview. The database is not always completed using a chronological approach if focusing on the current problem. Making interpretations of the data is not data collection. Data interpretation occurs while appropriate nursing diagnoses are assigned. The question is asking about data collection.

A patient with sepsis as a result of long-term leukemia dies 25 hours after admission to the hospital. A full code was conducted without success. The patient had a urinary catheter, an intravenous line, an oxygen cannula, and a nasogastric tube. What question is priority for the nurse to ask the family before beginning postmortem care? a. "Do you want to assist in bathing your loved one?" b. "Is an autopsy going to be done?" c. "To which funeral home do you want your loved one transported?" d. "Do you want me to remove the lines and tubes before you see your loved one?"

ANS: B An autopsy or postmortem examination may be requested by the patient or the patient's family, as part of an institutional policy, or if required by law. Because the patient's death occurred as a result of long-term illness, not under suspicious circumstances, and more than 24 hours after admission to the hospital, whether to conduct a postmortem examination would be decided by the family, and consent would have to be obtained from the family. The nurse needs to know the policy to follow regarding removal of lines when an autopsy is to be done. Asking about bathing the deceased patient is a valid question but is not priority, because the nurse needs to know the protocol to follow if an autopsy is to be done. Finding out which funeral home the deceased patient is to be transported to is valid but is not priority, because other actions must be taken before the deceased patient is transported from the hospital. Removal of lines and tubes is not a decision made by the family if an autopsy is to be done. The nurse must first check the protocol to be followed.

A 17-year-old patient, dying of heart failure, wants to have his organs removed for transplantation after his death. What action by the nurse is correct? a. Prepare the organ donation form for the patient to sign while he is still oriented. b. Instruct the patient to talk with his parents about his desire to donate his organs. c. Notify the physician about the patient's desire to donate his organs. d. Contact the United Network for Organ Sharing after talking with the patient.

ANS: B An individual over age 18 may sign the form allowing organ donation upon death. In this situation, the parents would need to sign the form because the teenager is under age 18. The nurse cannot allow the patient to sign the organ donation document because he is younger than age 18. The physician will be notified about the patient's wishes after the parents agree to donate the organs. The nurse caring for the patient does not contact the United Network for Organ Sharing. A transplant coordinator will be the liaison for this organization.

15. The nurse is emptying an ileostomy pouch for a patient. Which assessment finding will the nurse report immediately? a. Liquid consistency of stool b. Presence of blood in the stool c. Malodorous stool d. Continuous output from the stoma

ANS: B Blood in the stool indicates a problem, and the health care provider should be notified. All other options are expected findings for an ileostomy. The stool should be liquid, there should be an odor, and the output should be continuous. DIF:Analyze (analysis)REF:1153 | 1157

35. A nurse is caring for a patient who has had diarrhea for the past week. Which additional assessment finding will the nurse expect? a. Distended abdomen b. Decreased skin turgor c. Increased energy levels d. Elevated blood pressure

ANS: B Chronic diarrhea can result in dehydration. Patients with chronic diarrhea are dehydrated with decreased skin turgor and blood pressure. Diarrhea also causes loss of electrolytes, nutrients, and fluid, which decreases energy levels. A distended abdomen could indicate constipation. DIF:Apply (application)REF:1152

19. A new nurse is completing an assessment on an 80-year-old patient who is alert and oriented. The patient's daughter is present in the room. Which action by the nurse will require follow-up by the charge nurse? a. The nurse makes eye contact with the patient. b. The nurse speaks only to the patient's daughter. c. The nurse leans forward while talking with the patient. d. The nurse nods periodically while the patient is speaking.

ANS: B Gathering data from family members is acceptable, but when a patient is able to interact, nurses need to include information from the older adult to complete the assessment. Therefore, the charge nurse must correct this misconception. When assessing an older adult, nurses need to listen carefully and allow the patient to speak. Positive nonverbal communication, such as making eye contact, nodding, and leaning forward, shows interest in the patient. Thus, the charge nurse does not need to intervene or follow up.

22. Before administering a cleansing enema to an 80-year-old patient, the patient says "I don't think I will be able to hold the enema." Which is the next priority nursing action? a. Rolling the patient into right-lying Sims' position b. Positioning the patient in the dorsal recumbent position on a bedpan c. Inserting a rectal plug to contain the enema solution after administering d. Assisting the patient to the bedside commode and administering the enema

ANS: B If you suspect the patient of having poor sphincter control, position on bedpan in a comfortable dorsal recumbent position. Patients with poor sphincter control are unable to retain all of the enema solution. Administering an enema with the patient sitting on the toilet is unsafe because it is impossible to safely guide the tubing into the rectum, and it will be difficult for the patient to retain the fluid as he or she is in the position used for emptying the bowel. Rolling the patient into right-lying Sims' position will not help the patient retain the enema. Use of a rectal plug to contain the solution is inappropriate and unsafe. DIF:Apply (application)REF:1171

Conjoined twins are in the neonatal department of the community hospital until transfer to the closest medical center. A photographer from the local newspaper gets off the elevator on the neonatal floor and wants to take pictures of the infants. What initial action should the nurse take? a. Escort the cameraman to the neonatal unit while a few pictures are taken quietly. b. Tell the cameraman where the hospital's public relations department is located. c. Ask the cameraman to wait while permission is obtained from the physician. d. Ask the cameraman how the pictures are to be used in the local newspaper.

ANS: B In some cases, information about a scientific discovery or a major medical breakthrough or an unusual situation is newsworthy. In this case, anyone seeking information needs to contact the hospital's public relations department to ensure that invasion of privacy does not occur. It is not the nurse's responsibility to decide independently the legality of disclosing information. The nurse does not have the right to allow the cameraman access to the neonatal unit. This would constitute invasion of privacy. The physician has no responsibility regarding this situation and cannot allow the cameraman on the unit. It is not the nurse's responsibility to find out how the pictures are to be used. This is a task for the public relations department.

A nurse works full-time on the oncology unit at the hospital and works part-time on weekends giving immunizations at the local pharmacy. While giving an injection on a weekend, the nurse caused injury to the patient's arm and is now being sued. How will the hospital's malpractice insurance provide coverage for this nurse? a. It will provide coverage as long as the nurse followed all procedures, protocols, and policies correctly. b. The hospital's malpractice insurance covers this nurse only during the time the nurse is working at the hospital. c. As long as the nurse has never been sued before this incident, the hospital's malpractice insurance will cover the nurse. d. The hospital's malpractice insurance will provide approximately 50% of the coverage the nurse will need.

ANS: B Malpractice insurance provided by the employing institution covers nurses only while they are working within the scope of their employment at that institution. It is always wise to find out if malpractice insurance is provided by a secondary place of employment, in this case, the pharmacy, or the nurse should carry an individual malpractice policy to cover situations such as this.

7.The nurse receives the patient's most recent blood work results. Which laboratory value is of greatest concern? a. Sodium of 145 mEq/L b. Calcium of 15.5 mg/dL c. Potassium of 3.5 mEq/L d. Chloride of 100 mEq/L

ANS: B Normal calcium range is 8.4 to 10.5 mg/dL; therefore, a value of 15.5 mg/dL is abnormally high and of concern. The rest of the laboratory values are within their normal ranges: sodium 136 to 145 mEq/L; potassium 3.5 to 5.0 mEq/L; and chloride 98 to 106 mEq/L. DIF:Apply (application)REF:935

2. The patient tells the nurse that she is afraid to speak up regarding her desire to end care for fear of upsetting her husband and children. Which principle in the nursing code of ethics ensures that the nurse will promote the patient's cause? A. Responsibility B. Advocacy C. Confidentiality D. Accountability

ANS: B Nurses advocate for patients when they support the patient's cause. A nurse's ability to adequate advocate for a patient is based on the unique relationship that develops and the opportunity to better understand the patient's point of view. Responsibility refers to respectingone's professional obligations and following through on promises. Confidentiality deals with privacy issues, and accountability refers to owning one's actions

A pediatric oncology nurse floats to an orthopedic trauma unit. What actions should the nurse manager of the orthopedic unit take to enable safe care to be given by this nurse? a. Provide a complete orientation to the functioning of the entire unit. b. Determine patient acuity and care the nurse can safely provide. c. Allow the nurse to choose which meal time she would like. d. Assign nursing assistive personnel to assist her with care.

ANS: B Nurses who float need to inform the supervisor of any lack of experience in caring for the type of patients on the nursing unit. They also need to request and receive an orientation to the unit. Supervisors are liable if they give a staff nurse an assignment that he or she cannot safely handle. Before accepting employment, learn the policies of the institution regarding floating, and have an understanding as to what is expected. A basic orientation is needed, whereas a complete orientation of the functioning of the entire unit would take a period of time that would exceed what the nurse has to spend on orientation. Allowing the nurse to choose which meal time she would like is a nice gesture of thanks for the nurse, but it does not enable safe care. Having nursing assistive personnel may help the nurse complete basic tasks such as hygiene and turning, but it does not enable safe nursing care that she is ultimately responsible for.

39. A patient is receiving opioids for pain. Which bowel assessment is a priority? a. Clostridium difficile b. Constipation c. Hemorrhoids d. Diarrhea

ANS: B Patients receiving opiates for pain after surgery often require a stool softener or laxative to prevent constipation. C. difficile occurs from antibiotics, not opioids. Hemorrhoids are caused by conditions other than opioids. Diarrhea does not occur as frequently as constipation. DIF:Apply (application)REF:1152

8.The nurse observes that the patient's calcium is elevated. When checking the phosphate level, what does the nurse expect to see? a. Increased b. Decreased c. Equal to calcium d. No change in phosphate

ANS: B Phosphate will decrease. Serum calcium and phosphate have an inverse relationship. When one is elevated, the other decreases, except in some patients with end-stage renal disease. DIF:Apply (application)REF:941

28.A nurse is assessing a patient. Which assessment finding should cause a nurse to further assess for extracellular fluid volume deficit? a. Moist mucous membranes b. Postural hypotension c. Supple skin turgor d. Pitting edema

ANS: B Physical examination findings of deficit include postural hypotension, tachycardia, thready pulse, dry mucous membranes, and poor skin turgor. Pitting edema indicates that the patient may be retaining excess extracellular fluid. DIF:Apply (application)REF:940

25.A nurse is administering a diuretic to a patient and teaching the patient about foods to increase. Which food choices by the patient will best indicate successful teaching? a. Milk and cheese b. Potatoes and fresh fruit c. Canned soups and vegetables d. Whole grains and dark green leafy vegetables

ANS: B Potatoes and fruits are high in potassium. Milk and cheese are high in calcium. Canned soups and vegetables are high in sodium. Whole grains and dark green leafy vegetables are high in magnesium. DIF:Apply (application)REF:941

26. A nurse is pouching an ostomy on a patient with an ileostomy. Which action by the nurse is most appropriate? a. Changing the skin barrier portion of the ostomy pouch daily b. Emptying the pouch if it is more than one-third to one-half full c. Thoroughly cleansing the skin around the stoma with soap and water to remove excess stool and adhesive d. Measuring the correct size for the barrier device while leaving a 1/2-inch space around the stoma

ANS: B Pouches must be emptied when they are one-third to one-half full because the weight of the pouch may disrupt the seal of the adhesive on the skin. The barrier device should be changed every 3 to 7 days unless it is leaking or is no longer effective. Peristomal skin should be gently cleansed vigorous rubbing can cause further irritation or skin breakdown. Avoid soap. It leaves a residue on skin, which may irritate the skin. The pouch opening should fit around the stoma and cover the peristomal skin to prevent contact with the effluent. Excess space, like 1/2 inch, allows fecal matter to have prolonged exposure to skin, resulting in skin breakdown. DIF:Apply (application)REF:1179

13. Which issue has increased the attention paid to quality of life concerns in recent history? A. Health care disparities B. National movement regarding disabled persons C. Aging of the population D. Health care financial reform

ANS: B Quality of life (QOL) is often at the center of ethical dilemmas, including futile care and DNRdiscussions, and has been reshaped in the United States. The national effort to better respect the abilities of the disabled has forced Americans to reconsider the definition of QOL. Health care disparities, an aging population, and health care reform are components impacted by personal definitions of quality but are not the underlying reason why QOL discussions have arisen in the United States

13.Which blood gas result will the nurse expect to observe in a patient with respiratory alkalosis? a. pH 7.60, PaCO2 40 mm Hg, HCO3- 30 mEq/L b. pH 7.53, PaCO2 30 mm Hg, HCO3- 24 mEq/L c. pH 7.35, PaCO2 35 mm Hg, HCO3- 26 mEq/L d. pH 7.25, PaCO2 48 mm Hg, HCO3- 23 mEq/L

ANS: B Respiratory alkalosis should show an alkalotic pH and decreased CO2 (respiratory) values, with a normal HCO3-. In this case, pH 7.53 is alkaline (normal = 7.35 to 7.45), PaCO2 is 30 (normal 35 to 45 mm Hg), and HCO3- is 24 (normal = 22 to 26 mEq/L). A result of pH 7.60, PaCO2 40 mm Hg, HCO3- 30 mEq/L is metabolic alkalosis. pH 7.35, PaCO2 35 mm Hg, HCO3- 26 mEq/L is within normal limits. pH 7.25, PaCO2 48 mm Hg, HCO3- 23 mEq/L is respiratory acidosis. DIF:Analyze (analysis)REF:944-945

11.A 2-year-old child is brought into the emergency department after ingesting a medication that causes respiratory depression. For which acid-base imbalance will the nurse most closely monitor this child? a. Respiratory alkalosis b. Respiratory acidosis c. Metabolic acidosis d. Metabolic alkalosis

ANS: B Respiratory depression leads to hypoventilation. Hypoventilation results in retention of CO2 and respiratory acidosis. Respiratory alkalosis would result from hyperventilation, causing a decrease in CO2 levels. Metabolic acid-base imbalance would be a result of kidney dysfunction, vomiting, diarrhea, or other conditions that affect metabolic acids. DIF:Analyze (analysis)REF:944- 945

11. The nurse has become aware of missing narcotics in the patient care area. Which ethical principle obligates the nurse to report the missing medications? A. Advocacy B. Responsibility C. Confidentiality D. Accountability

ANS: B Responsibility refers to one's willingness to respect and adhere to one's professional obligations. One of the obligations nursing has is to protect patients and communities, including other nurses. If narcotics are missing, this may indicate that patients have not received medications ordered for their care, or it may suggest that a health care professional may be working under the influence. Accountability refers to the ability to answer for one's actions. Advocacy refers to the support of a particular cause. The concept of confidentiality is very important in health care and involves protecting patients' personal health information

Which action will the nurse take after the plan of care for a patient is developed? a. Place the original copy in the chart, so it cannot be tampered with or revised. b. Communicate the plan to all health care professionals involved in the patient's care. c. File the plan of care in the administration office for legal examination. d. Send the plan of care to quality assurance for review.

ANS: B Setting realistic goals and outcomes often means you must communicate these goals and outcomes to caregivers in other settings who will assume responsibility for patient care. The plan of care communicates nursing care priorities to nurses and other health care professionals. Know also that a plan of care is dynamic and changes as the patient's needs change. All health care professionals involved in the patient's care need to be informed of the plan of care. The plan of care is not sent to the administrative office or quality assurance office.

14.A nurse is caring for a patient whose ECG presents with changes characteristic of hypokalemia. Which assessment finding will the nurse expect? a. Dry mucous membranes b. Abdominal distention c. Distended neck veins d. Flushed skin

ANS: B Signs and symptoms of hypokalemia are muscle weakness, abdominal distention, decreased bowel sounds, and cardiac dysrhythmias. Distended neck veins occur in fluid overload. Thready peripheral pulses indicate hypovolemia. Dry mucous membranes and flushed skin are indicative of dehydration and hypernatremia. DIF:Apply (application)REF:942

11. A nurse is conducting a nursing health history. Which component will the nurse address? a. Nurse's concerns b. Patient expectations c. Current treatment orders d. Nurse's goals for the patient

ANS: B Some components of a nursing health history include chief concern, patient expectations, spiritual health, and review of systems. Current treatment orders are located under the Orders section in the patient's chart and are not a part of the nursing health history. Patient concerns, not nurse's concerns, are included in the database. Goals that are mutually established, not nurse's goals, are part of the nursing care plan.

8. Which patient is most at risk for increased peristalsis? a. A 5-year-old child who ignores the urge to defecate owing to embarrassment b. A 21-year-old female with three final examinations on the same day c. A 40-year-old female with major depressive disorder d. An 80-year-old male in an assisted-living environment

ANS: B Stress can stimulate digestion and increase peristalsis, resulting in diarrhea three finals on the same day is stressful. Ignoring the urge to defecate, depression, and age-related changes of the older adult (80-year-old man) are causes of constipation, which is from slowed peristalsis. DIF:Apply (application)REF:1151

A newly hired experienced nurse is preparing to change a patient's abdominal dressing and hasn't done it before at this hospital. Which action by the nurse is best? a. Ask another nurse to do it so the correct method can be viewed. b. Check the policy and procedure manual for the agency's method. c. Change the dressing using the method taught in nursing school. d. Ask the patient how the dressing change has been recently done.

ANS: B The Joint Commission requires accredited hospitals to have written nursing policies and procedures. These internal standards of care are specific and need to be accessible on all nursing units. For example, a policy/procedure outlining the steps to follow when changing a dressing or administering medication provides specific information about how nurses are to perform. The nurse being observed may not be doing the procedure according to the agency's policy or procedure. The procedure taught in nursing school may not be consistent with the policy or procedure for this agency. The patient is not responsible for maintaining the standards of practice. Patient input is important, but it's not what directs nursing practice.

Which action indicates the nurse is using a PICOT question to improve care for a patient? a. Practices nursing based on the evidence presented in court b. Implements interventions based on scientific research c. Uses standardized care plans for all patients. d. Plans care based on tradition

ANS: B The best answer is implementing interventions based on scientific research. Using results of a literature search to a PICOT question can help a nurse decide which interventions to use. Practicing based on evidence presented in court is incorrect. Practice is based on current research. Using standardized care plans may be one example of evidence-based practice, but it is not used on all patients. The nurse must be careful in using standardized care plans to ensure that each patient's plan of care is still individualized. Planning care based on tradition is incorrect because nursing care should be based on current research.

13. Which nursing intervention is most effective in promoting normal defecation for a patient who has muscle weakness in the legs? a. Administer a soapsuds enema every 2 hours. b. Use a mobility device to place the patient on a bedside commode. c. Give the patient a pillow to brace against the abdomen while bearing down. d. Elevate the head of the bed 20 degrees 60 minutes after breakfast while on bedpan.

ANS: B The best way to promote normal defecation is to assist the patient into a posture that is as normal as possible for defecation. Using a mobility device promotes nurse and patient safety. Elevating the head of the bed is appropriate but is not the most effective closer to 30 to 45 degrees is the proper position for the patient on a bedpan, and the patient is not on bed rest so a bedside commode is the best choice. Giving the patient a pillow may reduce discomfort, but this is not the best way to promote defecation. A soapsuds enema is indicated for a patient who needs assistance to stimulate peristalsis. It promotes non-natural defecation. DIF:Apply (application)REF:1151 | 1155

A nurse is completing a care plan. Which intervention is most appropriate for the nursing diagnostic statement Impaired skin integrity related to shearing forces? a. Administer pain medication every 4 hours as needed. b. Turn the patient every 2 hours, even hours. c. Monitor vital signs, especially rhythm. d. Keep the bed side rails up at all times.

ANS: B The most appropriate intervention for the diagnosis of Impaired skin integrity is to turn the patient. Choose interventions to alter the etiological (related to) factor or causes of the diagnosis. The other options do not directly address the shearing forces. The patient may need pain medication, but Acute pain would be another nursing diagnosis. Monitoring vital signs does not have when or how often these should be done. Keeping the side rails up addresses safety, not skin integrity.

16. While completing an admission database, the nurse is interviewing a patient who states "I am allergic to latex." Which action will the nurse take first? a. Immediately place the patient in isolation. b. Ask the patient to describe the type of reaction. c. Proceed to the termination phase of the interview. d. Document the latex allergy on the medication administration record.

ANS: B The nurse should further assess and ask the patient to describe the type of reaction. The patient will not need to be placed in isolation; before terminating the interview or documenting the allergy, health care personnel need to be aware of what type of response the patient suffered.

12.A patient is admitted for a bowel obstruction and has had a nasogastric tube set to low intermittent suction for the past 3 days. Which arterial blood gas values will the nurse expect to observe? a. Respiratory alkalosis b. Metabolic alkalosis c. Metabolic acidosis d. Respiratory acidosis

ANS: B The patient is losing acid from the nasogastric tube so the patient will have metabolic alkalosis. Lung problems will produce respiratory alkalosis or acidosis. Metabolic acidosis will occur when too much acid is in the body like kidney failure. DIF:Apply (application)REF:948-949

10. The nurse is attempting to prompt the patient to elaborate on the reports of daytime fatigue. Which question should the nurse ask? a. "Is there anything that you are stressed about right now that I should know?" b. "What reasons do you think are contributing to your fatigue?" c. "What are your normal work hours?" d. "Are you sleeping 8 hours a night?"

ANS: B The question asking the patient what factors might be contributing to the fatigue will elicit the best open-ended response. Asking whether the patient is stressed and asking if the patient is sleeping 8 hours a night are closed-ended questions eliciting simple yes or no responses. Asking about normal work hours will elicit a matter-of-fact response and does not prompt the patient to elaborate on the daytime fatigue or ask about the contributing reasons.

A confused patient with a urinary catheter, nasogastric tube, and intravenous line keeps touching these needed items for care. The nurse has tried to explain to the patient that he should not touch these lines, but the patient continues. What is the best action by the nurse at this time? a. Apply restraints loosely on the patient's dominant wrist. b. Try other approaches to prevent the patient from touching these care items. c. Notify the health care provider that restraints are needed immediately to maintain the patient's safety. d. Allow the patient to pull out lines to prove that the patient needs to be restrained.

ANS: B The risks associated with the use of restraints are serious. A restraint-free environment is the first goal of care for all patients. Many alternatives to the use of restraints are available, and the nurse should try all of them before notifying the patient's health care provider. The situation states that the patient is touching the items, not trying to pull them out. At this time, the patient's well-being is not at risk. The nurse will have to check on the patient frequently and then will determine if the health care provider needs to be informed of the situation. Restraints can be used (1) only to ensure the physical safety of the resident or other residents, (2) when less restrictive interventions are not successful, and (3) only on the written order of a health care provider. The health care provider needs to know the situation but also needs to know that all approaches possible have been used before writing an order for restraints. Allowing the patient to pull out any of these items could cause harm to the patient.

17. The nurse has attempted to administer a tap water enema for a patient with fecal impaction with no success. The fecal mass is too large for the patient to pass voluntarily. Which is the next priority nursing action? a. Preparing the patient for a second tap water enema b. Obtaining an order for digital removal of stool c. Positioning the patient on the left side d. Inserting a rectal tube

ANS: B When enemas are not successful, digital removal of the stool may be necessary to break up pieces of the stool or to stimulate the anus to defecate. Tap water enemas should not be repeated because of risk of fluid imbalance. Positioning the patient on the left side does not promote defecation. A rectal tube is indicated for a patient with liquid stool incontinence or flatus but would not be applicable or effective for this patient. DIF:Apply (application)REF:1152 | 1166

A patient's condition is slowly deteriorating. What actions should the nurse take to provide the best care possible? (Select all that apply.) a. Allow the nursing student to receive verbal orders from the physician in the room while the nurse is in the medication area down the hall. b. Document the patient's status changes in the medical record in a timely manner. c. Document that the health care provider has been notified of the specific patient status, including date and time that messages were left. d. Check the chart for frequent orders. e. Omit charting what the health provider's response is to notification of the patient's status change.

ANS: B, C Clear, concise, and timely communication is essential whenever charting in the patient's medical record occurs. Nursing students are not permitted to receive verbal orders. Documentation regarding communication with the health care provider must contain what was communicated by the nurse and the health care provider, orders if given, date, time, and identification of who is documenting the situation.

2. The nurse is using a forced air warmer for a surgical patient preoperatively. Which goals is the nurse trying to achieve? (Select all that apply.) a. Induce shivering. b. Reduce blood loss. c. Induce pressure ulcers. d. Reduce cardiac arrests. e. Reduce surgical site infection.

ANS: B, D, E Evidence suggests that pre-warming for a minimum of 30 minutes may reduce the occurrence of hypothermia. Prevention of hypothermia (core temperature < 36° C) helps to reduce complications such as shivering, cardiac arrest, blood loss, SSI, pressure ulcers, and mortality.

5. The nurse is providing preoperative education and reviews with the patient what it will be like to be in the surgical environment. Which points should the nurse include in the teaching session? (Select all that apply.) a. The operative suite will be very dark. b. The family is not allowed in the operating suite. c. The operating table or bed will be comfortable and soft. d. The nurses will be there to assist you through this process. e. The surgical staff will be dressed in special clothing with hats and masks.

ANS: B, D, E The surgical staff is dressed in special clothing, hats, and masks—all for infection control. Families are not allowed in the operating suite for several reasons, which include infection control and sterility. The nurse is there as the coordinator and patient advocate during a surgical procedure. The rooms are very bright so everyone can see, and the operating table is very uncomfortable for the patient.

1. A nurse is documenting end-of-life care. Which information will the nurse include in the patient's electronic medical record? (Select all that apply.) a. Reason for the death b. Time and date of death c. How ethically the family grieved d. Location of body identification tags e. Time of body transfer and destination

ANS: B, D, E Documentation of end-of-life care includes the following: time and date of death, location of body identification tags, time of body transfer and destination and personal articles left on and secured to the body. Reason for the death is not appropriate; this is a medical judgment and not a nursing judgment. How ethically the family grieved is judgmental and does not belong in the chart. We must remain open to the varying views and beliefs of grieving that are in contrast to our own in order to best support and care for our patients and their families.

3.A nurse is discontinuing a patient's peripheral IV access. Which actions should the nurse take? (Select all that apply.) a. Wear sterile gloves and a mask. b. Stop the infusion before removing the IV catheter. c. Use scissors to remove the IV site dressing and tape. d. Apply firm pressure with sterile gauze during removal. e. Keep the catheter parallel to the skin while removing it. f. Apply pressure to the site for 2 to 3 minutes after removal.

ANS: B, E, F The nurse should stop the infusion before removing the IV catheter, so the fluid does not drip on the patient's skin; keep the catheter parallel to the skin while removing it to reduce trauma to the vein; and apply pressure to the site for 2 to 3 minutes after removal to decrease bleeding from the site. Scissors should not be used because they may accidentally cut the catheter or tubing or may injure the patient. During removal of the IV catheter, light pressure, not firm pressure, is indicated to prevent trauma. Clean gloves are used for discontinuing a peripheral IV access because gloved hands will handle the external dressing, tubing, and tape, which are not sterile. DIF:Understand (comprehension)REF:985-986

18. The nurse is caring for a patient supported with a ventilator who has been unresponsive since arrival via ambulance 8 days ago. The patient has not been identified, and no family members have been found. The nurse is concerned about the plan of care regarding maintenance or withdrawal of life support measures. The nurse determines that this is an ethical dilemma not resolved by scientific data. Place the steps the nurse will use to resolve this ethical dilemma in the correct order. A. The nurse identifies possible solutions or actions to resolve the dilemma. B. The nurse reviews the medical record, including entries by all healthcare disciplines, to gather information relevant to this patient's situation. C. Health care providers use negotiation to redefine the patient's plan of care. D. The nurse evaluates the plan and revises it with input from other health care providers as necessary. E. The nurse arranges a meeting with health care team members to clarify opinions, values, and facts. F. The nurse states the problem.

ANS: B, E, F, A, C, D Using the steps of processing an ethical dilemma, once the nurse identifies that an ethical dilemma exists, the nurse then gathers information relevant to the case; clarifies values and distinguishes between fact, opinion, and values; and verbalizes the problem. Then the nurse identifies possible solutions or actions, works with the health care team to negotiate a plan, and evaluates the plan over time.

8. A patient with a spinal cord injury is seeking to enhance urinary elimination abilities by learning self-catheterization versus assisted catheterization by home health nurses and family members. The nurse adds Readiness for enhanced urinary elimination in the care plan. Which type of diagnosis did the nurse write? a.Risk b.Problem focused c.Health promotion d.Collaborative problem

ANS: C A health promotion nursing diagnosis is a clinical judgment concerning motivation and desire to increase well-being and actualize human health potential. A problem-focused nursing diagnosis describes a clinical judgment concerning an undesirable human response to a health condition/life process that exists in an individual, family, or community. A risk nursing diagnosis is a clinical judgment concerning the vulnerability of an individual, family, group or community for developing an undesirable human response to health conditions/life processes. A collaborative problem is an actual or potential physiological complication that nurses monitor to detect the onset of changes in a patient's health status.

10. A nurse adds the following diagnosis to a patient's care plan: Constipation related to decreased gastrointestinal motility secondary to pain medication administration as evidenced by the patient reporting no bowel movement in seven days, abdominal distention, and abdominal pain. Which element did the nurse write as the defining characteristic? a.Decreased gastrointestinal motility b.Pain medication c.Abdominal distention d.Constipation

ANS: C Abdominal distention, no reported bowel movement, and abdominal pain are the defining characteristics. Decreased gastrointestinal motility secondary to pain medication is an etiology or related to factor. Constipation (problem or NANDA-1 diagnosis) is the identified problem derived from the defining characteristics.

34. The nurse is monitoring a patient in the postanesthesia care unit (PACU) for postoperative fluid and electrolyte imbalance. Which action will be most appropriate for the nurse to take? a. Encourage copious amounts of water. b. Start an additional intravenous (IV) line. c. Measure and record all intake and output. d. Weigh the patient and compare with preoperative weight.

ANS: C Accurate recording of intake and output assesses renal and circulatory function. Measure and record all sources of intake and output. Encouraging copious amounts of water in a postoperative patient might encourage nausea and vomiting. In the PACU, it is impractical to weigh the patient while waking from surgery, but in the days afterward, it is a good assessment parameter for fluid imbalance. Starting an additional IV is not necessary and is not important at this juncture.

9. A nurse is caring for an older adult. Which goal is priority? a.Adjusting to career b.Adjusting to divorce c.Adjusting to retirement d.Adjusting to grandchildren

ANS: C Adjusting to retirement is one of the developmental tasks for an older person. A young or middle-aged adult has to adjust to career and/or divorce. A middle-aged adult has to adjust to grandchildren.

11. The nurse is caring for a postoperative patient on the medical-surgical floor. Which activity will the nurse encourage to prevent venous stasis and the formation of thrombus? a. Diaphragmatic breathing b. Incentive spirometry c. Leg exercises d. Coughing

ANS: C After general anesthesia, circulation slows, and when the rate of blood slows, a greater tendency for clot formation is noted. Immobilization results in decreased muscular contractions in the lower extremities; these promote venous stasis. Coughing, diaphragmatic breathing, and incentive spirometry are utilized to decrease atelectasis and pneumonia.

7. The nurse is caring for a patient who requires a complex dressing change. While in the patient's room, the nurse decides to change the dressing. Which action will the nurse take just before changing the dressing? a.Gathers and organizes needed supplies b.Decides on goals and outcomes for the patient c.Assesses the patient's readiness for the procedure d.Calls for assistance from another nursing staff member

ANS: C Always be sure a patient is physically and psychologically ready for any interventions or procedures. After determining the patient's readiness for the dressing change, the nurse gathers needed supplies. The nurse establishes goals and outcomes before intervening. The nurse needs to ask another staff member to help if necessary after determining readiness of the patient.

4. The nurse is caring for a patient in preadmission testing. The patient has been assigned a physical status classification by the American Society of Anesthesiologists of ASA III. Which assessment will support this classification? a. Normal, healthy patient b. Denial of any major illnesses or conditions c. Poorly controlled hypertension with implanted pacemaker d. Moribund patient not expected to survive without the operation

ANS: C An ASA III rating is a patient with a severe systemic disease, such as poorly controlled hypertension with an implanted pacemaker. ASA I is a normal healthy patient with no major illnesses or conditions. ASA II is a patient with mild systemic disease. ASA V is a moribund patient who is not expected to survive without the operation and includes patients with ruptured abdominal/thoracic aneurysm or massive trauma.

7. What is the best suggestion a nurse could make to a family requesting help in selecting a local nursing center? a.Have the family members evaluate nursing home staff according to their ability to get tasks done efficiently and safely. b.Make sure that nursing home staff members get patients out of bed and dressed according to staff's preferences. c.Explain that it is important for the family to visit the center and inspect it personally. d.Suggest a nursing center that has standards as close to hospital standards as possible.

ANS: C An important step in the process of selecting a nursing home is to visit the nursing home. The nursing home should not feel like a hospital. It is a home, a place where people live. Members of the nursing home staff should focus on the person, not the task. Residents should be out of bed and dressed according to their preferences, not staff preferences.

6. A patient presents to the emergency department following a motor vehicle crash and suffers a right femur fracture. The leg is stabilized in a full leg cast. Otherwise, the patient has no other major injuries, is in good health, and reports only moderate discomfort. Which is the most pertinent nursing diagnosis the nurse will include in the plan of care? a.Posttrauma syndrome b.Constipation c.Acute pain d.Anxiety

ANS: C Based on the assessment data provided, the only supportive evidence for one of the diagnosis options is "Reports only moderate discomfort," which would support Acute pain. No supportive evidence is provided for any of the other diagnoses. The patient may indeed develop signs or symptoms of the other problems, but supportive data are presently lacking in the provided information.

26. The nurse is preparing a patient for a surgical procedure on the right great toe. Which action will be most important to include in this patient's preparation? a. Place the patient in a clean surgical gown. b. Ask the patient to remove all hairpins and cosmetics. c. Ascertain that the surgical site has been correctly marked. d. Determine where the family will be located during the procedure.

ANS: C Because errors have occurred in the past with patients undergoing the wrong surgery on the wrong site, the universal protocol guidelines have been implemented and are used with all invasive procedures. Part of this protocol includes marking the operative site with indelible ink. Knowing where the family is during a procedure, placing the patient in a clean gown, and asking the patient to remove all hairpins and cosmetics are important but are not most important in this list of items.

9. A newly admitted patient who is morbidly obese asks the nurse for assistance to the bathroom for the first time. Which action should the nurse take initially? a.Ask for at least two other assistive personnel to come to the room. b.Medicate the patient to alleviate discomfort while ambulating. c.Review the patient's activity orders. d.Offer the patient a walker.

ANS: C Before beginning care, review the plan to determine the need for assistance and the type required. Before intervening, the nurse must check the patient's orders. For example, if the patient is on bed rest, the nurse will need to explain the use of a bedpan rather than helping the patient get out of bed to go to the bathroom. Asking for assistive personnel is appropriate after making sure the patient can get out of bed. If the patient is obese, the nurse will likely need assistance in getting the patient to the bathroom. Medicating the patient before checking the orders is not advised in this situation. Before medicating for pain, the nurse needs to perform a pain assessment. Offering the patient a walker is a premature intervention until the orders are verified.

12. Vital signs for a patient reveal a high blood pressure of 187/100. Orders state to notify the health care provider for diastolic blood pressure greater than 90. What is the nurse's first action? a.Follow the clinical protocol for a stroke. b.Review the most recent lab results for the patient's potassium level. c.Assess the patient for other symptoms or problems, and then notify the health care provider. d.Administer an antihypertensive medication from the stock supply, and then notify the health care provider.

ANS: C Communication to other health care professionals must be timely, accurate, and relevant to a patient's clinical situation. The best answer is to reassess the patient for other symptoms or problems, and then notify the health care provider according to the orders. Reviewing the potassium level does not address the problem of high blood pressure. The nurse does not follow the protocol since the order says to notify the health care provider. The orders read to notify the health care provider, not administer medications.

16. A charge nurse is evaluating a new nurse's plan of care. Which finding will cause the charge nurse to follow up? a.Assigning a documented nursing diagnosis of Risk for infection for a patient on intravenous (IV) antibiotics b.Completing an interview and physical examination before adding a nursing diagnosis c.Developing nursing diagnoses before completing the database d.Including cultural and religious preferences in the database

ANS: C Developing nursing diagnoses before completion of the database needs to be corrected by the charge nurse. Always identify a nursing diagnosis from the data, not the reverse. The data should be clustered and reviewed to see if any patterns are present before a nursing diagnosis is assigned. Risk for infection is an appropriate diagnosis for a patient with an intravenous (IV) site in place. The IV site involves a break in skin integrity and is a potential source of infection. The diagnostic process should proceed in steps. Completing the interview and physical examination before adding a nursing diagnosis is appropriate. The patient's cultural background and developmental stage are important to include in a patient database.

5. A nurse is using assessment data gathered about a patient and combining critical thinking to develop a nursing diagnosis. What is the nurse doing? a.Assigning clinical cues b.Defining characteristics c.Diagnostic reasoning d.Diagnostic labeling

ANS: C Diagnostic reasoning is defined as a process of using the assessment data gathered about a patient to logically explain a clinical judgment, in this case a nursing diagnosis. Defining characteristics are assessment findings that support the nursing diagnosis. Defining characteristics are the subjective and objective clinical cues, which a nurse gathers intentionally and unintentionally. The nurse organizes all of the patient's data into meaningful and usable data clusters, which lead to a diagnostic conclusion. Diagnostic labeling is simply the name of the diagnosis.

4. A severely depressed patient cannot state any positive attributes to life. The nurse patiently sits with this patient and assists the patient to identify several activities the patient is actually looking forward to in life. Which spiritual concept is the nurse trying to promote? a. Time management b. Reminiscence c. Hope d. Faith

ANS: C Hope gives a person the ability to see life as enduring or having meaning or purpose. The nurse's actions do not address time management, reminiscence, or faith. Time management is organizing and prioritizing activities to be completed in a timely manner. Reminiscence is the relationship by mentally or verbally anecdotally relieving and remembering the person and past experiences. Faith is belief in a higher power.

14. A patient has a bacterial infection in left lower leg. Which nursing diagnosis will the nurse add to the patient's care plan? a.Infection b.Risk for infection c.Impaired skin integrity d.Staphylococcal leg infection

ANS: C Impaired skin integrity is the only nursing diagnosis listed that will correlate to the patient information. While risk for infection is a nursing diagnosis, the patient is not at risk; the patient has an actual infection. Infection can be a medical diagnosis as well as a collaborative problem. Staphylococcal leg infection is a medical diagnosis.

1. A nurse is providing nursing care to patients after completing a care plan from nursing diagnoses. In which step of the nursing process is the nurse? a.Assessment b.Planning c.Implementation d.Evaluation

ANS: C Implementation, the fourth step of the nursing process, formally begins after a nurse develops a plan of care. With a care plan based on clear and relevant nursing diagnoses, a nurse initiates interventions that are designed to assist the patient in achieving the goals and expected outcomes needed to support or improve the patient's health status. The nurse gathers data during the assessment phase and mutually sets goals and prioritizes care during the planning phase. During the evaluation phase, the nurse determines the achievement of goals and effectiveness of interventions.

13. A female nurse is called into the supervisor's office regarding her deteriorating work performance since the loss of her spouse 2 years ago. The woman begins sobbing and says that she is "falling apart" at home as well. Which type of grief is the female nurse experiencing? a. Normal grief b. Perceived grief c. Complicated grief d. Disenfranchised grief

ANS: C In complicated grief, a person has a prolonged or significantly difficult time moving forward after a loss. Normal grief is the most common reaction to death; it involves a complex range of normal coping strategies. Disenfranchised grief involves a relationship that is not socially sanctioned. Perceived grief is not a type of grief; perceived loss is a loss that is less obvious to other people.

21. The nurse is making a preoperative education appointment with a patient. The patient asks if a family member should come to the appointment. Which is the best response by the nurse? a. "There is no need for an additional person at the appointment." b. "Your family can come and wait with you in the waiting room." c. "We recommend including family members at this appointment." d. "It is required that you have a family member at this appointment."

ANS: C Including family members in perioperative education is advisable. Often a family member is a coach for postoperative exercises when the patient returns from surgery. If anxious relatives do not understand routine postoperative events, it is likely that their anxiety will heighten the patient's fears and concerns. Preoperative preparation of family members before surgery helps to minimize anxiety and misunderstanding. An additional person is needed at the appointment if at all possible, and he or she needs to be involved in the process, not just waiting in the waiting room; however, it is certainly not a requirement for actually completing the surgery that someone comes to this appointment.

17. A nurse is helping an older-adult patient with instrumental activities of daily living. The nurse will be assisting the patient with which activity? a.Taking a bath b.Getting dressed c.Making a phone call d.Going to the bathroom

ANS: C Instrumental activities of daily living or IADLs (such as the ability to write a check, shop, prepare meals, or make phone calls) and activities of daily living or ADLs (such as bathing, dressing, and toileting) are essential to independent living.

13. A nurse is observing skin integrity of an older adult. Which finding will the nurse document as a normal finding? a.Oily skin b.Faster nail growth c.Decreased elasticity d.Increased facial hair in men

ANS: C Loss of skin elasticity is a common finding in the older adult. Other common findings include pigmentation changes, glandular atrophy (oil, moisture, and sweat glands), thinning hair (facial hair: decreased in men, increased in women), slower nail growth, and atrophy of epidermal arterioles.

16. The nurse inserts an intravenous (IV) catheter using the correct technique and following the recommended steps according to standards of care and hospital policy. Which type of implementation skill is the nurse using? a.Cognitive b.Interpersonal c.Psychomotor d.Judgmental

ANS: C Nursing practice includes cognitive, interpersonal, and psychomotor skills. Psychomotor skill requires the integration of cognitive and motor abilities. The nurse in this example displayed the psychomotor skill of inserting an intravenous catheter while following standards of care and integrating knowledge of anatomy and physiology. Cognitive involves the application of critical thinking and use of good judgment in making sound clinical decisions. Interpersonal skills involve developing trusting relationships with patients, conveying caring and compassion, and communicating clearly.

1. The nurse is caring for a surgical patient, when the family member asks what perioperative nursing means. How should the nurse respond? a. Perioperative nursing occurs in preadmission testing. b. Perioperative nursing occurs primarily in the postanesthesia care unit. c. Perioperative nursing includes activities before, during, and after surgery. d. Perioperative nursing includes activities only during the surgical procedure.

ANS: C Perioperative nursing care occurs before, during, and after surgery. Preadmission testing occurs before surgery and is considered preoperative. Nursing care provided during the surgical procedure is considered intraoperative, and in the postanesthesia care unit, it is considered postoperative. All of these are parts of the perioperative phase, but each individual phase does not explain the term completely.

5. A nurse is reviewing a patient's care plan. Which information will the nurse identify as a nursing intervention? a.The patient will ambulate in the hallway twice this shift using crutches correctly. b.Impaired physical mobility related to inability to bear weight on right leg. c.Provide assistance while the patient walks in the hallway twice this shift with crutches. d.The patient is unable to bear weight on right lower extremity.

ANS: C Providing assistance to a patient who is ambulating is a nursing intervention. The statement, "The patient will ambulate in the hallway twice this shift using crutches correctly" is a patient outcome. Impaired physical mobility is a nursing diagnosis. The statement that the patient is unable to bear weight and ambulate can be included with assessment data and is a defining characteristic for the diagnosis of Impaired physical mobility.

15. A nurse is assessing an older adult for cognitive changes. Which symptom will the nurse report as normal? a.Disorientation b.Poor judgment c.Slower reaction time d.Loss of language skills

ANS: C Slower reaction time is a common change in the older adult. Symptoms of cognitive impairment, such as disorientation, loss of language skills, loss of the ability to calculate, and poor judgment are not normal aging changes and require further investigation of underlying causes.

8. A 70-year-old patient who suffers from worsening dementia is no longer able to live alone. The nurse is discussing health care services and possible long-term living arrangements with the patient's only son. What will the nurse suggest? a.An apartment setting with neighbors close by b.Having the patient utilize weekly home health visits c.A nursing center because home care is no longer safe d.That placement is irrelevant because the patient is retreating to a place of inactivity

ANS: C Some family caregivers consider nursing center placement when in-home care becomes increasingly difficult or when convalescence from hospitalization requires more assistance than the family is able to provide. An apartment setting and the use of home health visits are not appropriate because living at home is unsafe. Dementia is not a time of inactivity but an impairment of intellectual functioning.

5. A nurse is teaching a group of older-adult patients. Which teaching strategy is best for the nurse to use? a.Provide several topics of discussion at once to promote independence and making choices. b.Avoid uncomfortable silences after questions by helping patients complete their statements. c.Ask patients to recall past experiences that correspond with their interests. d.Speak in a high pitch to help patients hear better.

ANS: C Teaching strategies include the use of past experiences to connect new learning with previous knowledge, focusing on a single topic to help the patient concentrate, giving the patient enough time in which to respond because older adults' reaction times are longer than those of younger persons, and keeping the tone of voice low; older adults are able to hear low sounds better than high-frequency sounds.

19. Which staff member does the nurse assign to provide morning care for an older-adult patient who requires assistance with activities of daily living? a. Licensed practical nurse b. Cardiac monitor technician c. Nursing assistive personnel (NAP) d. Another registered nurse on the floor

ANS: C The NAP is capable of caring for this patient and is the most cost-effective choice. The cardiac monitor technician's role is to watch the cardiac monitors for patients on the floor. The nurse and the licensed practical nurse are not the most cost-effective options in this case, even though each could assist with activities of daily living. These nurses would be better used to administer medications, perform assessments, etc.

12. Which approach will be most appropriate for a nurse to take when faced with the challenge of performing many tasks in one shift? a. Do as much as possible by oneself before seeking assistance from others. b. Evaluate the effectiveness of all tasks when all tasks are completed. c. Complete one task before starting another task. d. Delegate tasks the nurse does not like doing.

ANS: C The appropriate clinical care coordination skill in these options is to complete one task before starting another task. Good time management involves setting goals to help the nurse complete one task before starting another task. Evaluation is ongoing and should not be completed just at the end of task completion. The nurse should not delegate tasks simply because the nurse does not like doing them. The nurse should use delegation skills and time-management skills instead of trying to do as much as possible with no help.

11. A new nurse expresses frustration at not being to complete all interventions for a group of patients in a timely manner. The nurse leaves the rounds report sheets at the nurse's station when caring for patients and reports having to go back and forth between rooms for equipment and supplies. Which type of skill does the nurse need? a. Interpersonal communication b. Clinical decision making c. Organizational d. Evaluation

ANS: C The clinical care coordination skill this nurse needs to improve on is organization. This nurse needs to keep the patient report sheets in hand to anticipate what equipment and supplies a patient is going to need. Then the nurse may not have to leave the room so often; this will save time. The nurse is not having a problem communicating with others (interpersonal communication). The nurse is not having a problem using the nursing process for clinical decisions. The nurse is not having a problem comparing actual patient outcomes with expected outcomes (evaluation).

14. A nurse is making initial rounds on patients. Which intervention for a patient with poor wound healing should the nurse perform first? a.Reinforce the wound dressing as needed with 4 × 4 gauze. b.Perform the ordered dressing change twice daily. c.Observe wound appearance and edges. d.Document wound characteristics.

ANS: C The most appropriate initial intervention is to assess the wound (observe wound appearance and edges). The nurse must assess the wound first before the findings can be documented, reinforcement of the dressing, and the actual skill of dressing changes.

11. "I know it seems strange, but I feel guilty being pregnant after the death of my son last year," said a woman during her routine obstetrical examination. The nurse spends extra time with this woman, helping her realize bonding with this unborn child will not mean she is replacing the one who died. Which nursing technique does this demonstrate? a. Providing curative therapy b. Promoting spirituality c. Facilitating mourning d. Eradicating grief

ANS: C The nurse facilitates mourning in family members who are still surviving. By acknowledging the pregnant woman's emotions, the nurse helps the mother bond with her fetus and recognizes the emotions that still exist for the deceased child. The nurse is not attempting to help the patient eradicate grief, which would be unrealistic. Curative therapy (curing a disease) and spiritual promotion (belief in a higher power or in the meaning of life) are not addressed by the nurse's statement.

17. A patient exhibits the following symptoms: tachycardia, increased thirst, headache, decreased urine output, and increased body temperature. The nurse analyzes the data. Which nursing diagnosis will the nurse assign to the patient? a.Adult failure to thrive b.Hypothermia c.Deficient fluid volume d.Nausea

ANS: C The signs the patient is exhibiting are consistent with deficient fluid volume (dehydration). Even without knowing the clinical manifestations of dehydration, the question can be answered by the process of elimination. Adult failure to thrive, hypothermia, and nausea are not appropriate diagnoses because data are insufficient to support these diagnoses.

6. Family members gather in the emergency department after learning that a family member was involved in a motor vehicle accident. After learning of the family member's unexpected death, the surviving family members begin to cry and scream in despair. Which phase does the nurse determine the family is in according to the Attachment Theory? a. Numbing b. Reorganization c. Yearning and searching d. Disorganization and despair

ANS: C Yearning and searching characterize the second bereavement phase in the Attachment Theory. Emotional outbursts of tearful sobbing and acute distress are common in this phase. During the numbing phase, the family is protected from the full impact of the loss. During disorganization and despair, the reason why the loss occurred is constantly examined. Reorganization is the last stage of the Attachment Theory in which the person accepts the change and builds new relationships.

28. The nurse administers a cathartic to a patient. Which finding helps the nurse determine that the cathartic has a therapeutic effect? a. Reports decreased diarrhea. b. Experiences pain relief. c. Has a bowel movement. d. Passes flatulence.

ANS: C A cathartic is a laxative that stimulates a bowel movement. It would be effective if the patient experiences a bowel movement. The other options are not outcomes of administration of a cathartic. An antidiarrheal will provide relief from diarrhea. Pain medications will provide pain relief. Carminative enemas provide relief from gaseous distention (flatulence). DIF:Analyze (analysis)REF:1151 | 1163

Which information indicates a nurse has a good understanding of a goal? a. It is a statement describing the patient's accomplishments without a time restriction. b. It is a realistic statement predicting any negative responses to treatments. c. It is a broad statement describing a desired change in a patient's behavior. d. It is a measurable change in a patient's physical state.

ANS: C A goal is a broad statement that describes a desired change in a patient's condition or behavior. A goal is mutually set with the patient. An expected outcome is the measurable changes (patient behavior, physical state, or perception) that must be achieved to reach a goal. Expected outcomes are time limited, measurable ways of determining if a goal is met.

37. A patient is receiving a neomycin solution enema. Which primary goal is the nurse trying to achieve? a. Prevent gaseous distention b. Prevent constipation c. Prevent colon infection d. Prevent lower bowel inflammation

ANS: C A medicated enema is a neomycin solution, i.e., an antibiotic used to reduce bacteria in the colon before bowel surgery. Carminative enemas provide relief from gaseous distention. Bulk forming, emollient (wetting), and osmotic laxatives and cathartics help prevent constipation or treat constipation. An enema containing steroid medication may be used for acute inflammation in the lower colon. DIF:Apply (application)REF:1165

36. The nurse is caring for a patient who had a colostomy placed yesterday. The nurse should report which assessment finding immediately? a. Stoma is protruding from the abdomen. b. Stoma is flush with the skin. c. Stoma is purple. d. Stoma is moist.

ANS: C A purple stoma may indicate strangulation/necrosis or poor circulation to the stoma and may require surgical intervention. A stoma should be reddish-pink and moist in appearance. It can be flush with the skin, or it can protrude. DIF:Apply (application)REF:1181

19. The nurse is performing a fecal occult blood test. Which action should the nurse take? a. Test the quality control section before testing the stool specimens. b. Apply liberal amounts of stool to the guaiac paper. c. Report a positive finding to the provider. d. Don sterile disposable gloves.

ANS: C Abnormal findings such as a positive test (turns blue) should be reported to the provider. A fecal occult blood test is a clean procedure sterile gloves are not needed. A thin specimen smear is all that is required. The quality control section should be developed after it is determined whether the sample is positive or negative. DIF:Apply (application)REF:1158

The nurse completes a thorough assessment of a patient and analyzes the data to identify nursing diagnoses. Which step will the nurse take next in the nursing process? a. Assessment b. Diagnosis c. Planning d. Implementation

ANS: C After identifying a patient's nursing diagnoses and collaborative problems, a nurse prioritizes the diagnoses, sets patient-centered goals and expected outcomes, and chooses nursing interventions appropriate for each diagnosis. This is the third step of the nursing process, planning. The assessment phase of the nursing process involves gathering data. The implementation phase involves carrying out appropriate nursing interventions. During the evaluation phase, the nurse assesses the achievement of goals and effectiveness of interventions.

27. The nurse will irrigate a patient's nasogastric (NG) tube. Which action should the nurse take? a. Instill solution into pigtail slowly. b. Check placement after instillation of solution. c. Immediately aspirate after instilling fluid. d. Prepare 60 mL of tap water into Asepto syringe.

ANS: C After instilling saline, immediately aspirate or pull back slowly on syringe to withdraw fluid. Do not introduce saline through blue "pigtail" air vent of Salem sump tube. Checking placement before instillation of normal saline prevents accidental entrance of irrigating solution into lungs. Draw up 30 mL of normal saline into Asepto syringe to minimize loss of electrolytes from stomach fluids. DIF:Apply (application)REF:1177

22.The health care provider asks the nurse to monitor the fluid volume status of a heart failure patient and a patient at risk for clinical dehydration. Which is the most effective nursing intervention for monitoring both of these patients? a. Assess the patients for edema in extremities. b. Ask the patients to record their intake and output. c. Weigh the patients every morning before breakfast. d. Measure the patients' blood pressures every 4 hours.

ANS: C An effective measure of fluid retention or loss is daily weights; each kilogram (2.2 pounds) change is equivalent to 1 liter of fluid gained or lost. This measurement should be performed at the same time every day using the same scale and the same amount of clothing. Although intake and output records are important assessment measures, some patients are not able to keep their own records themselves. Blood pressure can decrease with extracellular volume (ECV) deficit but will not necessarily increase with recent ECV excess (heart failure patient). Edema occurs with ECV excess but not with clinical dehydration. DIF:Apply (application)REF:949 | 968

23. A nurse is providing care to a group of patients. Which patient will the nurse see first? a. A child about to receive a normal saline enema b. A teenager about to receive loperamide for diarrhea c. An older patient with glaucoma about to receive an enema d. A middle-aged patient with myocardial infarction about to receive docusate sodium

ANS: C An enema is contradicted in a patient with glaucoma this patient should be seen first. All the rest are expected. A child can receive normal saline enemas since they are isotonic. Loperamide, an antidiarrheal, is given for diarrhea. Docusate sodium is given to soften stool for patients with myocardial infarction to prevent straining. DIF:Analyze (analysis)REF:1171

A charge nurse is reviewing outcome statements using the SMART approach. Which patient outcome statement will the charge nurse praise to the new nurse? a. The patient will ambulate in hallways. b. The nurse will monitor the patient's heart rhythm continuously this shift. c. The patient will feed self at all mealtimes today without reports of shortness of breath. d. The nurse will administer pain medication every 4 hours to keep the patient free from discomfort.

ANS: C An expected outcome should be patient centered; should address one patient response; should be specific, measurable, attainable, realistic, and timed (SMART approach). The statement "The patient will feed self at all mealtimes today without reports of shortness of breath" includes all SMART criteria for goal writing. "The patient will ambulate in hallways" is missing a time limit. Administering pain medication and monitoring the patient's heart rhythm are nursing interventions; they do not reflect patient behaviors or actions.

An obstetrical nurse comes across an automobile accident. The patient seems to have a crushed upper airway, and while waiting for emergency medical services to arrive, the nurse makes a cut in the trachea and inserts a straw from her purse to provide an airway. The patient survives and has a permanent problem with his vocal cords, making it difficult to talk. Which statement is true regarding the nurse's performance? a. The nurse acted appropriately and saved the patient's life. b. The nurse acted within the guidelines of the Good Samaritan Law. c. The nurse took actions beyond those that are standard and appropriate. d. The nurse should have just stayed with the patient and waited for help.

ANS: C An obstetric nurse would not have been trained in performing a tracheostomy or a cricotomy, and doing so would be beyond what she has been trained or educated to do. The nurse did not do what another nurse would have done in the same situation. The nurse is not protected by the Good Samaritan Law because she acted outside of her scope of practice and training. The nurse should have acted within what she was trained and educated to do in this circumstance, not just stay with the patient.

A recent immigrant who does not speak English is alert and requires hospitalization. What is the initial action that the nurse must take to enable informed consent to be obtained? a. Ask a family member to translate what the nurse is saying. b. Notify the health care provider that the patient doesn't speak English. c. Request an official interpreter to explain the terms of consent. d. Use hand gestures and medical equipment while explaining in English.

ANS: C An official interpreter must be present to explain the terms of consent if a patient speaks only a foreign language. A family member or acquaintance who speaks a patient's language should not interpret health information. Family members can tell those caring for the patient what the patient is saying, but privacy regarding the patient's condition, assessment, etc., must be protected. There is no way that the nurse can know that the family member is translating exactly what the nurse is saying. Privacy must be ensured and accurate information must be provided to the patient. After consent is obtained for treatment, the health care provider would be notified because little can be done without consent. The health care provider needs to have the translator available during the history and physical, as well as at other times, but the first step is to get a translator to obtain informed consent because this is not an emergency situation. Using hand gestures and medical equipment is inappropriate when communicating with a patient who does not understand the language spoken. Certain hand gestures may be acceptable in one culture and not appropriate in another. The medical equipment may be unknown and frightening to the patient, and the patient still doesn't understand what is being said.

10. Which patient will the nurse assess most closely for an ileus? a. A patient with a fecal impaction b. A patient with chronic cathartic abuse c. A patient with surgery for bowel disease and anesthesia d. A patient with suppression of hydrochloric acid from medication

ANS: C Any surgery that involves direct manipulation of the bowel temporarily stops peristalsis. Anesthesia can also cause cessation of peristalsis. This condition, called an ileus, usually lasts about 24 to 48 hours. Fecal impaction, cathartic abuse, and medication to suppress hydrochloric acid will have bowel sounds, but they may be hypoactive or hyperactive. DIF:Apply (application)REF:1151

16. The nurse will anticipate which diagnostic examination for a patient with black tarry stools? a. Ultrasound b. Barium enema c. Endoscopy d. Anorectal manometry

ANS: C Black tarry stools are an indication of bleeding in the GI tract endoscopy would allow visualization of the bleeding. No other option (ultrasound, barium enema, and anorectal manometry) would allow GI visualization. DIF:Apply (application)REF:1157-1158

29. An older adult's perineal skin is dry and thin with mild excoriation. When providing hygiene care after episodes of diarrhea, what should the nurse do? a. Thoroughly scrub the skin with a washcloth and hypoallergenic soap. b. Tape an occlusive moisture barrier pad to the patient's skin. c. Apply a skin protective ointment after perineal care. d. Massage the skin with light kneading pressure.

ANS: C Cleansing with a no-rinse cleanser and application of a barrier ointment should be done after each episode of diarrhea. Tape and occlusive dressings can damage skin. Excessive pressure and massage are inappropriate and may cause skin breakdown. DIF:Apply (application)REF:1169

The nurse performs an intervention for a collaborative problem. Which type of intervention did the nurse perform? a. Dependent b. Independent c. Interdependent d. Physician-initiated

ANS: C Collaborative interventions, or interdependent interventions, are therapies that require the combined knowledge, skill, and expertise of multiple health care professionals. Health care provider-initiated (HCP) interventions are dependent nursing interventions, or actions that require an order from the HCP. Nurse-initiated interventions are the independent nursing interventions, or actions that a nurse initiates without supervision or direction from others.

17. Determinations regarding quality of life are A. Based on a person's ability to act according to ethical principles. B. Based on a patient's self-determination. C. Value judgments that can vary from person to person. D. Consistent and stable over the course of one's lifetime

ANS: C Determinations regarding quality of life are value judgments. This means that they are judgments based on what individuals believe is desirable. Beliefs about what people find desirable vary from person to person

TOPlanningMSC:Management of Care 25. A patient had an ileostomy surgically placed 2 days ago. Which diet will the nurse recommend to the patient to ease the transition of the new ostomy? a. Eggs over easy, whole wheat toast, and orange juice with pulp b. Chicken fried rice with fresh pineapple and iced tea c. Turkey meatloaf with white rice and apple juice d. Fish sticks with sweet corn and soda

ANS: C During the first few days after ostomy placement, the patient should consume easy-to-digest soft foods such as poultry, rice, and noodles. Fried foods can irritate digestion. Foods high in fiber will be useful later in the recovery process but can cause food blockage if the GI tract is not accustomed to digesting with an ileostomy. Foods with indigestible fiber such as sweet corn, popcorn, raw mushrooms, fresh pineapple, and Chinese cabbage could cause this problem. DIF:Apply (application)REF:1168

38. A guaiac test is ordered for a patient. Which type of blood is the nurse checking for in this patient's stool? a. Bright red blood b. Dark black blood c. Microscopic d. Mucoid

ANS: C Fecal occult blood tests are used to test for blood that may be present in stool but cannot be seen by the naked eye (microscopic). This is usually indicative of a gastrointestinal bleed. All other options are incorrect. Detecting bright red blood, dark black blood, and blood that contains mucus (mucoid) is not the purpose of a guaiac test. DIF:Understand (comprehension)REF:1156

17. A patient verbalizes a low pain level of 2 out of 10 but exhibits extreme facial grimacing while moving around in bed. What is the nurse's initial action in response to these observations? a. Proceed to the next patient's room to make rounds. b. Determine the patient does not want any pain medicine. c. Ask the patient about the facial grimacing with movement. d. Administer the pain medication ordered for moderate to severe pain.

ANS: C First, the nurse needs to clarify/verify what was observed with what the patient states. Proceeding to the next room is ignoring this visual cue. The nurse cannot assume the patient does not want pain medicine just because he reports a 2 out of 10 on the pain scale. The nurse should not administer medication for moderate to severe pain if it is not necessary.

5. A nurse is assisting a patient in making dietary choices that promote healthy bowel elimination. Which menu option should the nurse recommend? a. Broccoli and cheese soup with potato bread b. Turkey and mashed potatoes with brown gravy c. Grape and walnut chicken salad sandwich on whole wheat bread d. Dinner salad topped with hard-boiled eggs, cheese, and fat-free dressing

ANS: C Grapes and whole wheat bread are high fiber and should be chosen. Cheese, eggs, potato bread, and mashed potatoes do not contain as much fiber as whole wheat bread. A healthy diet for the bowel should include foods high in bulk-forming fiber. Whole grains, fresh fruit, and fresh vegetables are excellent sources. Foods without much fiber and with high levels of fat can slow down peristalsis, causing constipation. DIF:Apply (application)REF:1151 | 1168

26.The nurse is evaluating the effectiveness of the intravenous fluid therapy in a patient with hypernatremia. Which finding indicates goal achievement? a. Urine output increases to 150 mL/hr. b. Systolic and diastolic blood pressure decreases. c. Serum sodium concentration returns to normal. d. Large amounts of emesis and diarrhea decrease.

ANS: C Hypernatremia is diagnosed by elevated serum sodium concentration. Blood pressure is not an accurate indicator of hypernatremia. Emesis and diarrhea will not stop because of intravenous therapy. Urine output is influenced by many factors, including extracellular fluid volume. A large dilute urine output can cause further hypernatremia. DIF:Apply (application)REF:940 | 966

36.A nurse has just received a bag of packed red blood cells (RBCs) for a patient. What is the longest time the nurse can let the blood infuse? a. 30 minutes b. 2 hours c. 4 hours d. 6 hours

ANS: C Ideally a unit of whole blood or packed RBCs is transfused in 2 hours. This time can be lengthened to 4 hours if the patient is at risk for extracellular volume excess. Beyond 4 hours there is a risk for bacterial contamination of the blood. DIF:Understand (comprehension)REF:962-963

18. A nurse is checking orders. Which order should the nurse question? a. A normal saline enema to be repeated every 4 hours until stool is produced b. A hypertonic solution enema for a patient with fluid volume excess c. A Kayexalate enema for a patient with severe hypokalemia d. An oil retention enema for a patient with constipation

ANS: C Kayexalate binds to and helps excrete potassium, so it would be contraindicated in patients who are hypokalemic (have low potassium). Normal saline enemas can be repeated without risk of fluid or electrolyte imbalance. Hypertonic solutions are intended for patients who cannot handle large fluid volume and are contraindicated for dehydrated patients. Oil retention enemas lubricate the feces in the rectum and colon and are used for constipation. DIF:Analyze (analysis)REF:1165

29.A patient is to receive 1000 mL of 0.9% sodium chloride intravenously at a rate of 125 mL/hr. The nurse is using microdrip gravity drip tubing. Which rate will the nurse calculate for the minute flow rate (drops/min)? a. 12 drops/min b. 24 drops/min c. 125 drops/min d. 150 drops/min

ANS: C Microdrip tubing delivers 60 drops/mL. Calculation for a rate of 125 mL/hr using microdrip tubing: (125 mL/1 hr)(60 drops/1 mL)(1 hr/60 min) = 125 drop/min.

12. A young woman who is pregnant with a fetus exposed to multiple teratogens consents to have her fetus undergo serial PUBS (percutaneous umbilical blood sampling) to examine how exposure affects the fetus over time. Although these tests will not improve the fetus' outcomes and will expose it to some risks, the information gathered may help infants in the future. Which ethical principle is at greatest risk? A. Autonomy B. Fidelity C. Nonmaleficence D. Beneficence

ANS: C Nonmaleficence is the ethical principle that focuses on avoidance of harm or hurt. The nurse must balance risks and benefits of care. Repeated PUBS may place the mother and fetus at risk for infection and increased pain, and may place the mother at risk for increased emotional health stress. Fidelity refers to the agreement to keep promises. Autonomy refers to freedom from external control, and beneficence refers to taking positive actions to help others.

4. The nurse is gathering data on a patient. Which data will the nurse report as objective data? a. States "doesn't feel good" b. Reports a headache c. Respirations 16 d. Nauseated

ANS: C Objective data are observations or measurements of a patient's health status, like respirations. Inspecting the condition of a surgical incision or wound, describing an observed behavior, and measuring blood pressure are examples of objective data. States "doesn't feel good," reports a headache, and nausea are all subjective data. Subjective data include the patient's feelings, perceptions, and reported symptoms. Only patients provide subjective data relevant to their health condition.

40. Which nutritional instruction is a priority for the nurse to advise a patient about with an ileostomy? a. Keep fiber low. b. Eat large meals. c. Increase fluid intake. d. Chew food thoroughly.

ANS: C Patients with ileostomies will digest their food completely but will lose both fluid and salt through their stoma and will need to be sure to replace this to avoid dehydration. A good reminder for patients is to encourage drinking an 8-ounce glass of fluid when they empty their pouch. This helps patients to remember that they have greater fluid needs than they did before having an ileostomy. A low-fiber diet is not necessary. Eating large meals is not advised. While chewing food thoroughly is correct, it is not the priority liquid is the priority. DIF:Analyze (analysis)REF:1168

24.A patient presents to the emergency department with reports of vomiting and diarrhea for the past 48 hours. The health care provider orders isotonic intravenous (IV) therapy. Which IV will the nurse prepare? a. 0.225% sodium chloride (1/4 NS) b. 0.45% sodium chloride (1/2 NS) c. 0.9% sodium chloride (NS) d. 3% sodium chloride (3% NaCl)

ANS: C Patients with prolonged vomiting and diarrhea become hypovolemic. A solution to replace extracellular volume is 0.9% sodium chloride, which is an isotonic solution. 0.225% and 0.45% sodium chloride are hypotonic. 3% sodium chloride is hypertonic. DIF:Apply (application)REF:956

5. A patient expresses fear of going home and being alone. Vital signs are stable and the incision is nearly completely healed. What can the nurse infer from the subjective data? a. The patient can now perform the dressing changes without help. b. The patient can begin retaking all of the previous medications. c. The patient is apprehensive about discharge. d. The patient's surgery was not successful.

ANS: C Subjective data include expressions of fear of going home and being alone. These data indicate (use inference) that the patient is apprehensive about discharge. Expressing fear is not an appropriate sign that a patient is able to perform dressing changes independently. An order from a health care provider is required before a patient is taught to resume previous medications. The nurse cannot infer that surgery was not successful if the incision is nearly completely healed.

3. After reviewing the database, the nurse discovers that the patient's vital signs have not been recorded by the nursing assistive personnel (NAP). Which clinical decision should the nurse make? a. Administer scheduled medications assuming that the NAP would have reported abnormal vital signs. b. Have the patient transported to the radiology department for a scheduled x-ray, and review vital signs upon return. c. Ask the NAP to record the patient's vital signs before administering medications. d. Omit the vital signs because the patient is presently in no distress.

ANS: C The nurse should ask the nursing assistive personnel to record the vital signs for review before administering medicines or transporting the patient to another department. The nurse should not make assumptions when providing high-quality patient care, and omitting the vital signs is not an appropriate action.

3. The patient's son requests to view the documentation in his mother's medical record. What is the nurse's best response to this request? A."I'll be happy to get that for you." B. "You will have to talk to the physician about that." C."You will need your mother's permission." D."You are not allowed to see it.

ANS: C The nurse understands that sharing health information is governed by HIPAA legislation, which defines rights and privileges of patients for protection of privacy. Private health information cannot be shared without the patient's specific permission. The other three responses either are outright false and/or use poor communication techniques.

7. A nurse is gathering information about a patient's habits and lifestyle patterns. Which method of data collection will the nurse use that will best obtain this information? a. Carefully review lab results. b. Conduct the physical assessment. c. Perform a thorough nursing health history. d. Prolong the termination phase of the interview.

ANS: C The nursing health history also includes a description of a patient's habits and lifestyle patterns. Lab results and physical assessment will not reveal as much about the patient's habits and lifestyle patterns as the nursing health history. Collecting data is part of the working phase of the interview.

2. The nurse is caring for patients with ostomies. In which ostomy location will the nurse expect very liquid stool to be present? a. Sigmoid b. Transverse c. Ascending d. Descending

ANS: C The path of digestion goes from the ascending, across the transverse, to the descending and finally passing into the sigmoid therefore, the least formed stool (very liquid) would be in the ascending. DIF:Apply (application)REF:1150

42.While the nurse is taking a patient history, the nurse discovers the patient has a type of diabetes that results from a head injury and does not require insulin. Which dietary change should the nurse share with the patient? a. Reduce the quantity of carbohydrates ingested to lower blood sugar. b. Include a serving of dairy in each meal to elevate calcium levels. c. Drink plenty of fluids throughout the day to stay hydrated. d. Avoid food high in acid to avoid metabolic acidosis.

ANS: C The patient has diabetes insipidus, which places the patient at risk for dehydration and hypernatremia. Dehydration should be prevented by drinking plenty of fluids to replace the extra water excreted in the urine. Foods high in acid are not what causes metabolic acidosis. A reduction in carbohydrates to lower blood sugar will not help a patient with diabetes insipidus but it may help a patient with diabetes mellitus. Calcium-rich dairy products would be recommended for a hypocalcemic patient. DIF:Apply (application)REF:949

19.The nurse is assessing a patient and finds crackles in the lung bases and neck vein distention. Which action will the nurse take first? a. Offer calcium-rich foods. b. Administer diuretic. c. Raise head of bed. d. Increase fluids.

ANS: C The patient is in fluid overload. Raising the head of the bed to ease breathing is the first action. Offering calcium-rich foods is for hypocalcemia, not fluid overload. Administering a diuretic is the second action. Increasing fluids is contraindicated and would make the situation worse. DIF:Apply (application)REF:940 | 950 | 960

A new nurse notes that the health care unit keeps a listing of patient names in a closed book behind the front desk of the nursing station so patients can be located easily. What action is most appropriate for the nurse to take? a. Move the book to the upper ledge of the nursing station for easier access. b. Talk with the nurse manager about the listing being a violation of the Health Insurance Portability and Accountability Act (HIPAA). c. Use the book as needed while keeping it away from individuals not involved in patient care. d. Ask the nurse manager to move the book to a more secluded area.

ANS: C The privacy section of the HIPAA provides standards regarding accountability in the health care setting. These rules include patient rights to consent to the use and disclosure of their protected health information, to inspect and copy their medical record, and to amend mistaken or incomplete information. This document limits who is able to access a patient's record. It establishes the basis for privacy and confidentiality about patients in any manner. The book is located where only staff would have access. It is not the responsibility of the new nurse to move items used by others on the patient unit. The listing is protected as long as it is used appropriately as needed to provide care. There is no need to move the book to a more secluded area.

30. Which action will the nurse take to reduce the risk of excoriation to the mucosal lining of the patient's nose from a nasogastric tube? a. Instill Xylocaine into the nares once a shift. b. Tape tube securely with light pressure on nare. c. Lubricate the nares with water-soluble lubricant. d. Apply a small ice bag to the nose for 5 minutes every 4 hours.

ANS: C The tube constantly irritates the nasal mucosa, increasing the risk of excoriation. Frequent lubrication with a water-soluble lubricant decreases the likelihood of excoriation and is less toxic than oil-based if aspirated. Xylocaine is used to treat sore throat, not nasal mucosal excoriation. While the tape should be secure, pressure will increase excoriation. Ice is not applied to the nose. DIF:Apply (application)REF:1175

23.A nurse is caring for a cancer patient who presents with anorexia, blood pressure 100/60, and elevated white blood cell count. Which primary purpose for starting total parenteral nutrition (TPN) will the nurse add to the care plan? a. Stimulate the patient's appetite to eat. b. Deliver antibiotics to fight off infection. c. Replace fluid, electrolytes, and nutrients. d. Provide medication to raise blood pressure.

ANS: C Total parenteral nutrition is an intravenous solution composed of nutrients and electrolytes to replace the ones the patient is not eating or losing. TPN does not stimulate the appetite. TPN does not contain blood pressure medication or antibiotics. DIF:Apply (application)REF:955

14. Which statement by a nurse indicates a good understanding about the differences between data validation and data interpretation? a. "Data interpretation occurs before data validation." b. "Validation involves looking for patterns in professional standards." c. "Validation involves comparing data with other sources for accuracy." d. "Data interpretation involves discovering patterns in professional standards."

ANS: C Validation, by definition, involves comparing data with other sources for accuracy. Data interpretation involves identifying abnormal findings, clarifying information, and identifying patient problems. The nurse should validate data before interpreting the data and making inferences. The nurse is interpreting and validating patient data, not professional standards.

7. The nurse finds it difficult to care for a patient whose advance directive states that no extraordinary resuscitation measures should be taken. Which step may help the nurse to find resolution in this assignment? A. Call for an ethical committee consult. B. Decline the assignment on religious grounds. C. Scrutinize her own personal values. D. Convince the family to challenge the directive

ANS: C Values develop over time and are influenced by family, schools, religious traditions, and life experiences. The nurse must recognize that no two humans have the same set of experiences, and so differences in values are more likely the norm than the exception. Closer inspection of one's values may be a step in gaining understanding of another person's perspective. Calling for a consult, declining the assignment, and convincing the family to challenge the patient's directive are not ideal resolutions because they do not address the reason for the nurse's discomfort, which is the conflict between the nurse's values and those of the patient.

3.The nurse observes edema in a patient who has venous congestion from right heart failure. Which type of pressure facilitated the formation of the patient's edema? a. Osmotic b. Oncotic c. Hydrostatic d. Concentration

ANS: C Venous congestion increases capillary hydrostatic pressure. Increased hydrostatic pressure causes edema by causing increased movement of fluid into the interstitial area. Osmotic and oncotic pressures involve the concentrations of solutes and can contribute to edema in other situations, such as inflammation or malnutrition. Concentration pressure is not a nursing term.

6. Which method of data collection will the nurse use to establish a patient's database? a. Reviewing the current literature to determine evidence-based nursing actions b. Checking orders for diagnostic and laboratory tests c. Performing a physical examination d. Ordering medications

ANS: C You will learn to conduct different types of assessments: the patient-centered interview during a nursing health history, a physical examination, and the periodic assessments you make during rounding or administering care. A nursing database includes a physical examination. The nurse reviews the current literature in the implementation phase of the nursing process to determine evidence-based actions, and the health care provider is responsible for ordering medications. The nurse uses results from the diagnostic and laboratory tests to establish a patient database, not checking orders for tests.

5. A nurse is working in a facility that has fewer directors with managers and staff able to make shared decisions. In which type of organizational structure is the nurse employed? a. Delegation b. Research-based c. Decentralization d. Philosophy of care

ANS: C The decentralized management structure often has fewer directors, and managers and staff are able to make shared decisions. The American Nurses Association defines delegation as transferring responsibility for the performance of an activity or task while retaining accountability for the outcome. Research-based means care is based upon evidence. A philosophy of care includes the professional nursing staff's values and concerns for the way they view and care for patients. For example, a philosophy addresses the purpose of the nursing unit, how staff works with patients and families, and the standards of care for the work unit.

21. A nurse is developing nursing diagnoses for a group of patients. Which nursing diagnoses will the nurse use? (Select all that apply.) a.Anxiety related to barium enema b.Impaired gas exchange related to asthma c.Impaired physical mobility related to incisional pain d.Nausea related to adverse effect of cancer medication e.Risk for falls related to nursing assistive personnel leaving bedrail down

ANS: C, D Impaired physical mobility and Nausea are the only correctly written nursing diagnoses. All the rest are incorrectly written. Anxiety lists a diagnostic test as the etiology. Impaired gas exchange lists a medical diagnosis as the etiology. Risk for falls has a legally inadvisable statement for an etiology.

1. A nurse is implementing interventions for a group of patients. Which actions are nursing interventions? (Select all that apply.) a.Order chest x-ray for suspected arm fracture. b.Prescribe antibiotics for a wound infection. c.Reposition a patient who is on bed rest. d.Teach a patient preoperative exercises. e.Transfer a patient to another hospital unit.

ANS: C, D, E A nursing intervention is any treatment based on clinical judgment and knowledge that a nurse performs to enhance patient outcomes. Repositioning, teaching, and transferring a patient are examples of nursing interventions. Ordering a chest x-ray and prescribing antibiotics are examples of medical interventions performed by a health care provider.

A nurse is teaching the staff about the benefits of Nursing Outcomes Classification. Which information should the nurse include in the teaching session? (Select all that apply.) a. Includes seven domains for level 1 b. Uses an easy 3-point Likert scale c. Adds objectivity to judging a patient's progress d. Allows choice in which interventions to choose e. Measures nursing care on a national and international level

ANS: C, E Nursing Outcomes Classification (NOC) links outcomes to NANDA International nursing diagnoses. Such a rating system adds objectivity to judging a patient's progress. Using standardized nursing terminologies such as NOC makes it more possible to measure aspects of nursing care on a national and international level. The indicators for each NOC outcome allow measurement of the outcomes at any point on a 5-point Likert scale from most negative to most positive. This resource is an option you can use in selecting goals and outcomes (not interventions) for your patients. The Nursing Interventions Classification model includes three levels: domains, classes, and interventions for ease of use. The seven domains are the highest level (level 1) of the model, using broad terms (e.g., safety and basic physiological) to organize the more specific classes and interventions.

A nurse is completing an assessment. Which findings will the nurse report as subjective data? (Select all that apply.) a. Patient's temperature b. Patient's wound appearance c. Patient describing excitement about discharge d. Patient pacing the floor while awaiting test results e. Patient's expression of fear regarding upcoming surgery

ANS: C, E Subjective data include patient's feelings, perceptions, and reported symptoms. Expressing feelings such as excitement or fear is an example of subjective data. Objective data are observations or measurements of a patient's health status. In this question, the appearance of the wound and the patient's temperature are objective data. Pacing is an observable patient behavior and is also considered objective data.

20. A patient cancels a scheduled appointment because the patient will be attending a Shivah for a family member. Which response by the nurse is best? a. "When families come together for end-of-life decisions, it provides connections." b. "We will reschedule so the appointment does not fall on the Sabbath." c. "Missionary outreach is so important for spiritual comfort." d. "I'm so sorry for your loss."

ANS: D A death has occurred and saying that you are sorry for their loss is appropriate. The Jewish mourning ritual of Shivah is a time period when normal life activities come to a stop. Those mourning welcome friends into the home as a way of honoring the dead and receive support during the mourning period. Cultural variables can influence a person's response to grief. It is not when families come together for end-of-life decisions. It is not because the appointment fell on the Sabbath. It is not about missionary outreach.

7. A staff nurse delegates a task to a nursing assistive personnel (NAP), knowing that the NAP has never performed the task before. As a result, the patient is injured, and the nurse defensively states that the NAP should have known how to perform such a simple task. Which element of the decision-making process is the nurse lacking? a. Authority b. Autonomy c. Responsibility d. Accountability

ANS: D Accountability refers to individuals being answerable for their actions. The nurse in this situation is not taking ownership of the inappropriate delegation of a task. Autonomy is freedom of choice and responsibility for the choices. Responsibility refers to the duties and activities that an individual is employed to perform. Authority refers to legitimate power to give commands and make final decisions specific to a given position.

18. A male older-adult patient expresses concern and anxiety about decreased penile firmness during an erection. What is the nurse's best response? a.Tell the patient that libido will always decrease, as well as the sexual desires. b.Tell the patient that touching should be avoided unless intercourse is planned. c.Tell the patient that heterosexuality will help maintain stronger libido. d.Tell the patient that this change is expected in aging adults.

ANS: D Aging men typically experience an erection that is less firm and shorter acting and have a less forceful ejaculation. Testosterone lessens with age and sometimes (not always) leads to a loss of libido. However, for both men and women sexual desires, thoughts, and actions continue throughout all decades of life. Sexuality involves love, warmth, sharing, and touching, not just the act of intercourse. Touch complements traditional sexual methods or serves as an alternative sexual expression when physical intercourse is not desired or possible. Clearly not all older adults are heterosexual, and there is emerging research on older adult, lesbian, gay, bisexual, and transgender individuals and their health care needs.

14. A nurse is caring for a patient in the last stages of dying. Which finding indicates the nurse needs to prepare the family for death? a. Redness of skin b. Clear-colored urine c. Tense muscles tone d. Cheyne-Stokes breathing

ANS: D Altered breathing such as Cheyne-Stokes pattern, apnea, labored, or irregular breathing is a sign of impending death. Cyanotic, pallor, or mottling of skin occurs. Urine is decreased and a dark color. Decreased muscle tone, relaxed jaw muscles, and sagging mouth also occur.

3. The nurse is caring for a patient in the postanesthesia care unit. The patient has developed profuse bleeding from the surgical site, and the surgeon has determined the need to return to the operative area. How will the nurse classify this procedure? a. Major b. Urgent c. Elective d. Emergency

ANS: D An emergency procedure must be done immediately to save a life or preserve the function of a body part. An example would be repair of a perforated appendix, repair of a traumatic amputation, or control of internal hemorrhaging. An urgent procedure is necessary for a patient's health and often prevents additional problems from developing. An example would be excision of a cancerous tumor, removal of a gallbladder for stones, or vascular repair for an obstructed artery. An elective procedure is performed on the basis of the patient's choice; it is not essential and is not always necessary for health. An example would be a bunionectomy, plastic surgery, or hernia reconstruction. A major procedure involves extensive reconstruction or alteration in body parts; it poses great risks to well-being. An example would be a coronary artery bypass or colon resection.

16. A nurse is prioritizing care for four patients. Which patient should the nurse see first? a. A patient needing teaching about medications b. A patient with a healed abdominal incision c. A patient with a slight temperature d. A patient with difficulty breathing

ANS: D An immediate threat to a patient's survival or safety must be addressed first, like difficulty breathing. Teaching, healed incision, and slight temperature are not immediate needs.

11. Which action should the nurse take first during the initial phase of implementation? a.Determine patient outcomes and goals. b.Prioritize patient's nursing diagnoses. c.Evaluate interventions. d.Reassess the patient.

ANS: D Assessment is a continuous process that occurs each time the nurse interacts with a patient. During the initial phase of implementation, reassess the patient. Determining the patient's goals and prioritizing diagnoses take place in the planning phase before choosing interventions. Evaluation is the last step of the nursing process.

23. A patient's father died a week ago. Both the patient and the patient's spouse talk about the death. The patient's spouse is experiencing headaches and fatigue. The patient is having trouble sleeping, has no appetite, and gets choked up most of the time. How should the nurse interpret these findings as the basis for a follow-up assessment? a. The patient is dying and the spouse is angry. b. The patient is ill and the spouse is malingering. c. Both the patient and the spouse are likely in denial. d. Both the patient and the spouse are likely grieving.

ANS: D Both are likely grieving from the loss of the patient's father. Symptoms of normal grief include headache, fatigue, insomnia, appetite disturbance, and choking sensation. Different people manifest different symptoms. There is no data to support the spouse is angry or malingering. There is no data to support the patient is dying or ill. Denial is assessed when the person cannot accept the loss; both talked about the loss.

4. A nurse manager discovers that the readmission rate of hospitalized patients is very high on the hospital unit. The nurse manager desires improved coordination of care and accountability for cost-effective quality care. Which nursing care delivery model is best suited for these needs? a. Team nursing b. Total patient care c. Primary nursing d. Case management

ANS: D Case management is a care approach that coordinates and links health care services to patients and families while streamlining costs. In team nursing, the RN assumes the role of group or team leader and leads a team made up of other RNs, practical nurses, and nursing assistive personnel. Total patient care involves an RN being responsible for all aspects of care for one or more patients. The primary nursing model of care delivery was developed to place RNs at the bedside and improve the accountability of nursing for patient outcomes and the professional relationships among staff members.

2. A nurse is overseeing the care of patients with severe diabetes and patients with heart failure to improve cost-effectiveness and quality of care. Which nursing care delivery model is the nurse using? a. Team nursing b. Total patient care c. Primary nursing d. Case management

ANS: D Case management is unique because clinicians, either as individuals or as part of a collaborative group, oversee the management of patients with specific, complex health problems or are held accountable for some standard of cost management and quality. Case management is a care approach that coordinates and links health care services to patients and families while streamlining costs. In the team nursing care model, the RN assumes the role of group or team leader and leads a team made up of other RNs, practical nurses, and nursing assistive personnel. Total patient care involves an RN being responsible for all aspects of care for one or more patients. The primary nursing model of care delivery was developed to place RNs at the bedside and improve the accountability of nursing for patient outcomes and the professional relationships among staff members.

15. The nurse is encouraging a reluctant postoperative patient to deep breathe and cough. Which explanation can the nurse provide that may encourage the patient to comply? a. "If you don't deep breathe and cough, you will get pneumonia." b. "You will need to cough only a few times during this shift." c. "Let's try clearing the throat because that will work just as well." d. "Deep breathing and coughing will clear out the anesthesia."

ANS: D Deep breathing and coughing expel retained anesthetic gases and facilitate a patient's return to consciousness. Although it is correct that a patient may experience atelectasis and pneumonia if deep breathing and coughing are not performed, the way this is worded sounds threatening and could be communicated in a more therapeutic manner. Deep breathing and coughing are encouraged every 2 hours while the patient is awake. Just clearing the throat does not remove mucus from deeper airways.

14. An older-adult patient in no acute distress reports being less able to taste and smell. What is the nurse's best response to this information? a.Notify the health care provider immediately to rule out cranial nerve damage. b.Schedule the patient for an appointment at a smell and taste disorders clinic. c.Perform testing on the vestibulocochlear nerve and a hearing test. d.Explain to the patient that diminished senses are normal findings.

ANS: D Diminished taste and smell senses are common findings in older adults. Scheduling an appointment at a smell and taste disorders clinic, testing the vestibulocochlear nerve, or an attempt to rule out cranial nerve damage is unnecessary at this time as per the information provided.

19. A nurse lets the transplant coordinator make a request for organ and tissue donation from the patient's family. What is the primary rationale for the nurse's action? a. The nurse is not as knowledgeable as the coordinator. b. The nurse is uncomfortable asking the family. c. The nurse does not want to upset the family. d. The nurse is following a federal law.

ANS: D In accordance with federal law, a specially trained professional (e.g., transplant coordinator or social worker) makes requests for organ and tissue donation at the time of every death. Given the complex and sensitive nature of such requests, only specially trained personnel make the requests. Although the nurse may be less knowledgeable than the coordinator, uncomfortable asking the family, or not wanting to upset the family, the primary rationale is to be in accordance with federal law.

16. The nurse and the nursing assistive personnel are assisting a postoperative patient to turn in bed. To assist in minimizing discomfort, which instruction should the nurse provide to the patient? a. "Close your eyes and think about something pleasant." b. "Hold your breath and count to three." c. "Grab my shoulders with your hands." d. "Place your hand over your incision."

ANS: D Instruct the patient to place the right hand over the incisional area to splint it, providing support and minimizing pulling during turning. Closing one's eyes, holding one's breath, and holding the nurse's shoulders do not help support the incision during a turn.

13. Which assessment of a patient who is 1 day postsurgery to repair a hip fracture requires immediate nursing intervention? a. Patient ate 40% of clear liquid breakfast. b. Patient's oral temperature is 98.9° F. c. Patient states, "I did not realize I would be so tired after this surgery." d. Patient reports severe pain 30 minutes after receiving pain medication.

ANS: D It is important to prioritize in all caregiving situations because it allows you to see relationships among patient problems and avoid delays in taking action that possibly leads to serious complications for a patient. The nurse needs to report severe pain that is unrelieved by pain medication to the health care provider. The nurse needs to recognize and differentiate normal from abnormal findings and set priorities. Eating 40% of breakfast, having a slightly elevated temperature, and being tired the day after surgery are expected findings following surgery and do not require immediate intervention.

16. A nurse is caring for a dying patient. One of the nurse's goals is to promote dignity and validation of the dying person's life. Which action will the nurse take to best achieve this goal? a. Take pictures of visitors. b. Provide quiet visiting time. c. Call the organ donation coordinator. d. Listen to family stories about the person.

ANS: D Listening to family members' stories validates the importance of the dying individual's life and reinforces the dignity of the person's life. Taking pictures of visitors does not address the value of a person's life. Calling organ donation and providing private visiting time are components of the dying process, but they do not validate a dying person's life.

5. In preparation for the eventual death of a female hospice patient of the Muslim faith, the nurse organizes a meeting of all hospice caregivers. A plan of care to be followed when this patient dies is prepared. Which information will be included in the plan? a. Prepare the body for autopsy. b. Prepare the body for cremation. c. Allow male Muslims to care for the body after death has occurred. d. Allow female Muslims to care for the body after death has occurred.

ANS: D Muslims of the same gender prepare the body for burial. Muslim faith discourages cremation and autopsy to preserve the sanctity of the soul of the deceased and promote burial as soon as possible after death.

6. An older patient has fallen and suffered a hip fracture. As a consequence, the patient's family is concerned about the patient's ability to care for self, especially during this convalescence. What should the nurse do? a.Stress that older patients usually ask for help when needed. b.Inform the family that placement in a nursing center is a permanent solution. c.Tell the family to enroll the patient in a ceramics class to maintain quality of life. d.Provide information and answer questions as family members make choices among care options.

ANS: D Nurses help older adults and their families by providing information and answering questions as they make choices among care options. Some older adults deny functional declines and refuse to ask for assistance with tasks that place their safety at great risk. The decision to enter a nursing center is never final, and a nursing center resident sometimes is discharged to home or to another less-acute residence. What defines quality of life varies and is unique for each person.

3. A terminally ill patient is experiencing constipation secondary to pain medication. Which is the best method for the nurse to improve the patient's constipation problem? a. Contact the health care provider to discontinue pain medication. b. Administer enemas twice daily for 7 days. c. Massage the patient's abdomen. d. Use a laxative.

ANS: D Opioid medication is known to slow peristalsis, which places the patient at high risk for constipation. Laxatives are indicated for opioid-induced constipation. Massaging the patient's abdomen may cause further discomfort. Discontinuing pain medication is inappropriate for a terminally ill patient. Enema administration twice a day is not the best step in the treatment of opioid-induced constipation.

2. A cancer patient asks the nurse what the criteria are for hospice care. Which information should the nurse share with the patient? a. It is for those needing assistance with pain management. b. It is for those having a terminal illness, such as cancer. c. It is for those with completion of an advance directive. d. It is for those expected to live less than 6 months.

ANS: D Patients accepted into a hospice program usually have less than 6 months to live. Patients with a terminal illness are not eligible until that point. Palliative care provides assistance with pain management when a patient is not eligible for hospice care. An advance directive can be completed by any person, even those who are healthy.

19. A patient asks the nurse what the term polypharmacy means. Which information should the nurse share with the patient? a.This is multiple side effects experienced when taking medications. b.This is many adverse drug effects reported to the pharmacy. c.This is the multiple risks of medication effects due to aging. d.This is concurrent use of many medications.

ANS: D Polypharmacy refers to the concurrent use of many medications. It does not have anything to do with side effects, adverse drug effects, or risks of medication use due to aging.

14. A nurse has a transactional leader as a manager. Which finding will the nurse anticipate from working with this leader? a. Increased turnover rate b. Increased patient mortality rate c. Increased rate of medication errors d. Increased level of patient satisfaction

ANS: D Research has found that on nursing units where the nurse manager uses transactional leadership there is an increased level of patient satisfaction, a lower patient mortality rate, and a lower rate of medication errors. Turnover rate is decreased since staff retention is increased with transformational leadership.

25. A 72-year-old woman was recently widowed. She worked as a teller at a bank for 40 years and has been retired for the past 5 years. She never learned how to drive. She lives in a rural area that does not have public transportation. Which psychosocial change does the nurse focus on as a priority? a.Sexuality b.Retirement c.Environment d.Social isolation

ANS: D The highest priority at this time is the potential for social isolation. This woman does not know how to drive and lives in a rural community that does not have public transportation. All of these factors contribute to her social isolation. Other possible changes she may be going through right now include sexuality related to her advanced age and recent death of her spouse; however, this is not the priority at this time. She has been retired for 5 years, so this is also not an immediate need. She may eventually experience needs related to environment, but the data do not support this as an issue at this time.

10. A nurse manager conducts rounds on the unit and discovers that expired stock medicine is still in the cabinet despite the e-mail that was sent stating that it had to be discarded. The staff nurse dress code is not being adhered to as requested in the same e-mail. Several staff nurses deny having received the e-mail. Which action should the nurse manager take? a. Close the staff lounge. b. Enforce a stricter dress code. c. Include the findings on each staff member's annual evaluation. d. Place a hard copy of announcements and unit policies in each staff member's mailbox.

ANS: D The identified problem is lack of staff communication. Sending an e-mail was not effective; therefore, giving each staff member a hard copy along with e-mailing is another approach the manager can take. An effective manager uses a variety of approaches to communicate quickly and accurately to all staff. For example, many managers distribute biweekly or monthly newsletters of ongoing unit or facility activities. Including the findings on evaluations, closing the lounge, and enforcing stricter dress codes do not address the problem.

18. The nurse and the nursing assistive personnel (NAP) are caring for a group of postoperative patients who need turning, coughing, deep breathing, incentive spirometer, and leg exercises. Which task will the nurse assign to the NAP? a. Teach postoperative exercises. b. Do nothing associated with postoperative exercises. c. Document in the medical record when exercises are completed. d. Inform the nurse if the patient is unwilling to perform exercises.

ANS: D The nurse can delegate to the NAP to encourage patients to practice postoperative exercises regularly after instruction and to inform the nurse if the patient is unwilling to perform these exercises. The skills of demonstrating and teaching postoperative exercises and documenting are not within the scope of practice for the nursing assistant. Doing nothing is not appropriate.

6. A patient recovering from a leg fracture after a fall reports having dull pain in the affected leg and rates it as a 7 on a 0 to 10 scale. The patient is not able to walk around in the room with crutches because of leg discomfort. Which nursing intervention is priority? a.Assist the patient to walk in the room with crutches. b.Obtain a walker for the patient. c.Consult physical therapy. d.Administer pain medication.

ANS: D The patient's pain is a 7, indicating the priority is pain relief (administer pain medication). Acute pain is the priority because the nurse can address the problem of immobility after the patient receives adequate pain relief. Assisting the patient to walk or obtaining a walker will not address the pain the patient is experiencing.

9. A palliative team is caring for a dying patient in severe pain. Which action is the priority? a. Provide postmortem care for the patient. b. Support the patient's nurse in grieving. c. Teach the patient the stages of grief. d. Enhance the patient's quality of life.

ANS: D The primary goal of palliative care is to help patients and families achieve the best quality of life. Providing support for the patient's nurse is not the primary obligation when the patient is experiencing severe pain. Not all collaborative team members on the palliative team would be able to provide postmortem care, as is the case for nutritionists, social workers, and pharmacists. Teaching about stages of grief should not be the focus when severe pain is present.

3. A nurse develops a nursing diagnostic statement for a patient with a medical diagnosis of pneumonia with chest x-ray results of lower lobe infiltrates. Which nursing diagnosis did the nurse write? a.Ineffective breathing pattern related to pneumonia b.Risk for infection related to chest x-ray procedure c.Risk for deficient fluid volume related to dehydration d.Impaired gas exchange related to alveolar-capillary membrane changes

ANS: D The related to factor of alveolar-capillary membrane changes is accurately written because it is a patient response to the disease process of pneumonia that the nurse can treat. The related to factor should be the cause of the problem (nursing diagnosis) that a nurse can address. The related to factors of dehydration and pneumonia are all medical diagnoses that the nurse cannot change. A diagnostic test or a chronic dysfunction is not an etiology or a condition that a nursing intervention is able to treat.

28. The nurse is caring for a patient in the preoperative holding area of an ambulatory surgery center. Which nursing action will be most appropriate for this area? a. Count the sterile surgical instruments. b. Empty the urinary drainage bag. c. Check the surgical dressing. d. Apply a warm blanket.

ANS: D The temperature in the preoperative holding area and in adjacent operating suites is usually cold. Offer the patient an extra warm blanket. Counts are taken by the circulating and scrub nurses in the operating room. Emptying a urinary drainage bag and checking the surgical dressing occur in the postanesthesia care unit, not in the holding area.

11. A patient has a fecal impaction. Which portion of the colon will the nurse assess? a. Descending b. Transverse c. Ascending d. Rectum

ANS: D A fecal impaction is a collection of hardened feces wedged in the rectum that cannot be expelled. It results from unrelieved constipation. Feces at this point in the colon contain the least amount of moisture. Feces found in the ascending, transverse, and descending colon still consist mostly of liquid and do not form a hardened mass. DIF:Apply (application)REF:1152

20.A chemotherapy patient has gained 5 pounds in 2 days. Which assessment question by the nurse is most appropriate? a. "Are you following any weight loss program?" b. "How many calories a day do you consume?" c. "Do you have dry mouth or feel thirsty?" d. "How many times a day do you urinate?"

ANS: D A rapid gain in weight usually indicates extracellular volume (ECV) excess if the person began with normal ECV. Asking the patient about urination habits will help determine whether the body is trying to excrete the excess fluid or if renal dysfunction is contributing to ECV excess. This is too rapid a weight gain to be dietary; it is fluid retention. Asking about following a weight loss program will not help determine the cause of the problem. Caloric intake does not account for rapid weight changes. Dry mouth and thirst accompany ECV deficit, which would be associated with rapid weight loss. DIF:Apply (application)REF:940 | 948-949

33.A nurse is assisting the health care provider in inserting a central line. Which action indicates the nurse is following the recommended bundle protocol to reduce central line-associated bloodstream infections (CLABSI)? a. Preps skin with povidone-iodine solution. b. Suggests the femoral vein for insertion site. c. Applies double gloving without hand hygiene. d. Uses chlorhexidine skin antisepsis prior to insertion.

ANS: D A recommended bundle at insertion of a central line is hand hygiene prior to catheter insertion; use of maximum sterile barrier precautions upon insertion; chlorhexidine skin antisepsis prior to insertion and during dressing changes; avoidance of the femoral vein for central venous access for adults; and daily evaluation of line necessity, with prompt removal of non-essential lines. Povidone-iodine is not recommended. DIF:Apply (application)REF:957

8. The nurse values autonomy above all other principles. Which patient assignment will the nurse find most difficult to accept? A. Teenager in labor who requests epidural anesthesia B. Middle-aged father of three with an advance directive declining life support C. Elderly patient who requires dialysis D. Family elder who is making the decisions for a 30-year-old female member

ANS: D Autonomy refers to freedom from external control. A person who values autonomy highly may find it difficult to accept situations where the patient is not the primary decision maker regarding his or her care. A teenager requesting an epidural, a father with an advanced directive, and an elderly patient requiring dialysis all describe a patient or family that can make their own decisions and choices regarding care

40.The nurse is caring for a patient with hyperkalemia. Which body system assessment is the priority? a. Gastrointestinal b. Neurological c. Respiratory d. Cardiac

ANS: D Cardiac is the priority. Hyperkalemia places the patient at risk for potentially serious dysrhythmias and cardiac arrest. Potassium balance is necessary for cardiac function. Respiratory is the priority with hypokalemia. Monitoring of gastrointestinal and neurological systems would be indicated for other electrolyte imbalances. DIF:Apply (application)REF:941-942 | 956

9. A patient expresses concerns over having black stool. The fecal occult test is negative. Which response by the nurse is most appropriate? a. "This is probably a false negative we should rerun the test." b. "You should schedule a colonoscopy as soon as possible." c. "Are you under a lot of stress?" d. "Do you take iron supplements?"

ANS: D Certain medications and supplements, such as iron, can alter the color of stool (black or tarry). Since the fecal occult test is negative, bleeding is not occurring. The fecal occult test takes three separate samples over a period of time and is a fairly reliable test. A colonoscopy is health prevention screening that should be done every 5 to 10 years it is not the nurse's initial priority. Stress alters GI motility and stool consistency, not color. DIF:Analyze (analysis)REF:1156-1157

While recovering from a severe illness, a hospitalized patient states that he wants to change his living will, which he signed nine months ago. Which response by the nurse is most appropriate? a. "Check with your admitting health care provider whether a copy is on your chart." b. "Have you talked with your attorney recently about a living will?" c. "Your living will can be changed only once each calendar year." d. "Let me check with someone here in the hospital who can assist you."

ANS: D Each health care facility has personnel who are familiar with the state laws and can assist the patient in revising a living will. They may be in the admissions or risk management department. Checking with the health care provider about the presence of a living will on the chart has nothing to do with the patient's desire to change the living will. The question states that the patient wants to change his living will. Asking whether he has talked to his lawyer recently is a closed-ended question that passes the responsibility to someone else, that is, the attorney, and does not address the patient's current desire to change the living will. It is the nurse's responsibility to find an appropriate person in the facility to assist the patient. A living will can be changed whenever the patient decides to change it, as long as the patient is competent.

7. A patient with a hip fracture is having difficulty defecating into a bedpan while lying in bed. Which action by the nurse will assist the patient in having a successful bowel movement? a. Preparing to administer a barium enema b. Withholding narcotic pain medication c. Administering laxatives to the patient d. Raising the head of the bed

ANS: D Lying in bed is an unnatural position raising the head of the bed assists the patient into a more normal position that allows proper contraction of muscles for elimination. Laxatives would not give the patient control over bowel movements. A barium enema is a diagnostic test, not an intervention to promote defecation. Pain relief measures should be given however, preventative action should be taken to prevent constipation. DIF:Apply (application)REF:1151

20. A nurse is preparing a patient for a magnetic resonance imaging (MRI) scan. Which nursing action is most important? a. Ensuring that the patient does not eat or drink 2 hours before the examination. b. Administering a colon cleansing product 6 hours before the examination. c. Obtaining an order for a pain medication before the test is performed. d. Removing all of the patient's metallic jewelry.

ANS: D No jewelry or metal products should be in the same room as an MRI machine because of the high-power magnet used in the machine. The patient needs to be NPO 4 to 6 hours before the examination. Colon cleansing products are not necessary for MRIs. Pain medication is not needed before the examination is performed. DIF:Apply (application)REF:1158

A patient's plan of care includes the goal of increasing mobility this shift. As the patient is ambulating to the bathroom at the beginning of the shift, the patient suffers a fall. Which initial action will the nurse take next to revise the plan of care? a. Consult physical therapy. b. Establish a new plan of care. c. Set new priorities for the patient. d. Assess the patient.

ANS: D Nurses revise a plan when a patient's status changes; assessment is the first step. Know also that a plan of care is dynamic and changes as the patient's needs change. Asking physical therapy to assist the patient is premature before assessing the patient and awaiting the health care provider's orders. The nurse may not need to disregard all previous diagnoses. Some diagnoses may still apply, but the patient needs to be assessed first. Setting new priorities is not recommended before assessment and establishing diagnoses.

33. The nurse is caring for a patient with Clostridium difficile. Which nursing actions will have the greatest impact in preventing the spread of the bacteria? a. Appropriate disposal of contaminated items in biohazard bags b. Monthly in-services about contact precautions c. Mandatory cultures on all patients d. Proper hand hygiene techniques

ANS: D Proper hand hygiene is the best way to prevent the spread of bacteria. Soap and water are mandatory. Monthly in-services place emphasis on education, not on action. Biohazard bags are appropriate but cannot be used on every item that C. difficile comes in contact with, such as a human. Mandatory cultures are expensive and unnecessary and would not prevent the spread of bacteria. DIF:Apply (application)REF:1152

A nursing assessment for a patient with a spinal cord injury leads to several pertinent nursing diagnoses. Which nursing diagnosis is the highest priority for this patient? a. Risk for impaired skin integrity b. Risk for infection c. Spiritual distress d. Reflex urinary incontinence

ANS: D Reflex urinary incontinence is highest priority. If a patient's incontinence is not addressed, then the patient is at higher risk of impaired skin integrity and infection. Remember that the Risk for diagnoses are potential problems. They may be prioritized higher in some cases but not in this situation. Spiritual distress is an actual diagnosis, but the adverse effects that could result from not assisting the patient with urinary elimination take priority in this case.

A patient has reduced muscle strength following a left-sided stroke and is at risk for falling. Which intervention is most appropriate for the nursing diagnostic statement Risk for falls? a. Keep all side rails down at all times. b. Encourage patient to remain in bed most of the shift. c. Place patient in room away from the nurses' station if possible. d. Assist patient into and out of bed every 4 hours or as tolerated.

ANS: D Risk for falls is a risk (potential) nursing diagnosis; therefore, the nurse needs to implement actions that will prevent a fall. Assisting the patient into and out of bed is the most appropriate intervention to prevent the patient from falling. Encouraging activity builds muscle strength, and helping the patient with transfers ensures patient safety. Encouraging the patient to stay in bed will not promote muscle strength. Decreased muscle strength is the risk factor placing the patient in jeopardy of falling. The side rails should be up, not down, according to agency policy. This will remind the patient to ask for help to get up and will keep the patient from rolling out of bed. The patient should be placed near the nurses' station, so a staff member can quickly get to the room and assist the patient if necessary.

32.A nurse is caring for a patient who is receiving peripheral intravenous (IV) therapy. When the nurse is flushing the patient's peripheral IV, the patient reports pain. Upon assessment, the nurse notices a red streak that is warm to the touch. What is the nurse's initial action? a. Record a phlebitis grade of 4. b. Assign an infiltration grade. c. Apply moist compress. d. Discontinue the IV.

ANS: D The IV site has phlebitis. The nurse should discontinue the IV. The phlebitis score is 3. The site has phlebitis, not infiltration. A moist compress may be needed after the IV is discontinued. DIF:Apply (application)REF:960 | 982

1. The nurse is teaching a health class about the gastrointestinal tract. The nurse will explain that which portion of the digestive tract absorbs most of the nutrients? a. Ileum b. Cecum c. Stomach d. Duodenum

ANS: D The duodenum and jejunum absorb most nutrients and electrolytes in the small intestine. The ileum absorbs certain vitamins, iron, and bile salts. Food is broken down in the stomach. The cecum is the beginning of the large intestine. DIF:Understand (comprehension)REF:1150

A patient's son decides to stay at the bedside while his father is confused. When developing the plan of care for this patient, what should the nurse do? a. Individualize the care plan only according to the patient's needs. b. Request that the son leave at bedtime, so the patient can rest. c. Suggest that a female member of the family stay with the patient. d. Involve the son in the plan of care as much as possible.

ANS: D The family is often a resource to help the patient meet health care goals. Family should be included in the plan of care as much as possible. Meeting some of the family's needs as well as the patient's needs will possibly improve the patient's level of wellness. The son should not be asked to leave if at all possible. In some situations, it may be best that family members not remain in the room, but no evidence in the question stem suggests that this is the case in this situation. The suggestion of asking a female member to stay is not a justified action without a legitimate reason. No reason is given in this question stem for such a suggestion.

A registered nurse administers pain medication to a patient suffering from fractured ribs. Which type of nursing intervention is this nurse implementing? a. Collaborative b. Independent c. Interdependent d. Dependent

ANS: D The nurse does not have prescriptive authority to order pain medications, unless the nurse is an advanced practice nurse. The intervention is therefore dependent. Administering a medication, implementing an invasive procedure (e.g., inserting a Foley catheter, starting an intravenous [IV] infusion), and preparing a patient for diagnostic tests are examples of health care provider-initiated interventions. A collaborative, or an interdependent, intervention involves therapies that require combined knowledge, skill, and expertise from multiple health care professionals. Nurse-initiated interventions are the independent nursing interventions, or actions that a nurse initiates without supervision or direction from others.

13. The nurse begins a shift assessment by examining a surgical dressing that is saturated with serosanguineous drainage on a patient who had open abdominal surgery yesterday (or 1 day ago). Which type of assessment approach is the nurse using? a. Gordon's Functional Health Patterns b. Activity-exercise pattern assessment c. General to specific assessment d. Problem-oriented assessment

ANS: D The nurse is not doing a complete, general assessment and then focusing on specific problem areas. Instead, the nurse focuses immediately on the problem at hand (dressing and drainage from surgery) and performs a problem-oriented assessment. Utilizing Gordon's Functional Health Patterns is an example of a structured database-type assessment technique that includes 11 patterns to assess. The nurse in this question is performing a specific problem-oriented assessment approach, not a general approach. The nurse is not performing an activity-exercise pattern assessment in this question.

12. While the patient's lower extremity, which is in a cast, is assessed, the patient tells the nurse about an inability to rest at night. The nurse disregards this information, thinking that no correlation has been noted between having a leg cast and developing restless sleep. Which action would have been best for the nurse to take? a. Tell the patient to just focus on the leg and cast right now. b. Document the sleep patterns and information in the patient's chart. c. Explain that a more thorough assessment will be needed next shift. d. Ask the patient about usual sleep patterns and the onset of having difficulty resting.

ANS: D The nurse must use critical thinking skills in this situation to assess first in this situation. The best response is to gather more assessment data by asking the patient about usual sleep patterns and the onset of having difficulty resting. The nurse should assess before documenting and should not ignore the patient's report of a problem or postpone it till the next shift.

37.A patient has an acute intravascular hemolytic reaction to a blood transfusion. After discontinuing the blood transfusion, which is the nurse's next action? a. Discontinue the IV catheter. b. Return the blood to the blood bank. c. Run normal saline through the existing tubing. d. Start normal saline at TKO rate using new tubing.

ANS: D The nurse should first attach new tubing and begin running in normal saline at a rate to keep the vein open, in case any medications need to be delivered through an IV site. The existing tubing should not be used because that would infuse the blood in the tubing into the patient. It is necessary to preserve the IV catheter in place for IV access to treat the patient. After the patient has been assessed and stabilized, the blood can be returned to the blood bank. DIF:Apply (application)REF:963-964

12. The nurse is managing bowel training for a patient. To which patient is the nurse most likely providing care? a. A 25-year-old patient with diarrhea b. A 30-year-old patient with Clostridium difficile c. A 40-year-old patient with an ileostomy d. A 70-year-old patient with stool incontinence

ANS: D The patient with chronic constipation or fecal incontinence secondary to cognitive impairment may benefit from bowel training, also called habit training. An ileostomy, diarrhea, and C. difficile all relate to uncontrollable bowel movements, for which no method can be used to set up a schedule of elimination. DIF:Analyze (analysis)REF:1168

14. Which patient is most likely to have difficulty with the ethical concept of autonomy? A. 18-year-old patient in labor B. 35-year-old patient with appendicitis C. 53-year-old patient with pancreatitis D. 78-year-old patient with rheumatoid arthritis

ANS: D The principle of autonomy refers to freedom from external control and includes commitment to include patients in decisions about their care. People from different generations have differing expectations regarding inclusion in their care. Often, patients who are part of the Silent Generation (born 1925-1945) value formality and authority, which may make them less comfortable with making their own healthcare decisions.

5.A nurse is preparing to start peripheral intravenous (IV) therapy. In which order will the nurse perform the steps starting with the first one? 1. Clean site. 2. Select vein. 3. Apply tourniquet. 4. Release tourniquet. 5. Reapply tourniquet. 6. Advance and secure. 7. Insert vascular access device. a. 1, 3, 2, 7, 5, 4, 6 b. 1, 3, 2, 5, 7, 6, 4 c. 3, 2, 1, 5, 7, 6, 4 d. 3, 2, 4, 1, 5, 7, 6

ANS: D The steps for inserting an intravenous catheter are as follows: Apply tourniquet; select vein; release tourniquet; clean site; reapply tourniquet; insert vascular access device; and advance and secure. DIF:Apply (application)REF:971-974

41.Which assessment finding will the nurse expect for a patient with the following laboratory values: sodium 145 mEq/L, potassium 4.5 mEq/L, calcium 4.5 mg/dL? a. Weak quadriceps muscles b. Decreased deep tendon reflexes c. Light-headedness when standing up d. Tingling of extremities with possible tetany

ANS: D This patient has hypocalcemia because the normal calcium range is 8.4 to 10.5 mg/dL. Hypocalcemia causes muscle tetany, positive Chvostek's sign, and tingling of the extremities. Sodium and potassium values are within their normal ranges: sodium 135 to 145 mEq/L; potassium 3.5 to 5.0 mEq/L. Light-headedness when standing up is a manifestation of ECV deficit or sometimes hypokalemia. Weak quadriceps muscles are associated with potassium imbalances. Decreased deep tendon reflexes are related to hypercalcemia or hypermagnesemia. DIF:Analyze (analysis)REF:935 | 942 | 950

10. A nurse argues that we need to reform our healthcare system because we have a large number of people who are uninsured and end up needing expensive emergent care when low-cost measures could have prevented their illnesses. What ethical framework is she using to make this case? A. Deontology B. Ethics of care C. Feminist ethics D. Utilitarianism

ANS: D Utilitarianism is a system of ethics that believes that value is determined by usefulness. This system of ethics focuses on the outcome of the greatest good for the greatest number of people. Deontology would not look to consequences of actions. The ethics of care would not be helpful because consensus on this issue is not achievable. Relationships, which are an important component of feminist ethics, are not addressed in this case.

4. When people work together to solve ethical dilemmas, individuals must examine their own values. This step is crucial to ensure that A. The group identifies the one correct solution. B. Fact is separated from opinion. C. Judgmental attitudes are not provoked. D. Different perspectives are respected

ANS: D Values are personal beliefs that influence opinions. To be able to negotiate differences in opinions, the nurse must first be clear about personal values, which will influence behaviors, decisions, and actions. Ethical dilemmas are a problem in that no one right solution exists.

A nursing student in the final term of nursing school is overheard by a nursing faculty member telling another student that she got to insert a nasogastric tube in the emergency department while she was working as a nursing assistant. What advice is best for the nursing faculty member to give to the nursing student? a. "Just be careful when you are doing new procedures and make sure you are following directions by the nurse." b. "Review your procedures before you go to work, so you will be prepared to do them if you have a chance." c. "The nurse should not have allowed you to insert the nasogastric tube because something bad could have happened." d. "You are not allowed to perform any procedures other than those in your job description even with the nurse's permission."

ANS: D When nursing students work as nursing assistants or nurse's aides when not attending classes, they should not perform tasks that do not appear in a job description for a nurse's aide or assistant. The nursing student should always follow the directions of the nurse, unless doing so violates the institution's guidelines or job description under which the nursing student was hired. The nursing student should be able to safely complete the procedures delegated as a nursing assistant, and reviewing those not done recently is a good idea, but it has nothing to do with the situation. This option does not address the situation that the nursing student acted outside the job description for the nursing assistant position. The focus of the discussion between the nursing faculty member and the nursing student should be on following the job description under which the nursing student is working.

The new nurse is caring for six patients in this shift. After completing their assessments, the nurse asks where to begin in developing care plans for these patients. Which statement is an appropriate suggestion by another nurse? a. "Choose all the interventions and perform them in order of time needed for each one." b. "Make sure you identify the scientific rationale for each intervention first." c. "Decide on goals and outcomes you have chosen for the patients." d. "Begin with the highest priority diagnoses, then select appropriate interventions."

ANS: D Work from your plan of care and use patients' priorities to organize the order for delivering interventions and organizing documentation of care. When developing a plan of care, the nurse needs to rank the nursing diagnoses in order of priority, then select appropriate interventions. Choosing all the interventions should take place after ranking of the diagnoses, and interventions should be prioritized by patient needs, not just by time. The chosen interventions should be evidence based with scientific rationales, but the diagnoses need to be prioritized first to prioritize interventions. Goals for a patient should be mutually set, not just chosen by the nurse.

1. A registered nurse (RN) is the group leader of practical nurses and nursing assistive personnel. Which nursing care model is the RN using? a. Case management b. Total patient care c. Primary nursing d. Team nursing

ANS: D In team nursing, the RN assumes the role of group or team leader and leads a team made up of other RNs, practical nurses, and nursing assistive personnel. Case management is a care approach that coordinates and links health care services to patients and families while streamlining costs. Total patient care involves an RN being responsible for all aspects of care for one or more patients. The primary nursing model of care delivery was developed to place RNs at the bedside and improve the accountability of nursing for patient outcomes and the professional relationships among staff members.

7. The nurse is caring for a group of postoperative patients on the surgical unit. Which patient assessments indicate the nurse needs to follow up? (Select all that apply.) a. Patient with abdominal surgery has patent airway. b. Patient with knee surgery has approximated incision. c. Patient with femoral artery surgery has strong pedal pulse. d. Patient with lung surgery has 20 mL/hr of urine output via catheter. e. Patient with bladder surgery has bloody urine within the first 12 hours. f. Patient with appendix surgery has thready pulse and blood pressure is 90/60.

ANS: D, F Thready pulse, low blood pressure, and urine output of 20 mL/hr need to have follow-up by the nurse. Hemorrhage results in a fall in blood pressure; elevated heart and respiratory rates; thready pulse; cool, clammy, pale skin; and restlessness. Notify the surgeon if these changes occur. If the patient has a urinary catheter, there should be a continuous flow of urine of approximately 30 to 50 mL/hr in adults; this patient requires follow-up since the output is 20 mL/hr. All the rest are normal findings. A patent airway, a strong distal pulse, and approximated incision are all normal findings. Surgery involving portions of the urinary tract normally causes bloody urine for at least 12 to 24 hours, depending on the type of surgery.

The nurse is caring for a patient who has an order to change a dressing twice a day, at 0600 and 1800. At 1400, the nurse notices that the dressing is saturated and leaking. What is the nurse's next action? a. Wait and change the dressing at 1800 as ordered. b. Revise the plan of care and change the dressing now. c. Reassess the dressing and the wound in 2 hours. d. Discontinue the plan of care for wound care.

B (Because the dressing is saturated and leaking, the nurse needs to revise the plan of care and change the dressing now. Reflection-in-action involves a nurse's ability to recognize how a patient is responding and then adjusting interventions as a result. A nurse will either change the frequency of an intervention, change how the intervention is delivered, or select a new intervention. Waiting until 1800 or for another 2 hours is not appropriate because assessment data reflect that the dressing is saturated and needs to be changed now. Data are insufficient to support discontinuing the plan of care. Instead, data at this time indicate the need for revision of the plan of care.)

A nurse is providing education to a patient about self-administering subcutaneous injections. The patient demonstrates the self-injection. Which type of indicator did the nurse evaluate? a. Health status b. Health behavior c. Psychological self-control d. Health service utilization

B (Health behavior involves demonstrating a psychomotor skill such as self-injection. Health status is a clinical indicator such as exercise tolerance or blood pressure control. The skill is psychomotor, not psychological self-control. Health service utilization is readmission within 30 days or emergency department use.)

A nurse is modifying a patient's care plan after evaluation of patient care. In which order, starting with the first step, will the nurse perform the tasks? 1. Revise nursing diagnosis. 2. Reassess blood pressure reading. 3. Retake blood pressure after medication. 4. Administer new blood pressure medication. 5. Change goal to blood pressure less than 140/90. a. 1, 5, 2, 4, 3 b. 2, 1, 5, 4, 3 c. 4, 3, 1, 5, 2 d. 5, 4, 5, 1, 2

B (If a nursing diagnosis is unresolved or if you determine that a new problem has perhaps developed, reassessment is necessary. A complete reassessment of patient factors relating to an existing nursing diagnosis and etiology is necessary when modifying a plan. After reassessment, determine which nursing diagnoses are accurate for the situation; revise as needed. When revising a care plan, review the goals and expected outcomes for necessary changes after the diagnosis. Then evaluate and revise interventions as needed.)

The nurse is evaluating whether patient goals and outcomes have been met for a patient with physical mobility problems due to a fractured leg. Which finding indicates the patient has met an expected outcome? a. The nurse provides assistance while the patient is walking in the hallways. b. The patient is able to ambulate in the hallway with crutches. c. The patient will deny pain while walking in the hallway. d. The patient's level of mobility will improve.

B (The patient's being able to ambulate in the hallway with crutches is an expected outcome of nursing care. The outcomes of nursing practice are the measurable conditions of patient, family or community status, behavior, or perception. These outcomes are the criteria used to judge success in delivering nursing care. The option stating, "The patient's level of mobility will improve" is a broader goal statement. The nurse's assisting a patient to ambulate is an intervention. The patient's denying pain is an expected outcome for pain, not for physical mobility problems.)

A new nurse is confused about using evaluative measures when caring for patients and asks the charge nurse for an explanation. Which response by the charge nurse is most accurate? a. "Evaluative measures are multiple-page documents used to evaluate nurse performance." b. "Evaluative measures include assessment data used to determine whether patients have met their expected outcomes and goals." c. "Evaluative measures are used by quality assurance nurses to determine the progress a nurse is making from novice to expert nurse." d. "Evaluative measures are objective views for completion of nursing interventions."

B (You conduct evaluative measures to determine if your patients met expected outcomes, not if nursing interventions were completed. Evaluative measures are assessment skills and techniques. Evaluative measures are not multiple-page documents, and they are used to assess the patient's status, not the nurse's performance or progress from novice to expert.)

A nurse is getting ready to discharge a patient who has a problem with physical mobility. What does the nurse need to do before discontinuing the patient's plan of care? a. Determine whether the patient has transportation to get home. b. Evaluate whether patient goals and outcomes have been met. c. Establish whether the patient has a follow-up appointment scheduled. d. Ensure that the patient's prescriptions have been filled to take home.

B (You evaluate whether the results of care match the expected outcomes and goals set for a patient before discontinuing a patient's plan of care. The patient needs transportation, but that does not address the patient's mobility status. Whether the patient has a follow-up appointment and ensuring that prescriptions are filled do not evaluate the problem of mobility.)

The nurse is caring for a patient who has an open wound and is evaluating the progress of wound healing. Which priority action will the nurse take? a. Ask the nursing assistive personnel if the wound looks better. b. Document the progress of wound healing as "better" in the chart. c. Measure the wound and observe for redness, swelling, or drainage. d. Leave the dressing off the wound for easier access and more frequent assessments.

C ( You examine the results of care by using evaluative measures, which are assessment skills and techniques (e.g., observations, physiological measurements, use of measurement scales, and patient interview). The nurse performs evaluative measures, such as completing a wound assessment, to evaluate wound healing. Nurses do not delegate assessment to nursing assistive personnel. Documenting "better" is subjective and does not objectively describe the wound. Leaving the dressing off for the nurse's benefit of easier access is not a part of the evaluation process.)

A new nurse asks the preceptor to describe the primary purpose of evaluation. Which statement made by the nursing preceptor is most accurate? a. "An evaluation helps you determine whether all nursing interventions were completed." b. "During evaluation, you determine when to downsize staffing on nursing units." c. "Nurses use evaluation to determine the effectiveness of nursing care." d. "Evaluation eliminates unnecessary paperwork and care planning."

C (Evaluation is a methodical approach for determining if nursing implementation was effective in influencing a patient's progress or condition in a favorable way. During evaluation, you do not simply determine whether nursing interventions were completed. The evaluation process is not used to determine when to downsize staffing or how to eliminate paperwork and care planning.)

A nurse is evaluating the goal of acceptance of body image in a young teenage girl. Which statement made by the patient is the best indicator of progress toward the goal? a. "I'm worried about what those other girls will think of me." b. "I can't wear that color. It makes my hips stick out." c. "I'll wear the blue dress. It matches my eyes." d. "I will go to the pool next summer."

C (The nurse is evaluating the improvement in body image. The only positive comment made is that the patient is wearing the blue dress to match her eyes. Worrying about others, making my hips stick out, and going to the pool next summer do not reflect positive changes in body image.)

After assessing the patient and identifying the need for headache relief, the nurse administers acetaminophen for the patient's headache. Which action by the nurse is priority for this patient? a. Eliminate headache from the nursing care plan. b. Direct the nursing assistive personnel to ask if the headache is relieved. c. Reassess the patient's pain level in 30 minutes. d. Revise the plan of care.

C (The nurse's priority action for this patient is to evaluate whether the nursing intervention of administering acetaminophen was effective. The nurse does not have enough evaluative data at this point to determine whether headache needs to be discontinued. Assessment is the nurse's responsibility and is not to be delegated to nursing assistive personnel. The nurse does not have enough evaluative data to determine whether the patient's plan of care needs to be revised.)

A patient was recently diagnosed with pneumonia. The nurse and the patient have established a goal that the patient will not experience shortness of breath with activity in 3 days with an expected outcome of having no secretions present in the lungs in 48 hours. Which evaluative measure will the nurse use to demonstrate progress toward this goal? a. No sputum or cough present in 4 days b. Congestion throughout all lung fields in 2 days c. Shallow, fast respirations 30 breaths per minute in 1 day d. Lungs clear to auscultation following use of inhaler

D (In this case, the patient's goal is to not experience shortness of breath with activity in 3 days. If the lung sounds are clear following use of inhaler, the nurse can determine that the patient is making progress toward achieving the expected outcome. One way for the nurse to evaluate the expected outcome is to assess the patient's lung sounds. Goals are broad statements that describe changes in a patient's condition or behavior. Expected outcomes are measurable criteria used to evaluate goal achievement. When an outcome is met, you know that the patient is making progress toward goal achievement. The time frame of 4 days in the first option is not appropriate because this time frame exceeds the time frame stated in the goal. Congestion indicates fluid in the lungs, and a respiratory rate of 30 breaths per minute is elevated/abnormal. This indicates that the patient is still probably experiencing shortness of breath and secretions in the lungs.)

A nurse is evaluating goals and expected outcomes for a confused patient. Which finding indicates positive progress toward resolving the confusion? a. Patient wanders halls at night. b. Patient's side rails are up with bed alarm activated. c. Patient denies pain while ambulating with assistance. d. Patient correctly states names of family members in the room.

D (The goal for this patient would address a decrease or absence of confusion. Thus, one possible sign that a patient's confusion is improving is seen when a patient can correctly state the names of family members in the room. You examine the results of care by using evaluative measures that relate to goals and expected outcomes. Keeping the side rails up and using a bed alarm are interventions to promote patient safety and prevent falls. The patient's denying pain indicates positive progress toward resolving pain. The patient's wandering the halls is a sign of confusion.)

The nurse is evaluating whether a patient's turning schedule was effective in preventing the formation of pressure ulcers. Which finding indicates success of the turning schedule? a. Staff documentation of turning the patient every 2 hours b. Presence of redness only on the heels of the patient c. Patient's eating 100% of all meals d. Absence of skin breakdown

D (To determine whether a turning schedule is successful, the nurse needs to assess for the presence of skin breakdown. Redness on any part of the body, including only the patient's heels, indicates that the turning schedule was not successful. Documentation of interventions does not evaluate whether patient outcomes were met. Eating 100% of meals does not evaluate the effectiveness of a turning schedule.)

2.Which assessments will alert the nurse that a patient's IV has infiltrated? (Select all that apply.) a. Edema of the extremity near the insertion site b. Reddish streak proximal to the insertion site c. Skin discolored or pale in appearance d. Pain and warmth at the insertion site e. Palpable venous cord f. Skin cool to the touch

ANS: A, C, F Infiltration results in skin that is edematous near the IV insertion site. Skin is cool to the touch and may be pale or discolored. Pain, warmth, erythema, a reddish streak, and a palpable venous cord are all symptoms of phlebitis. DIF:Understand (comprehension)REF:960-961

9. A nurse has already set the agenda during a patient-centered interview. What will the nurse do next? a. Begin with introductions. b. Ask about the chief concerns or problems. c. Explain that the interview will be over in a few minutes. d. Tell the patient "I will be back to administer medications in 1 hour."

ANS: B After setting the agenda, the nurse should conduct the actual interview and proceed with data collection, such as asking about the patient's current chief concerns or problems. Introductions occur before setting the agenda. Begin an interview by introducing yourself and your position and explaining the purpose of the interview. Your aim is to set an agenda for how you will gather information about a patient's current chief concerns or problems. The termination phase includes telling the patient when the interview is nearing an end. Telling the patient that medications will be given later when the nurse returns would typically take place during the termination phase of the interview.

16. During a severe respiratory epidemic, the local health care organizations decide to give health care providers priority access to ventilators over other members of the community who also need that resource. Which philosophy would give the strongest support for this decision? A. Feminist ethics B. Utilitarianism C. Deontology D. Ethics of care

ANS: B Focusing on the greatest good for the most people, the organizations decide to ensure that as many health care workers as possible will survive to care for other members of the community.

A nurse notices that his neighbor's preschool children are often playing on the sidewalk and in the street unsupervised and repeatedly takes them back to their home and talks with the parent available, but the situation continues. What immediate action by the nurse is mandated by law? a. Talk with both parents about safety needs of their children. b. Contact the appropriate community child protection agency. c. Tell the parents that the authorities will be contacted shortly. d. Take pictures of the children to support the overt child abuse.

ANS: B The nurse has a duty to report this situation to protect the children. Any health care professional who does not report suspected child abuse or neglect may be liable for civil or criminal legal action. The person making the report has legal immunity if the report is made in good faith. Talking with the parents is not mandated by law. There is no obligation to tell the parents that they will be reported to authorities. There is no obligation for the nurse to take pictures of the children.

4. The nurse is planning care for a group of patients. Which task will the nurse assign to the nursing assistive personnel (NAP)? a. Performing the first postoperative pouch change b. Maintaining a nasogastric tube c. Administering an enema d. Digitally removing stool

ANS: C The skill of administering an enema can be delegated to an NAP. The skill of inserting and maintaining a nasogastric (NG) tube cannot be delegated to an NAP. The nurse should do the first postoperative pouch change. Digitally removing stool cannot be delegated to nursing assistive personnel. DIF:Apply (application)REF:1170

39.A nurse is preparing to start a blood transfusion. Which type of tubing will the nurse obtain? a. Two-way valves to allow the patient's blood to mix and warm the blood transfusing b. An injection port to mix additional electrolytes into the blood c. A filter to ensure that clots do not enter the patient d. An air vent to let bubbles into the blood

ANS: C When administering a transfusion you need an appropriate-size IV catheter and blood administration tubing that has a special in-line filter. The patient's blood should not be mixed with the infusion blood. Air bubbles should not be allowed to enter the blood. The only substance compatible with blood is normal saline; no additives should be mixed with the infusing blood. DIF:Apply (application)REF:962

Which nursing actions will the nurse perform in the evaluation phase of the nursing process? (Select all that apply.) a. Set priorities for patient care. b. Determine whether outcomes or standards are met. c. Ambulate patient 25 feet in the hallway. d. Document results of goal achievement. e. Use self-reflection and correct errors.

B, D, E (The expected outcomes established during planning are the standards against which you judge whether goals have been met and if care is successful. You evaluate whether the results of care match the expected outcomes and goals set for a patient. Documentation and reporting are important parts of evaluation because it is crucial to share information about a patient's progress and current status. Using self-reflection and correcting errors are indicators a nurse is performing evaluation. Setting priorities is part of planning, and ambulating with a patient in the hallway is an intervention, so it is included in the implementation step of the nursing process.)

A nurse determines that the patient's condition has improved and has met expected outcomes. Which step of the nursing process is the nurse exhibiting? a. Assessment b. Planning c. Implementation d. Evaluation

D (Evaluation, the final step of the nursing process, is crucial to determine whether, after application of the first four steps of the nursing process, a patient's condition or well-being improves and if goals have been met. Assessment, the first step of the process, includes data collection. Planning, the third step of the process, involves setting priorities, identifying patient goals and outcomes, and selecting nursing interventions. During implementation, nurses carry out nursing care, which is necessary to help patients achieve their goals.)

A nurse completes a thorough database and carries out nursing interventions based on priority diagnoses. Which action will the nurse take next? a. Assessment b. Planning c. Implementation d. Evaluation

D (Evaluation, the final step of the nursing process, is crucial to determine whether, after application of the first four steps of the nursing process, a patient's condition or well-being improves. Assessment involves gathering information about the patient. During the planning phase, patient outcomes are determined. Implementation involves carrying out appropriate nursing interventions.)


Related study sets

HKIN 446 Module 3 Lecture Shoudler and Arm Group 4 Fill in Blank Pt 5

View Set

MBA650 - Ch 7 Economies of Scale and Scope

View Set

describing feelings of anger, sadness, anxiety, shock and confusion

View Set